MED SURG FINAL 5/2/17

Lakukan tugas rumah & ujian kamu dengan baik sekarang menggunakan Quizwiz!

The best nursing intervention for a patient in shock who has a nursing diagnosis of fear related to perceived threat of death is to a. arrange for the hospital pastoral care staff to visit the patient. b. ask the health care provider to prescribe a sedative drug for the patient. c. leave the patient alone with family members whenever possible. d. place the patient's call bell where it can be easily reached.

D

The nurse is caring for a patient admitted with a urinary tract infection and sepsis. Which information obtained in the assessment indicates a need for a change in therapy? a. The patient is restless and anxious. b. The patient has a heart rate of 134. c. The patient has hypotonic bowel sounds. d. The patient has a temperature of 94.1° F.

D

The nurse evaluates that fluid resuscitation for a 70 kg patient in shock is effective on finding that the patient's a. urine output is 40 ml over the last hour. b. hemoglobin is within normal limits. c. CVP has decreased. d. mean arterial pressure (MAP) is 65 mm Hg.

A

When assessing the hemodynamic information for a newly admitted patient in shock of unknown etiology, the nurse will anticipate administration of large volumes of crystalloids when the a. cardiac output is increased and the central venous pressure (CVP) is low. b. pulmonary artery wedge pressure (PAWP) is increased, and the urine output is low. c. heart rate is decreased, and the systemic vascular resistance is low. d. cardiac output is decreased and the PAWP is high.

A

While assessing a patient in shock who has an arterial line in place, the nurse notes a drop in the systolic BP from 92 mm Hg to 76 mm Hg when the head of the patient's bed is elevated to 75 degrees. This finding indicates a need for a. additional fluid replacement. b. antibiotic administration. c. infusion of a sympathomimetic drug. d. administration of increased oxygen.

A

When teaching a patient infected with HIV regarding transmission of the virus to others, which statement made by the patient would indicate a need for further teaching? A "I will need to isolate any tissues I use so as not to infect my family." B "I will notify all of my sexual partners so they can get tested for HIV." C "Unprotected sexual contact is the most common mode of transmission." D "I do not need to worry about spreading this virus to others by sweating at the gym."

A "I will need to isolate any tissues I use so as not to infect my family." Correct HIV is not spread casually. The virus cannot be transmitted through hugging, dry kissing, shaking hands, sharing eating utensils, using toilet seats, or attending school with an HIV-infected person. It is not transmitted through tears, saliva, urine, emesis, sputum, feces, or sweat.

Norepinephrine (Levophed) has been ordered for the patient in hypovolemic shock. Before administering the drug, the nurse ensures that the a. patient's heart rate is less than 100. b. patient has received adequate fluid replacement. c. patient's urine output is within normal range. d. patient is not receiving other sympathomimetic drugs.

B

The nurse caring for a patient in shock notifies the health care provider of the patient's deteriorating status when the patient's ABG results include a. pH 7.48, PaCO2 33 mm Hg. b. pH 7.33, PaCO2 30 mm Hg. c. pH 7.41, PaCO2 50 mm Hg. d. pH 7.38, PaCO2 45 mm Hg.

B

To monitor a patient with severe acute pancreatitis for the early organ damage associated with MODS, the most important assessments for the nurse to make are a. stool guaiac and bowel sounds. b. lung sounds and oxygenation status. c. serum creatinine and urinary output. d. serum bilirubin levels and skin color.

B

While caring for a seriously ill patient, the nurse determines that the patient may be in the compensatory stage of shock on finding a. cold, mottled extremities. b. restlessness and apprehension. c. a heart rate of 120 and cool, clammy skin. d. systolic BP less than 90 mm Hg.

B

The nurse would recognize which of the following clinical manifestations as suggestive of sepsis? A) Respiratory rate of seven breaths per minute B) Hyperglycemia in the absence of diabetes C) Sudden diuresis unrelated to drug therapy D) Bradycardia with sudden increase in blood pressure

B Hyperglycemia in patients with no history of diabetes is a diagnostic criterion for sepsis. Oliguria, not diuresis, typically accompanies sepsis along with tachycardia and tachypnea.

A patient's localized infection has progressed to the point where septic shock is now suspected. Which of the following is an appropriate treatment modality for this patient? A) Insulin infusion B) Aggressive fluid resuscitation C) Intravenous administration of epinephrine D) Administration of nitrates and β-adrenergic blockers

B Patients in septic shock require large amounts of fluid replacement. Nitrates and β-adrenergic blockers are most often used in the treatment of patients in cardiogenic shock. Epinephrine is indicated in anaphylactic shock, and insulin infusion is not normally necessary in the treatment of septic shock (but can be).

When caring for a patient in acute septic shock, the nurse would anticipate A) Administering osmotic and/or loop diuretics. B) Infusing large amounts of intravenous fluids. C) Administering intravenous diphenhydramine (Benadryl). D) Assisting with insertion of a ventricular assist device (VAD).

B Septic shock is characterized by a decreased circulating blood volume. Volume expansion with the administration of intravenous fluids is the cornerstone of therapy. The administration of diuretics is inappropriate. VADs are useful for cardiogenic shock, not septic shock. Diphenhydramine (Benadryl) may be used for anaphylactic shock, but would not be helpful with septic shock.

The patient is admitted to the ED with fever, swollen lymph glands, sore throat, headache, malaise, joint pain, and diarrhea. What nursing measures will help identify the need for further assessment of the cause of this patient's manifestations (select all that apply)? A Assessment of lung sounds Incorrect B Assessment of sexual behavior Correct C Assessment of living conditions Incorrect D Assessment of drug and syringe use Correct E Assessment of exposure to an ill person Incorrect

B Assessment of sexual behavior Correct D Assessment of drug and syringe use Correct With these symptoms, assessing this patient's sexual behavior and possible exposure to shared drug equipment will identify if further assessment for the HIV virus should be made or the manifestations are from some other illness (e.g., lung sounds and living conditions may indicate further testing for TB).

Which of the following laboratory findings fits with a diagnosis of cardiogenic shock? A) Decreased liver enzymes B) Increased white blood cells C) Increased blood urea nitrogen and creatinine levels D) Decreased red blood cells, hemoglobin, and hematocrit

C The renal hypoperfusion that accompanies cardiogenic shock results in increased BUN and creatinine levels. Impaired perfusion of the liver results in increased liver enzymes while red blood cell indices are typically normal because of relative hypovolemia. White blood cell levels do not typically rise in cardiogenic shock.

If surgery is needed (for bladder cancer), what elements will be considered in deciding the type of surgery?

The patient's ability and readiness to learn must be considered. His anxiety and fear may affect the teaching. Psychosocial aspects of living with a stoma may affect the choice. Acceptance of the surgery and of alterations in body image is needed to ensure the patient's best adjustment. If a continent diversion is used, the patient must be able to catheterize the pouch every 4 to 6 hours and irrigate it daily. Orthotopic bladder reconstruction may be considered if the cancer did not involve the bladder neck or urethra and the patient has normal renal and liver function, a longer than 1- to 2-year life expectancy, adequate motor skills, and no history of inflammatory bowel disease or colon cancer. Obese patients are not good candidates for orthotopic bladder reconstruction.

What does staging of bladder tumors indicate?

The staging of bladder cancer is determined by the depth of invasion of the bladder wall and surrounding tissue. Stage II indicates that the tumor has grown into the muscle layer of the bladder but not passed completely through it. The TNM grading system indicates the characteristics of the tumor (T), the nodal involvement (N), and the presence of distant metastasis (M). (See Chapter 16 for TNM classification.)

The nurse teaches a 21-year-old female patient who came to the clinic to discuss interventions to prevent a recurrence of urinary tract infections. Which statement, if made by the patient, indicates that teaching was effective? a. "I will urinate before and after having intercourse." Correct b. "I will use vinegar as a vaginal douche every week." c. "I should drink three 8-ounce glasses of water daily." d. "I can stop the antibiotics when symptoms disappear."

a. "I will urinate before and after having intercourse." The woman should empty her bladder before and after sexual intercourse. She should avoid vaginal douches and maintain adequate oral fluid intake (15 mL per pound of body weight). All of the antibiotics should be taken as prescribed even if symptoms are no longer present.

Number in sequence the following ascending pathologic changes that occur in the urinary tract in the presence of a bladder outlet obstruction. a. Hydronephrosis b. Reflux of urine into ureter c. Bladder detrusor muscle hypertrophy d. Ureteral dilation e. Renal atrophy f. Vesicoureteral reflux g. Large residual urine in bladder h. Chronic pyelonephritis

a. 6; b. 3; c. 1; d. 4; e. 8; f. 5; g. 2; h. 7

Which nursing diagnosis is a priority in the care of a patient with renal calculi? a. Acute pain b. Risk for constipation c. Deficient fluid volume d. Risk for powerlessness

a. Acute pain Urinary stones are associated with severe abdominal or flank pain. Deficient fluid volume is unlikely to result from urinary stones, whereas constipation is more likely to be an indirect consequence rather than a primary clinical manifestation of the problem. The presence of pain supersedes powerlessness as an immediate focus of nursing care.

Which classification of urinary tract infection (UTI) is described as infection of the renal parenchyma, renal pelvis, and ureters? a. Upper UTI b. Lower UTI c. Complicated UTI d. Uncomplicated UTI

a. An upper urinary tract infection (UTI) affects the renal parenchyma, renal pelvis, and ureters. A lower UTI is an infection of the bladder and/or urethra. A complicated UTI exists in the presence of obstruction, stones, or preexisting diseases. An uncomplicated UTI occurs in an otherwise normal urinary tract.

When obtaining a nursing history from a patient with cancer of the urinary system, what does the nurse recognize as a risk factor associated with both kidney cancer and bladder cancer? a. Smoking b. Family history of cancer c. Chronic use of phenacetin d. Chronic, recurrent nephrolithiasis

a. Both cancer of the kidney and cancer of the bladder are associated with smoking. A family history of renal cancer is a risk factor for kidney cancer and cancer of the bladder has been associated with the use of phenacetin-containing analgesics and recurrent upper UTIs.

Besides being mixed with struvite or oxalate stones, what characteristic is associated with calcium phosphate calculi? a. Associated with alkaline urine b. Genetic autosomal recessive defect c. Three times as common in women as in men d. Defective gastrointestinal (GI) and kidney absorption

a. Calcium phosphate calculi are typically mixed with struvite or oxalate stones and related to alkaline urine. Cystine calculi are associated with a genetic autosomal recessive defect and defective GI and kidney absorption of cystine. Struvite calculi are three to four times more common in women than in men.

The patient with type 2 diabetes has a second UTI within one month of being treated for a previous UTI. Which medication should the nurse expect to teach the patient about taking for this infection? a. Ciprofloxacin (Cipro) b. Fosfomycin (Monurol) c. Nitrofurantoin (Macrodantin) d. Trimethoprim/ sulfamethoxazole (Bactrim)

a. Ciprofloxacin (Cipro) This UTI is a complicated UTI because the patient has type 2 diabetes and the UTI is recurrent. Ciprofloxacin (Cipro) would be used for a complicated UTI. Fosfomycin (Monurol), nitrofurantoin (Macrodantin), and trimethoprim/sulfamethoxazole (Bactrim) should be used for uncomplicated UTIs.

The nurse is preparing to care for a burn client scheduled for an escharotomy procedure being performed for a third degree circumferential arm burn. The nurse understands that the anticipated therapeutic outcome of the escharotomy is: a. return of distal pulses b. brisk bleeding from the site c. decreasing edema formation d. formation of granulation tissue

a. Escharotomies arepreformed to relieve the compartment syndrome that can occur when edema forms under nondistensible eschar in a circumferential third degree burn.

A client is admitted to a burn intensive care unit with extensive full thickness burns. What should be the nurse's initial concern? a. fluid status b. risk for infection c. body image d. level of pain

a. in early burn care, the client's greatest need has to do with fluid resuscitation because of large volume fluid loss through the damaged skin.

A pt in the emergent phase of burn care for thermal burns on 20% of the total body surface area is unconscious. Which assessment data is the most important for the nurse's evaluation of the pt's injuries? a. condition of the oropharynx b. percentage of TBSA affected c. location of the pt in the fire d. comorbidities of the pt

a. the pt is likely to have suffered a smoke inhalation injury because thermal burns are caused by flames that emit smoke and because the pt is unconscious.

The male patient is Jewish, has a history of gout, and has been diagnosed with renal calculi. Which treatment will be used with this patient (select all that apply)? a. Reduce dietary oxalate b. Administer allopurinol c. Administer α-penicillamine d. Administer thiazide diuretics e. Reduce animal protein intake f. Reduce intake of milk products

b, e. This patient is most likely to have uric acid calculi, which have a high incidence in Jewish men, and gout is a predisposing factor. The treatment will include allopurinol and reducing animal protein intake to reduce purine, as uric acid is a waste product from purine metabolism. Reducing oxalate and using thiazide diuretics help to treat calcium oxalate calculi. Administration of α-penicillamine and tiopronin prevent cystine crystallization for cystine calculi. Reducing intake of milk products to reduce calcium intake may be used with calcium calculi.

When caring for a client with extensive burns, the nurse anticipates that pain medication will be administered via which route? a. oral b. IV c. IM d. Subq

b.

A patient is to undergo skin grafting with the use of cultured epithelial autografts full-thickness burns. The nurse explains to the patient that this treatment involves a) Shaving a split-thickness layer of the patient's skin to cover the burn wound. b) Using epidermal growth factor to cultivate cadaver skin for temporary wound coverage. c) Growing small specimens of the patient's skin into sheets to use as permanent skin coverage. d) Exposing animal skin to growth factors to decrease antigenicity so it can be used for permanent wound coverage.

c Rationale: Cultured epithelial autograft (CEA) is a method of obtaining permanent skin from a person with limited available skin for harvesting. CEA is grown from biopsy specimens obtained from the patient's own unburned skin.

A triage nurse in the ED admits a 50 year old male client with second degree burns on the anterior and posterior portions of both legs. Based on the Rule of Nines, what percentage of his body is burned? Record your answer using a whole number.

36 The anterior and posterior portions of one leg are 18%, if both legs are burned, the total is 36%.

An 80-kg patient with burns over 30% of total body surface area (TBSA) is admitted to the burn unit. Using the Parkland formula of 4 mL/kg/%TBSA, what is the IV infusion rate (mL/hour) for lactated Ringer's solution that the nurse will administer during the first 8 hours?

600 mL The Parkland formula states that patients should receive 4 mL/kg/%TBSA burned during the first 24 hours. Half of the total volume is given in the first 8 hours and then the last half is given over 16 hours: 4 80 30 = 9600 mL total volume; 9600/2 = 4800 mL in the first 8 hours; 4800 mL/8 hr = 600 mL/hr.

A patient in compensated septic shock has hemodynamic monitoring with a pulmonary artery catheter and an arterial catheter. Which information obtained by the nurse indicates that the patient is still in the compensatory stage of shock? a. The cardiac output is elevated. b. The central venous pressure (CVP) is increased. c. The systemic vascular resistance (SVR) is high. d. The PAWP is high.

A

A patient outcome that is appropriate for the patient in shock who has a nursing diagnosis of decreased cardiac output related to relative hypovolemia is a. urine output of 0.5 ml/kg/hr. b. decreased peripheral edema. c. decreased CVP. d. oxygen saturation 90% or more.

A

A patient with a myocardial infarction (MI) and cardiogenic shock has the following vital signs: BP 86/50, pulse 126, respirations 30. Hemodynamic monitoring reveals an elevated PAWP and decreased cardiac output. The nurse will anticipate a. administration of furosemide (Lasix) IV. b. titration of an epinephrine (Adrenalin) drip. c. administration of a normal saline bolus. d. assisting with endotracheal intubation.

A

All of these collaborative interventions are ordered by the health care provider for a patient stung by a bee who develops severe respiratory distress and faintness. Which one will the nurse administer first? a. Epinephrine (Adrenalin) b. Normal saline infusion c. Dexamethasone (Decadron) d. Diphenhydramine (Benadryl)

A

The nurse reminds the staff that standard precautions should be used when providing care for which type of patient? A All patients regardless of diagnosis Correct B Pediatric and gerontologic patients C Patients who are immunocompromised Incorrect D Patients with a history of infectious diseases

A All patients regardless of diagnosis Correct Standard precautions are designed for all care of all patients in hospitals and health care facilities.

The HIV-infected patient is taught health promotion activities including good nutrition; avoiding alcohol, tobacco, drug use, and exposure to infectious agents; keeping up to date with vaccines; getting adequate rest; and stress management. What is the rationale behind these interventions that the nurse knows? A Delaying disease progression B Preventing disease transmission C Helping to cure the HIV infection D Enabling an increase in self-care activities

A Delaying disease progression Correct These health promotion activities along with mental health counseling, support groups, and a therapeutic relationship with health care providers will promote a healthy immune system, which may delay disease progression. These measures will not cure HIV infection, prevent disease transmission, or increase self-care activities.

What should the nurse teach the patients in the assisted living facility to decrease their risk for antibiotic-resistant infection (select all that apply)? A Wash hands frequently. Correct B Take antibiotics as prescribed. Correct C Take the antibiotic until it is gone. Correct D Take antibiotics to prevent illnesses like colds. E Save leftover antibiotics to take if needed later.

A Wash hands frequently. Correct B Take antibiotics as prescribed. Correct C Take the antibiotic until it is gone. Correct To decrease the risk for antibiotic-resistant infections, people should wash their hands frequently, follow the directions when taking the antibiotics, finish the antibiotic, do not request antibiotics for colds or flu, do not save leftover antibiotics, or take antibiotics to prevent an illness without them being prescribed by a health care provider.

What surgery might be indicated if the chemotherapy is not effective (in eradicating bladder cancer)?

A cystectomy with urinary diversion would be indicated. This could be an incontinent urinary diversion or a continent urinary diversion.

Which statement by the nurse regarding continuous ambulatory peritoneal dialysis (CAPD) would be of highest priority when teaching a patient new to this procedure? A. "It is essential that you maintain aseptic technique to prevent peritonitis." B. "You will be allowed a more liberal protein diet once you complete CAPD." C. "It is important for you to maintain a daily written record of blood pressure and weight." D. "You will need to continue regular medical and nursing follow-up visits while performing CAPD."

A. "It is essential that you maintain aseptic technique to prevent peritonitis." Peritonitis is a potentially fatal complication of peritoneal dialysis, and thus it is imperative to teach the patient methods of preventing this from occurring. Although the other teaching statements are accurate, they do not have the potential for morbidity and mortality as does peritonitis, thus making that statement of highest priority.

Which is a priority nursing intervention for a patient during the acute phase of rheumatic fever? A. Administration of antibiotics as ordered B. Management of pain with opioid analgesics C. Encouragement of fluid intake for hydration D. Performance of frequent, active ROM exercises

A. Administration of antibiotics as ordered

A patient is admitted with myocarditis. While performing the initial assessment, which clinical s/sx might the nurse find (select all that apply): A. Angina B. Pleuritic chest pain C. Splinter hemorrhages D. Pericardial friction rub E. Presence of Osler's nodes

A. Angina B. Pleuritic chest pain D. Pericardial friction rub

A 78-year-old patient has Stage 3 CKD and is being taught about a low potassium diet. The nurse knows the patient understands the diet when the patient selects which foods to eat? A. Apple, green beans, and a roast beef sandwich B. Granola made with dried fruits, nuts, and seeds C. Watermelon and ice cream with chocolate sauce D. Bran cereal with ½ banana and milk and orange juice

A. Apple, green beans, and a roast beef sandwich When the patient selects an apple, green beans, and a roast beef sandwich, the patient demonstrates understanding of the low potassium diet. Granola, dried fruits, nuts and seeds, milk products, chocolate sauce, bran cereal, banana, and orange juice all have elevated levels of potassium, at or above 200 mg per 1/2 cup.

The nurse knows the patient with AKI has entered the diuretic phase when what assessments occur (select all that apply)? A. Dehydration B. Hypokalemia C. Hypernatremia D. BUN increases E. Serum creatinine increases

A. Dehydration B. Hypokalemia Dehydration, hypokalemia, and hyponatremia occur in the diuretic phase of AKI because the nephrons can excrete wastes but not concentrate urine. Therefore the serum BUN and serum creatinine levels also begin to decrease.

Which of the following diagnostic studies best differentiates the different types of cardiomyopathy? A. Echocardiography B. ABGs C. Cardiac catheterization D. Endomyocardial biopsy

A. Echocardiography

A key aspect of teaching for the patient on anticoagulation therapy includes which instructions? A. Monitor and report for any signs of bleeding B. Do not take acetaminophen (Tylenol) for a headache C. Decrease your dietary intake of food containing vitamin K D. Arrange to have blood drawn routinely to check drug levels

A. Monitor and report for any signs of bleeding

A patient with chronic heart failure and atrial fibrillation is treated with a digitalis glycoside and a loop diuretic. To prevent possible complications of this drug combination, the nurse needs to A. Monitor serum potassium levels B. Keep an accurate measure of intake and output C. Teach the patient about dietary restriction of potassium D. Withhold digitalis and notify provider if heart rate is irregular

A. Monitor serum potassium levels

The patient has a form of glomerular inflammation that is progressing rapidly. She is gaining weight, and the urine output is steadily declining. What is the priority nursing intervention? A. Monitor the patient's cardiac status. B. Teach the patient about hand washing. C. Obtain a serum specimen for electrolytes. D. Increase direct observation of the patient.

A. Monitor the patient's cardiac status The nurse's priority is to monitor the patient's cardiac status. With the rapidly progressing glomerulonephritis, renal function begins to fail and fluid, potassium, and hydrogen retention lead to hypervolemia, hyperkalemia, and metabolic acidosis. Excess fluid increases the workload of the heart, and hyperkalemia can lead to life-threatening dysrhythmias. Teaching about hand washing and observation of the patient are important nursing interventions but are not the priority. Electrolyte measurement is a collaborative intervention that will be done as ordered by the health care provider.

A patient is diagnosed with with mitral valve stenosis and new onset atrial fibrillation. Which interventions could the nurse delegate to nursing assisstive personnel (select all that apply)? A. Obtain and record daily weight B. Determine apical-radial pulse rate C. Observe for overt signs of bleeding D. Obtain and record vital signs, including pulse oximetry E. Teach patient how to purchase a Medic Alert bracelet

A. Obtain and record daily weight C. Observe for overt signs of bleeding D. Obtain and record vital signs, including pulse oximetry

Assessment of an IV cocaine user with infective endocarditis should focus on which s/sx (select all that apply): A. Retinal hemorrhages B. Splinter hemorrhages C. Presence of Osler's nodes D. Painless nodules on bony prominences E. Painless erythmatous macules on the palms and soles

A. Retinal hemorrhages B. Splinter hemorrhages C. Presence of Osler's nodes E. Painless erythmatous macules on the palms and soles

The nurse recognizes that primary manifestations of systolic failure include A. ⬇EF and ⬆PAWP B. ⬇PAWP and ⬆EF C.⬇pulmonary hypertension associate with normal EF D. ⬇afterload and ⬇left ventricular end-diastolic pressure

A. ⬇EF and ⬆PAWP

When teaching a patient about long term consequences of rheumatic fever, the nurse should discuss the possibility of A. Valvular heart disease B. Pulmonary hypertension C. Superior vena cava syndrome D. Hypertrophy of the right ventricle

A. Valvular heart disease

A patient who is having an acute exacerbation of multiple sclerosis has a prescription for methylprednisolone (Solu-Medrol) 160 mg IV. The label on the vial reads: methylprednisolone 125 mg in 2 mL. How many mL will the nurse administer?

ANS: 2.56 With a concentration of 125 mg/2 mL, the nurse will need to administer 2.56 mL to obtain 160 mg of methylprednisolone.

A patient's temperature has been 101° F (38.3° C) for several days. The patient's normal caloric intake to meet nutritional needs is 2000 calories per day. Knowing that the metabolic rate increases 7% for each Fahrenheit degree above 100° in body temperature, how many total calories should the patient receive each day?

ANS: 2140 calories

1. A 198-lb patient is to receive a dobutamine infusion at 5 mcg/kg/minute. The label on the infusion bag states: dobutamine 250 mg in 250 mL normal saline. When setting the infusion pump, the nurse will set the infusion rate at how many mL per hour?

ANS: 27 In order to administer the dobutamine at the prescribed rate of 5 mcg/kg/minute from a concentration of 250 mg in 250 mL, the nurse will need to infuse 27 mL/hour. DIF: Cognitive Level: Apply (application) REF: 1642 TOP: Nursing Process: Implementation MSC: NCLEX: Physiological Integrity

A patient who has an infected abdominal wound develops a temperature of 104° F (40° C). All the following interventions are included in the patient's plan of care. In which order should the nurse perform the following actions? (Put a comma and a space between each answer choice [A, B, C, D]). a. Administer IV antibiotics. b. Sponge patient with cool water. c. Perform wet-to-dry dressing change. d. Administer acetaminophen (Tylenol).

ANS: A, D, B, C The first action should be to administer the antibiotic because treating the infection that has caused the fever is the most important aspect of fever management. The next priority is to lower the high fever, so the nurse should administer acetaminophen to lower the temperature set point. A cool sponge bath should be done after the acetaminophen is given to lower the temperature further. The wet-to-dry dressing change will not have an immediate impact on the infection or fever and should be done last

In which order will the nurse take these actions when doing a dressing change for a partial-thickness burn wound on a patient's chest? (Put a comma and a space between each answer choice [A, B, C, D, E].) a. Apply sterile gauze dressing. b. Document wound appearance. c. Apply silver sulfadiazine cream. d. Administer IV fentanyl (Sublimaze). e. Clean wound with saline-soaked gauze

ANS: D, E, C, A, B Because partial-thickness burns are very painful, the nurse's first action should be to administer pain medications. The wound will then be cleaned, antibacterial cream applied, and covered with a new sterile dressing. The last action should be to document the appearance of the wound.

1. The health care provider orders the following interventions for a 67-kg patient who has septic shock with a BP of 70/42 mm Hg and oxygen saturation of 90% on room air. In which order will the nurse implement the actions? (Put a comma and a space between each answer choice [A, B, C, D, E].) a. Obtain blood and urine cultures. b. Give vancomycin (Vancocin) 1 g IV. c. Start norepinephrine (Levophed) 0.5 mcg/min. d. Infuse normal saline 2000 mL over 30 minutes. e. Titrate oxygen administration to keep O2 saturation >95%.

ANS: E, D, C, A, B The initial action for this hypotensive and hypoxemic patient should be to improve the oxygen saturation, followed by infusion of IV fluids and vasopressors to improve perfusion. Cultures should be obtained before administration of antibiotics. DIF: Cognitive Level: Analyze (analysis) REF: 1645 OBJ: Special Questions: Prioritization TOP: Nursing Process: Implementation MSC: NCLEX: Physiological Integrity

A nurse is caring for a patient in the emergent/resuscitative phase of burn injury. During this phase, the nurse should monitor for evidence of what alteration in laboratory values? A) Sodium deficit B) Decreased prothrombin time (PT) C) Potassium deficit D) Decreased hematocrit

ANS: A Feedback: Anticipated fluid and electrolyte changes that occur during the emergent/resuscitative phase of burn injury include sodium deficit, potassium excess, base-bicarbonate deficit, and elevated hematocrit. PT does not typically decrease.

A 64-year-old patient who has amyotrophic lateral sclerosis (ALS) is hospitalized with pneumonia. Which nursing action will be included in the plan of care? a. Assist with active range of motion (ROM). b. Observe for agitation and paranoia. c. Give muscle relaxants as needed to reduce spasms. d. Use simple words and phrases to explain procedures.

ANS: A ALS causes progressive muscle weakness, but assisting the patient to perform active ROM will help maintain strength as long as possible. Psychotic manifestations such as agitation and paranoia are not associated with ALS. Cognitive function is not affected by ALS, and the patient's ability to understand procedures will not be impaired. Muscle relaxants will further increase muscle weakness and depress respirations

A hospitalized 31-year-old patient with a history of cluster headache awakens during the night with a severe stabbing headache. Which action should the nurse take first? a. Start the ordered PRN oxygen at 6 L/min. b. Put a moist hot pack on the patient's neck. c. Give the ordered PRN acetaminophen (Tylenol). d. Notify the patient's health care provider immediately.

ANS: A Acute treatment for cluster headache is administration of 100% oxygen at 6 to 8 L/min. If the patient obtains relief with the oxygen, there is no immediate need to notify the health care provider. Cluster headaches last only 60 to 90 minutes, so oral pain medications have minimal effect. Hot packs are helpful for tension headaches but are not as likely to reduce pain associated with a cluster headache

11. Norepinephrine (Levophed) has been prescribed for a patient who was admitted with dehydration and hypotension. Which patient data indicate that the nurse should consult with the health care provider before starting the norepinephrine? a. The patient's central venous pressure is 3 mm Hg. b. The patient is in sinus tachycardia at 120 beats/min. c. The patient is receiving low dose dopamine (Intropin). d. The patient has had no urine output since being admitted.

ANS: A Adequate fluid administration is essential before administration of vasopressors to patients with hypovolemic shock. The patient's low central venous pressure indicates a need for more volume replacement. The other patient data are not contraindications to norepinephrine administration. DIF: Cognitive Level: Apply (application) REF: 1644 TOP: Nursing Process: Implementation MSC: NCLEX: Physiological Integrity

After change-of-shift report, which patient should the nurse assess first? a. Patient with myasthenia gravis who is reporting increased muscle weakness b. Patient with a bilateral headache described as "like a band around my head" c. Patient with seizures who is scheduled to receive a dose of phenytoin (Dilantin) d. Patient with Parkinson's disease who has developed cogwheel rigidity of the arms

ANS: A Because increased muscle weakness may indicate the onset of a myasthenic crisis, the nurse should assess this patient first. The other patients should also be assessed, but do not appear to need immediate nursing assessments or actions to prevent life-threatening complications.

15. A patient with septic shock has a BP of 70/46 mm Hg, pulse 136, respirations 32, temperature 104° F, and blood glucose 246 mg/dL. Which intervention ordered by the health care provider should the nurse implement first? a. Give normal saline IV at 500 mL/hr. b. Give acetaminophen (Tylenol) 650 mg rectally. c. Start insulin drip to maintain blood glucose at 110 to 150 mg/dL. d. Start norepinephrine (Levophed) to keep systolic blood pressure >90 mm Hg.

ANS: A Because of the low systemic vascular resistance (SVR) associated with septic shock, fluid resuscitation is the initial therapy. The other actions also are appropriate, and should be initiated quickly as well. DIF: Cognitive Level: Apply (application) REF: 1644-1645 OBJ: Special Questions: Prioritization TOP: Nursing Process: Implementation MSC: NCLEX: Physiological Integrity

When admitting a patient with stage III pressure ulcers on both heels, which information obtained by the nurse will have the most impact on wound healing? a. The patient takes insulin daily. b. The patient states that the ulcers are very painful. c. The patient has had the heel ulcers for the last 6 months. d. The patient has several old incisions that have formed keloids.

ANS: A Chronic insulin use indicates diabetes, which can interfere with wound healing. The persistence of the ulcers over the last 6 months is a concern, but changes in care may be effective in promoting healing. Keloids are not disabling or painful, although the cosmetic effects may be distressing for some patients. Actions to reduce the patient's pain will be implemented, but pain does not directly affect wound healing

Which infection, reported in the health history of a woman who is having difficulty conceiving, will the nurse identify as a risk factor for infertility? a. N. gonorrhoeae b. Treponema pallidum c. Condyloma acuminatum d. Herpes simplex virus type 2

ANS: A Complications of gonorrhea include scarring of the fallopian tubes, which can lead to tubal pregnancies and infertility. Syphilis, genital warts, and genital herpes do not lead to problems with conceiving, although transmission to the fetus (syphilis) or newborn (genital warts or genital herpes) is a concern

A 31-year-old woman who has multiple sclerosis (MS) asks the nurse about risks associated with pregnancy. Which response by the nurse is accurate? a. "MS symptoms may be worse after the pregnancy." b. "Women with MS frequently have premature labor." c. "MS is associated with an increased risk for congenital defects." d. "Symptoms of MS are likely to become worse during pregnancy."

ANS: A During the postpartum period, women with MS are at greater risk for exacerbation of symptoms. There is no increased risk for congenital defects in infants born of mothers with MS. Symptoms of MS may improve during pregnancy. Onset of labor is not affected by MS

A patient who has diabetes is admitted for an exploratory laparotomy for abdominal pain. When planning interventions to promote wound healing, what is the nurse's highest priority? a. Maintaining the patient's blood glucose within a normal range b. Ensuring that the patient has an adequate dietary protein intake c. Giving antipyretics to keep the temperature less than 102° F (38.9° C) d. Redressing the surgical incision with a dry, sterile dressing twice daily

ANS: A Elevated blood glucose will have an impact on multiple factors involved in wound healing. Ensuring adequate nutrition also is important for the postoperative patient, but a higher priority is blood glucose control. A temperature of 102° F will not impact adversely on wound healing, although the nurse may administer antipyretics if the patient is uncomfortable. Application of a dry, sterile dressing daily may be ordered, but frequent dressing changes for a wound healing by primary intention is not necessary to promote wound healing

1. A 78-kg patient with septic shock has a urine output of 30 mL/hr for the past 3 hours. The pulse rate is 120/minute and the central venous pressure and pulmonary artery wedge pressure are low. Which order by the health care provider will the nurse question? a. Give PRN furosemide (Lasix) 40 mg IV. b. Increase normal saline infusion to 250 mL/hr. c. Administer hydrocortisone (Solu-Cortef) 100 mg IV. d. Titrate norepinephrine (Levophed) to keep systolic BP >90 mm Hg.

ANS: A Furosemide will lower the filling pressures and renal perfusion further for the patient with septic shock. The other orders are appropriate. DIF: Cognitive Level: Apply (application) REF: 1640 TOP: Nursing Process: Implementation MSC: NCLEX: Physiological Integrity

Which patient will the nurse plan on teaching about the Gardasil vaccine? a. A 24-year-old female who has not been sexually active b. A 34-year-old woman who has multiple sexual partners c. A 19-year-old woman who is pregnant for the first time d. A 29-year-old woman who is in a monogamous relationship

ANS: A Gardasil is recommended for females ages 9 through 26, preferably those who have never been sexually active. It is not recommended for women during pregnancy or for older women

A patient has an open surgical wound on the abdomen that contains deep pink granulation tissue. How would the nurse document this wound? a. Red wound b. Yellow wound c. Full-thickness wound d. Stage III pressure ulcer

ANS: A The description is consistent with a red wound. A stage III pressure ulcer would expose subcutaneous fat. A yellow wound would have creamy colored exudate. A full-thickness wound involves subcutaneous tissue, which is not indicated in the wound description

The nurse advises a patient with myasthenia gravis (MG) to a. perform physically demanding activities early in the day. b. anticipate the need for weekly plasmapheresis treatments. c. do frequent weight-bearing exercise to prevent muscle atrophy. d. protect the extremities from injury due to poor sensory perception.

ANS: A Muscles are generally strongest in the morning, and activities involving muscle activity should be scheduled then. Plasmapheresis is not routinely scheduled, but is used for myasthenia crisis or for situations in which corticosteroid therapy must be avoided. There is no decrease in sensation with MG, and muscle atrophy does not occur because although there is muscle weakness, they are still used

A 46-year-old man who has had blood drawn for an insurance screening has a positive Venereal Disease Research Laboratory (VDRL) test. Which action should the nurse take next? a. Ask the patient about past treatment for syphilis. b. Explain the need for blood and spinal fluid cultures. c. Obtain a specimen for fluorescent treponemal antibody absorption (FAT-Abs) testing. d. Assess for the presence of chancres, flulike symptoms, or a bilateral rash on the trunk.

ANS: A Once antibody testing is positive for syphilis, the antibodies remain present for an indefinite period of time even after successful treatment, so the nurse should inquire about previous treatment before doing other assessments or testing. Culture, FAT-Abs testing, and assessment for symptoms may be appropriate, based on whether the patient has been previously treated for syphilis

A patient has been taking phenytoin (Dilantin) for 2 years. Which action will the nurse take when evaluating for adverse effects of the medication? a. Inspect the oral mucosa. b. Listen to the lung sounds. c. Auscultate the bowel tones. d. Check pupil reaction to light.

ANS: A Phenytoin can cause gingival hyperplasia, but does not affect bowel tones, lung sounds, or pupil reaction to light

A 29-year-old female patient is diagnosed with Chlamydia during a routine pelvic examination. The nurse knows that teaching regarding the management of the condition has been effective when the patient says which of the following? a. "My partner will need to take antibiotics at the same time I do." b. "Go ahead and give me the antibiotic injection, so I will be cured." c. "I will use condoms during sex until I finish taking all the antibiotics." d. "I do not plan on having children, so treating the infection is not important."

ANS: A Sex partners should be treated simultaneously to prevent reinfection. Chlamydia is treated with oral antibiotics. Abstinence from sexual intercourse is recommended for 7 days after treatment, and condoms should be recommended during all sexual contacts to prevent infection. Chronic pelvic pain, as well as infertility, can result from untreated Chlamydia

A patient arrives in the emergency department with a swollen ankle after an injury incurred while playing soccer. Which action by the nurse is most appropriate? a. Elevate the ankle above heart level. b. Apply a warm moist pack to the ankle. c. Assess the ankle's range of motion (ROM). d. Assess whether the patient can bear weight on the affected ankle.

ANS: A Soft tissue injuries are treated with rest, ice, compression, and elevation (RICE). Elevation of the ankle will decrease tissue swelling. Moving the ankle through the ROM will increase swelling and risk further injury. Cold packs should be applied the first 24 hours to reduce swelling. The nurse should not ask the patient to move or bear weight on the swollen ankle because immobilization of the inflamed or injured area promotes healing by decreasing metabolic needs of the tissues

The charge nurse observes the following actions being taken by a new nurse on the burn unit. Which action by the new nurse would require an intervention by the charge nurse? a. The new nurse uses clean latex gloves when applying antibacterial cream to a burn wound. b. The new nurse obtains burn cultures when the patient has a temperature of 95.2° F (35.1° C). c. The new nurse administers PRN fentanyl (Sublimaze) IV to a patient 5 minutes before a dressing change. d. The new nurse calls the health care provider for a possible insulin order when a nondiabetic patient's serum glucose is elevated.

ANS: A Sterile gloves should be worn when applying medications or dressings to a burn. Hypothermia is an indicator of possible sepsis, and cultures are appropriate. Nondiabetic patients may require insulin because stress and high calorie intake may lead to temporary hyperglycemia. Fentanyl peaks 5 minutes after IV administration, and should be used just before and during dressing changes for pain management

A 20-year-old woman who is being seen in the family medicine clinic for an annual physical exam reports being sexually active. The nurse will plan to teach the patient about a. testing for Chlamydia infection. b. immunization for herpes simplex. c. infertility associated with the human papillomavirus (HPV). d. the relationship between the herpes virus and cervical cancer.

ANS: A Testing for Chlamydia is recommended for all sexually active females under age 25 by the Centers for Disease Control and Prevention. HPV infection does not cause infertility. There is no vaccine available for herpes simplex, and herpes simplex infection does not cause cervical cancer

A patient who has type 1 diabetes plans to take a swimming class daily at 1:00 PM. The clinic nurse will plan to teach the patient to a. check glucose level before, during, and after swimming. b. delay eating the noon meal until after the swimming class. c. increase the morning dose of neutral protamine Hagedorn (NPH) insulin. d. time the morning insulin injection so that the peak occurs while swimming.

ANS: A The change in exercise will affect blood glucose, and the patient will need to monitor glucose carefully to determine the need for changes in diet and insulin administration. Because exercise tends to decrease blood glucose, patients are advised to eat before exercising. Increasing the morning NPH or timing the insulin to peak during exercise may lead to hypoglycemia, especially with the increased exercise.

19. During change-of-shift report, the nurse is told that a patient has been admitted with dehydration and hypotension after having vomiting and diarrhea for 4 days. Which finding is most important for the nurse to report to the health care provider? a. New onset of confusion b. Heart rate 112 beats/minute c. Decreased bowel sounds d. Pale, cool, and dry extremities

ANS: A The changes in mental status are indicative that the patient is in the progressive stage of shock and that rapid intervention is needed to prevent further deterioration. The other information is consistent with compensatory shock. DIF: Cognitive Level: Apply (application) REF: 1639 OBJ: Special Questions: Prioritization TOP: Nursing Process: Assessment MSC: NCLEX: Physiological Integrity

14. Which data collected by the nurse caring for a patient who has cardiogenic shock indicate that the patient may be developing multiple organ dysfunction syndrome (MODS)? a. The patient's serum creatinine level is elevated. b. The patient complains of intermittent chest pressure. c. The patient's extremities are cool and pulses are weak. d. The patient has bilateral crackles throughout lung fields.

ANS: A The elevated serum creatinine level indicates that the patient has renal failure as well as heart failure. The crackles, chest pressure, and cool extremities are all consistent with the patient's diagnosis of cardiogenic shock. DIF: Cognitive Level: Apply (application) REF: 1649 | 1633 TOP: Nursing Process: Assessment MSC: NCLEX: Physiological Integrity

A 46-year-old patient tells the nurse about using acetaminophen (Tylenol) several times every day for recurrent bilateral headaches. Which action will the nurse plan to take first? a. Discuss the need to stop taking the acetaminophen. b. Suggest the use of biofeedback for headache control. c. Describe the use of botulism toxin (Botox) for headaches. d. Teach the patient about magnetic resonance imaging (MRI).

ANS: A The headache description suggests that the patient is experiencing medication overuse headache. The initial action will be withdrawal of the medication. The other actions may be needed if the headaches persist

A patient with an open leg wound has a white blood cell (WBC) count of 13, 500/µL and a band count of 11%. What action should the nurse take first? a. Obtain wound cultures. b. Start antibiotic therapy. c. Redress the wound with wet-to-dry dressings. d. Continue to monitor the wound for purulent drainage.

ANS: A The increase in WBC count with the increased bands (shift to the left) indicates that the patient probably has a bacterial infection, and the nurse should obtain wound cultures. Antibiotic therapy and/or dressing changes may be started, but cultures should be done first. The nurse will continue to monitor the wound, but additional actions are needed as well

18. A patient is admitted to the emergency department (ED) for shock of unknown etiology. The first action by the nurse should be to a. administer oxygen. b. obtain a 12-lead electrocardiogram (ECG). c. obtain the blood pressure. d. check the level of consciousness.

ANS: A The initial actions of the nurse are focused on the ABCs—airway, breathing, and circulation—and administration of oxygen should be done first. The other actions should be accomplished as rapidly as possible after oxygen administration. DIF: Cognitive Level: Apply (application) REF: 1641 OBJ: Special Questions: Prioritization TOP: Nursing Process: Implementation MSC: NCLEX: Physiological Integrity

A patient reports feeling numbness and tingling of the left arm before experiencing a tonic-clonic seizure. The nurse determines that this history is consistent with what type of seizure? a. Focal b. Atonic c. Absence d. Myoclonic

ANS: A The initial symptoms of a focal seizure involve clinical manifestations that are localized to a particular part of the body or brain. Symptoms of an absence seizure are staring and a brief loss of consciousness. In an atonic seizure, the patient loses muscle tone and (typically) falls to the ground. Myoclonic seizures are characterized by a sudden jerk of the body or extremities

The nurse observes a patient ambulating in the hospital hall when the patient's arms and legs suddenly jerk and the patient falls to the floor. The nurse will first a. assess the patient for a possible head injury. b. give the scheduled dose of divalproex (Depakote). c. document the timing and description of the seizure. d. notify the patient's health care provider about the seizure.

ANS: A The patient who has had a myoclonic seizure and fall is at risk for head injury and should first be evaluated and treated for this possible complication. Documentation of the seizure, notification of the seizure, and administration of antiseizure medications are also appropriate actions, but the initial action should be assessment for injury

2. A nurse is caring for a patient with shock of unknown etiology whose hemodynamic monitoring indicates BP 92/54, pulse 64, and an elevated pulmonary artery wedge pressure. Which collaborative intervention ordered by the health care provider should the nurse question? a. Infuse normal saline at 250 mL/hr. b. Keep head of bed elevated to 30 degrees. c. Hold nitroprusside (Nipride) if systolic BP <90 mm Hg. d. Titrate dobutamine (Dobutrex) to keep systolic BP >90 mm Hg.

ANS: A The patient's elevated pulmonary artery wedge pressure indicates volume excess. A saline infusion at 250 mL/hr will exacerbate the volume excess. The other actions are appropriate for the patient. DIF: Cognitive Level: Apply (application) REF: 1633 TOP: Nursing Process: Planning MSC: NCLEX: Physiological Integrity

A 19-year-old patient has genital warts around her external genitalia and perianal area. She tells the nurse that she has not sought treatment until now because "the warts are so disgusting." Which nursing diagnosis is most appropriate? a. Disturbed body image related to feelings about the genital warts b. Ineffective coping related to denial of increased risk for infection c. Risk for infection related to lack of knowledge about transmission d. Anxiety related to impact of condition on interpersonal relationships

ANS: A The patient's statement that her lesions are disgusting suggests that disturbed body image is the major concern. There is no evidence to indicate ineffective coping or lack of knowledge about mode of transmission. The patient may be experiencing anxiety, but there is nothing in the data indicating that the genital warts are impacting interpersonal relationships.

Which finding is most important for the nurse to communicate to the health care provider when caring for a patient who is receiving negative pressure wound therapy? a. Low serum albumin level b. Serosanguineous drainage c. Deep red and moist wound bed d. Cobblestone appearance of wound

ANS: A With negative pressure therapy, serum protein levels may decrease, which will adversely affect wound healing. The other findings are expected with wound healing

When assessing the patient experiencing the onset of symptoms of type 1 diabetes, which question is most appropriate for the nurse to ask? a. "Have you lost any weight lately?" b. "How long have you felt anorexic?" c. "Is your urine unusually dark colored?" d. "Do you crave fluids containing sugar?"

ANS: A "Have you lost any weight lately?" Weight loss occurs because the body is no longer able to absorb glucose and starts to break down protein and fat for energy. The patient is thirsty but does not necessarily crave sugar-containing fluids. Increased appetite is a classic symptom of type 1 diabetes. With the classic symptom of polyuria, urine will be very dilute.

The nurse has been teaching the patient to administer a dose of 10 units of regular insulin and 28 units of NPH insulin. The statement by the patient that indicates a need for additional instruction is, a. "I need to rotate injection sites among my arms, legs, and abdomen each day." b. "I will buy the 0.5 mL syringes because the line markings will be easier to see." c. "I should draw up the regular insulin first after injecting air into the NPH bottle." d. "I do not need to aspirate the plunger to check for blood before injecting insulin."

ANS: A "I need to rotate injection sites among my arms, legs, and abdomen each day." Rotating sites is no longer recommended because there is more consistent insulin absorption when the same site is used consistently. The other patient statements are accurate and indicate that no additional instruction is needed.

A patient receives aspart (NovoLog) insulin at 8:00 AM. Which time will it be most important for the nurse to monitor for symptoms of hypoglycemia? a. 9:00 AM b. 11:30 AM c. 4:00 PM d. 8:00 PM

ANS: A 9:00AM The rapid-acting insulins peak in 1 to 3 hours. The patient is not at a high risk for hypoglycemia at the other listed times, although hypoglycemia may occur.

The health care provider suspects the Somogyi effect in a patient whose 7:00 AM blood glucose is 220 mg/dL. Which action will the nurse plan to take? a. Check the patient's blood glucose at 3:00 AM. b. Administer a larger dose of long-acting insulin. c. Educate about the need to increase the rapid-acting insulin dose. d. Remind the patient about the need to avoid snacking at bedtime.

ANS: A Check the patient's blood glucose at 3:00 AM If the Somogyi effect is causing the patient's increased morning glucose level, the patient will experience hypoglycemia between 2 and 4 AM. The dose of insulin will be reduced, rather than increased. A bedtime snack is used to prevent hypoglycemic episodes during the night.

A patient with type 2 diabetes has sensory neuropathy of the feet and legs and peripheral arterial disease. Which information will the nurse include in patient teaching? a. Choose flat-soled leather shoes. b. Set heating pads on a low temperature. c. Buy callus remover for corns or calluses. d. Soak the feet in warm water for an hour every day.

ANS: A Choose flat-soled leather shoes. The patient is taught to avoid high heels and that leather shoes are preferred. The feet should be washed, but not soaked, in warm water daily. Heating pad use should be avoided. Commercial callus and corn removers should be avoided. The patient should see a specialist to treat these problems.

Which information from the patient's health history is most important for the nurse to communicate to the health care provider when a patient has an order for an oral glucose tolerance test? a. The patient uses oral contraceptives. b. The patient runs several days a week. c. The patient has a family history of diabetes. d. The patient had a viral illness 2 months ago.

ANS: A The patient uses oral contraceptives. Oral contraceptive use may falsely elevate oral glucose tolerance test (OGTT) values. A viral illness 2 months previously may be associated with the onset of type 1 diabetes but will not falsely affect the OGTT. Exercise and a family history of diabetes both can affect blood glucose but will not lead to misleading information from the OGTT.

A patient who has just been diagnosed with type 2 diabetes has a nursing diagnosis of imbalanced nutrition: more than body requirements. Which patient goal is most important for this patient? a. The patient will have a glycosylated hemoglobin level of less than 7%. b. The patient will have a diet and exercise plan that results in weight loss. c. The patient will choose a diet that distributes calories throughout the day. d. The patient will state the reasons for eliminating simple sugars in the diet.

ANS: A The patient will have a glycosylated hemoglobin level of less than 7%. The complications of diabetes are related to elevated blood glucose, and the most important patient outcome is the reduction of glucose to near-normal levels. The other outcomes also are appropriate but are not as high in priority.

2. Which preventive actions by the nurse will help limit the development of systemic inflammatory response syndrome (SIRS) in patients admitted to the hospital (select all that apply)? a. Use aseptic technique when caring for invasive lines or devices. b. Ambulate postoperative patients as soon as possible after surgery. c. Remove indwelling urinary catheters as soon as possible after surgery. d. Advocate for parenteral nutrition for patients who cannot take oral feedings. e. Administer prescribed antibiotics within 1 hour for patients with possible sepsis.

ANS: A, B, C, E Because sepsis is the most frequent etiology for SIRS, measures to avoid infection such as removing indwelling urinary catheters as soon as possible, use of aseptic technique, and early ambulation should be included in the plan of care. Adequate nutrition is important in preventing SIRS. Enteral, rather than parenteral, nutrition is preferred when patients are unable to take oral feedings because enteral nutrition helps maintain the integrity of the intestine, thus decreasing infection risk. Antibiotics should be administered within 1 hour after being prescribed to decrease the risk of sepsis progressing to SIRS. DIF: Cognitive Level: Analyze (analysis) REF: 1649 TOP: Nursing Process: Planning MSC: NCLEX: Physiological Integrity

A patient with Parkinson's disease is admitted to the hospital for treatment of pneumonia. Which nursing interventions will be included in the plan of care (select all that apply)? a. Use an elevated toilet seat. b. Cut patient's food into small pieces. c. Provide high-protein foods at each meal. d. Place an armchair at the patient's bedside. e. Observe for sudden exacerbation of symptoms.

ANS: A, B, D Because the patient with Parkinson's has difficulty chewing, food should be cut into small pieces. An armchair should be used when the patient is seated so that the patient can use the arms to assist with getting up from the chair. An elevated toilet seat will facilitate getting on and off the toilet. High-protein foods will decrease the effectiveness of L-dopa. Parkinson's is a steadily progressive disease without acute exacerbations.

A patient with Parkinson's disease is admitted to the hospital for treatment of pneumonia. Which nursing interventions will be included in the plan of care (select all that apply)? a. Use an elevated toilet seat. b. Cut patient's food into small pieces. c. Provide high-protein foods at each meal. d. Place an armchair at the patient's bedside. e. Observe for sudden exacerbation of symptoms.

ANS: A, B, D Because the patient with Parkinson's has difficulty chewing, food should be cut into small pieces. An armchair should be used when the patient is seated so that the patient can use the arms to assist with getting up from the chair. An elevated toilet seat will facilitate getting on and off the toilet. High-protein foods will decrease the effectiveness of L-dopa. Parkinson's is a steadily progressive disease without acute exacerbations.

The nurse in the outpatient clinic notes that the following patients have not received the human papillomavirus (HPV) vaccine. Which patients should the nurse plan to teach about benefits of the vaccine (select all that apply)? a. 24-year-old man who has a history of genital warts b. 18-year-old man who has had one male sexual partner c. 28-year-old woman who has never been sexually active d. 20-year-old woman who has a newly diagnosed Chlamydia infection e. 30-year-old woman whose sexual partner has a history of genital warts

ANS: A, B, D The HPV vaccines are recommended for male and female patients between ages 9 through 26. Ideally, the vaccines are administered before patients are sexually active, but they offer benefit even to those who already have HPV infection

1. A patient with suspected neurogenic shock after a diving accident has arrived in the emergency department. A cervical collar is in place. Which actions should the nurse take (select all that apply)? a. Prepare to administer atropine IV. b. Obtain baseline body temperature. c. Infuse large volumes of lactated Ringer's solution. d. Provide high-flow oxygen (100%) by non-rebreather mask. e. Prepare for emergent intubation and mechanical ventilation.

ANS: A, B, D, E All of the actions are appropriate except to give large volumes of lactated Ringer's solution. The patient with neurogenic shock usually has a normal blood volume, and it is important not to volume overload the patient. In addition, lactated Ringer's solution is used cautiously in all shock situations because the failing liver cannot convert lactate to bicarbonate. DIF: Cognitive Level: Apply (application) REF: 1646 TOP: Nursing Process: Implementation MSC: NCLEX: Physiological Integrity

A 27-year-old patient who has been treated for status epilepticus in the emergency department will be transferred to the medical nursing unit. Which equipment should the nurse have available in the patient's assigned room (select all that apply)? a. Side-rail pads b. Tongue blade c. Oxygen mask d. Suction tubing e. Urinary catheter f. Nasogastric tube

ANS: A, C, D The patient is at risk for further seizures, and oxygen and suctioning may be needed after any seizures to clear the airway and maximize oxygenation. The bed's side rails should be padded to minimize the risk for patient injury during a seizure. Use of tongue blades during a seizure is contraindicated. Insertion of a nasogastric (NG) tube is not indicated because the airway problem is not caused by vomiting or abdominal distention. A urinary catheter is not required unless there is urinary retention

A 39-year-old patient with a history of IV drug use is seen at a community clinic. The patient reports difficulty walking, stating "I don't know where my feet are." Diagnostic screening reveals positive Venereal Disease Research Laboratory (VDRL) and fluorescent treponemal antibody absorption (FTA-Abs) tests. Based on the patient history, what will the nurse assess (select all that apply)? a. Heart sounds b. Genitalia for lesions c. Joints for swelling and inflammation d. Mental state for judgment and orientation e. Skin and mucous membranes for gummas

ANS: A, D, E The patient's clinical manifestations and laboratory tests are consistent with tertiary syphilis. Valvular insufficiency, gummas, and changes in mentation are other clinical manifestations of this stage

24. After change-of-shift report in the progressive care unit, who should the nurse care for first? a. Patient who had an inferior myocardial infarction 2 days ago and has crackles in the lung bases b. Patient with suspected urosepsis who has new orders for urine and blood cultures and antibiotics c. Patient who had a T5 spinal cord injury 1 week ago and currently has a heart rate of 54 beats/minute d. Patient admitted with anaphylaxis 3 hours ago who now has clear lung sounds and a blood pressure of 108/58 mm Hg

ANS: B Antibiotics should be administered within the first hour for patients who have sepsis or suspected sepsis in order to prevent progression to systemic inflammatory response syndrome (SIRS) and septic shock. The data on the other patients indicate that they are more stable. Crackles heard only at the lung bases do not require immediate intervention in a patient who has had a myocardial infarction. Mild bradycardia does not usually require atropine in patients who have a spinal cord injury. The findings for the patient admitted with anaphylaxis indicate resolution of bronchospasm and hypotension. DIF: Cognitive Level: Analyze (analysis) REF: 1644 | 1646 OBJ: Special Questions: Prioritization; Multiple Patients TOP: Nursing Process: Assessment MSC: NCLEX: Safe and Effective Care Environment

9. Which finding is the best indicator that the fluid resuscitation for a patient with hypovolemic shock has been effective? a. Hemoglobin is within normal limits. b. Urine output is 60 mL over the last hour. c. Central venous pressure (CVP) is normal. d. Mean arterial pressure (MAP) is 72 mm Hg.

ANS: B Assessment of end organ perfusion, such as an adequate urine output, is the best indicator that fluid resuscitation has been successful. The hemoglobin level, CVP, and MAP are useful in determining the effects of fluid administration, but they are not as useful as data indicating good organ perfusion. DIF: Cognitive Level: Apply (application) REF: 1642 TOP: Nursing Process: Evaluation MSC: NCLEX: Physiological Integrity

17. The nurse is caring for a patient who has septic shock. Which assessment finding is most important for the nurse to report to the health care provider? a. Blood pressure (BP) 92/56 mm Hg b. Skin cool and clammy c. Oxygen saturation 92% d. Heart rate 118 beats/minute

ANS: B Because patients in the early stage of septic shock have warm and dry skin, the patient's cool and clammy skin indicates that shock is progressing. The other information will also be reported, but does not indicate deterioration of the patient's status. DIF: Cognitive Level: Apply (application) REF: 1638 OBJ: Special Questions: Prioritization TOP: Nursing Process: Assessment MSC: NCLEX: Physiological Integrity

A 22-year-old patient with gonorrhea is treated with a single IM dose of ceftriaxone (Rocephin) and is given a prescription for doxycycline (Vibramycin) 100 mg bid for 7 days. The nurse explains to the patient that this combination of antibiotics is prescribed to a. prevent reinfection during treatment. b. treat any coexisting chlamydial infection. c. eradicate resistant strains of N. gonorrhoeae. d. prevent the development of resistant organisms.

ANS: B Because there is a high incidence of co-infection with gonorrhea and chlamydia, patients are usually treated for both. The other explanations about the purpose of the antibiotic combination are not accurate

Which medication taken by a patient with restless legs syndrome should the nurse discuss with the patient? a. Multivitamin (Stresstabs) b. Acetaminophen (Tylenol) c. Ibuprofen (Motrin, Advil) d. Diphenhydramine (Benadryl)

ANS: D Antihistamines can aggravate restless legs syndrome. The other medications will not contribute to restless legs syndrome

Which nursing action is a priority for a patient who has suffered a burn injury while working on an electrical power line? a. Obtain the blood pressure. b. Stabilize the cervical spine. c. Assess for the contact points. d. Check alertness and orientation.

ANS: B Cervical spine injuries are commonly associated with electrical burns. Therefore stabilization of the cervical spine takes precedence after airway management. The other actions are also included in the emergent care after electrical burns, but the most important action is to avoid spinal cord injury.

22. The following interventions are ordered by the health care provider for a patient who has respiratory distress and syncope after eating strawberries. Which will the nurse complete first? a. Start a normal saline infusion. b. Give epinephrine (Adrenalin). c. Start continuous ECG monitoring. d. Give diphenhydramine (Benadryl).

ANS: B Epinephrine rapidly causes peripheral vasoconstriction, dilates the bronchi, and blocks the effects of histamine and reverses the vasodilation, bronchoconstriction, and histamine release that cause the symptoms of anaphylaxis. The other interventions are also appropriate but would not be the first ones completed. DIF: Cognitive Level: Apply (application) REF: 1645 OBJ: Special Questions: Prioritization TOP: Nursing Process: Implementation MSC: NCLEX: Physiological Integrity

A patient admitted with chest pain is also found to have positive Venereal Disease Research Laboratory (VDRL) and fluorescent treponemal antibody absorption (FAT-Abs) tests, rashes on the palms and the soles of the feet, and moist papules in the anal and vulvar area. Which action will the nurse include in the plan of care? a. Assess for arterial aneurysms. b. Wear gloves for patient contact. c. Place the patient in a private room. d. Apply antibiotic ointment to the perineum.

ANS: B Exudate from any lesions with syphilis is highly contagious. Systemic antibiotics, rather than local treatment of lesions, are used to treat syphilis. The patient does not require a private room because the disease is spread through contact with the lesions. This patient has clinical manifestations of secondary syphilis and does not need to be monitored for manifestations of tertiary syphilis

The home health registered nurse (RN) is planning care for a patient with a seizure disorder related to a recent head injury. Which nursing action can be delegated to a licensed practical/vocational nurse (LPN/LVN)? a. Make referrals to appropriate community agencies. b. Place medications in the home medication organizer. c. Teach the patient and family how to manage seizures. d. Assess for use of medications that may precipitate seizures.

ANS: B LPN/LVN education includes administration of medications. The other activities require RN education and scope of practice

A 48-year-old male patient who has been diagnosed with gonococcal urethritis tells the nurse he had recent sexual contact with a woman but says she did not appear to have any disease. In responding to the patient, the nurse explains that a. women do not develop gonorrhea infections but can serve as carriers to spread the disease to males. b. women may not be aware they have gonorrhea because they often do not have symptoms of infection. c. women develop subclinical cases of gonorrhea that do not cause tissue damage or clinical manifestations. d. when gonorrhea infections occur in women, the disease affects only the ovaries and not the genital organs.

ANS: B Many women with gonorrhea are asymptomatic or have minor symptoms that are overlooked. The disease may affect both the genitals and the other reproductive organs and cause complications such as pelvic inflammatory disease (PID). Women who can transmit the disease have active infections

Which intervention will the nurse include in the plan of care for a patient with primary restless legs syndrome (RLS) who is having difficulty sleeping? a. Teach about the use of antihistamines to improve sleep. b. Suggest that the patient exercise regularly during the day. c. Make a referral to a massage therapist for deep massage of the legs. d. Assure the patient that the problem is transient and likely to resolve.

ANS: B Nondrug interventions such as getting regular exercise are initially suggested to improve sleep quality in patients with RLS. Antihistamines may aggravate RLS. Massage does not alleviate RLS symptoms and RLS is likely to progress in most patients

When teaching a diabetic patient who has just been started on intensive insulin therapy about mealtime coverage, which type of insulin will the nurse need to discuss? a. glargine (Lantus) b. lispro (Humalog) c. detemir (Levemir) d. NPH (Humulin N)

ANS: B Rapid- or short-acting insulin is used for mealtime coverage for patients receiving intensive insulin therapy. NPH, glargine, or detemir will be used as the basal insulin.

A 73-year-old patient with Parkinson's disease has a nursing diagnosis of impaired physical mobility related to bradykinesia. Which action will the nurse include in the plan of care? a. Instruct the patient in activities that can be done while lying or sitting. b. Suggest that the patient rock from side to side to initiate leg movement. c. Have the patient take small steps in a straight line directly in front of the feet. d. Teach the patient to keep the feet in contact with the floor and slide them forward.

ANS: B Rocking the body from side to side stimulates balance and improves mobility. The patient will be encouraged to continue exercising because this will maintain functional abilities. Maintaining a wide base of support will help with balance. The patient should lift the feet and avoid a shuffling gait

A 73-year-old patient with Parkinson's disease has a nursing diagnosis of impaired physical mobility related to bradykinesia. Which action will the nurse include in the plan of care? a. Instruct the patient in activities that can be done while lying or sitting. b. Suggest that the patient rock from side to side to initiate leg movement. c. Have the patient take small steps in a straight line directly in front of the feet. d. Teach the patient to keep the feet in contact with the floor and slide them forward.

ANS: B Rocking the body from side to side stimulates balance and improves mobility. The patient will be encouraged to continue exercising because this will maintain functional abilities. Maintaining a wide base of support will help with balance. The patient should lift the feet and avoid a shuffling gait

10. Which intervention will the nurse include in the plan of care for a patient who has cardiogenic shock? a. Check temperature every 2 hours. b. Monitor breath sounds frequently. c. Maintain patient in supine position. d. Assess skin for flushing and itching.

ANS: B Since pulmonary congestion and dyspnea are characteristics of cardiogenic shock, the nurse should assess the breath sounds frequently. The head of the bed is usually elevated to decrease dyspnea in patients with cardiogenic shock. Elevated temperature and flushing or itching of the skin are not typical of cardiogenic shock. DIF: Cognitive Level: Apply (application) REF: 1633 TOP: Nursing Process: Implementation MSC: NCLEX: Physiological Integrity

Which action will the nurse plan to take for a 40-year-old patient with multiple sclerosis (MS) who has urinary retention caused by a flaccid bladder? a. Decrease the patient's evening fluid intake. b. Teach the patient how to use the Credé method. c. Suggest the use of adult incontinence briefs for nighttime only. d. Assist the patient to the commode every 2 hours during the day.

ANS: B The Credé method can be used to improve bladder emptying. Decreasing fluid intake will not improve bladder emptying and may increase risk for urinary tract infection (UTI) and dehydration. The use of incontinence briefs and frequent toileting will not improve bladder emptying

7. A patient with cardiogenic shock has the following vital signs: BP 102/50, pulse 128, respirations 28. The pulmonary artery wedge pressure (PAWP) is increased and cardiac output is low. The nurse will anticipate an order for which medication? a. 5% human albumin b. Furosemide (Lasix) IV c. Epinephrine (Adrenalin) drip d. Hydrocortisone (Solu-Cortef)

ANS: B The PAWP indicates that the patient's preload is elevated, and furosemide is indicated to reduce the preload and improve cardiac output. Epinephrine would further increase heart rate and myocardial oxygen demand. 5% human albumin would also increase the PAWP. Hydrocortisone might be considered for septic or anaphylactic shock. DIF: Cognitive Level: Apply (application) REF: 1645 TOP: Nursing Process: Planning MSC: NCLEX: Physiological Integrity

When a 74-year-old patient is seen in the health clinic with new development of a stooped posture, shuffling gait, and pill rolling-type tremor, the nurse will anticipate teaching the patient about a. oral corticosteroids. b. antiparkinsonian drugs. c. magnetic resonance imaging (MRI). d. electroencephalogram (EEG) testing.

ANS: B The diagnosis of Parkinson's is made when two of the three characteristic manifestations of tremor, rigidity, and bradykinesia are present. The confirmation of the diagnosis is made on the basis of improvement when antiparkinsonian drugs are administered. This patient has symptoms of tremor and bradykinesia. The next anticipated step will be treatment with medications. MRI and EEG are not useful in diagnosing Parkinson's disease, and corticosteroid therapy is not used to treat it

When a 74-year-old patient is seen in the health clinic with new development of a stooped posture, shuffling gait, and pill rolling-type tremor, the nurse will anticipate teaching the patient about a. oral corticosteroids. b. antiparkinsonian drugs. c. magnetic resonance imaging (MRI). d. electroencephalogram (EEG) testing.

ANS: B The diagnosis of Parkinson's is made when two of the three characteristic manifestations of tremor, rigidity, and bradykinesia are present. The confirmation of the diagnosis is made on the basis of improvement when antiparkinsonian drugs are administered. This patient has symptoms of tremor and bradykinesia. The next anticipated step will be treatment with medications. MRI and EEG are not useful in diagnosing Parkinson's disease, and corticosteroid therapy is not used to treat it

The nurse assesses a patient's surgical wound on the first postoperative day and notes redness and warmth around the incision. Which action by the nurse is most appropriate? a. Obtain wound cultures. b. Document the assessment. c. Notify the health care provider. d. Assess the wound every 2 hours.

ANS: B The incisional redness and warmth are indicators of the normal initial (inflammatory) stage of wound healing by primary intention. The nurse should document the wound appearance and continue to monitor the wound. Notification of the health care provider, assessment every 2 hours, and obtaining wound cultures are not indicated because the healing is progressing normally.

A high school teacher who has just been diagnosed with epilepsy after having a generalized tonic-clonic seizure tells the nurse, "I cannot teach anymore, it will be too upsetting if I have a seizure at work." Which response by the nurse is best? a. "You might benefit from some psychologic counseling." b. "Epilepsy usually can be well controlled with medications." c. "You will want to contact the Epilepsy Foundation for assistance." d. "The Department of Vocational Rehabilitation can help with work retraining."

ANS: B The nurse should inform the patient that most patients with seizure disorders are controlled with medication. The other information may be necessary if the seizures persist after treatment with antiseizure medications is implemented

A patient is admitted to the burn unit with burns to the head, face, and hands. Initially, wheezes are heard, but an hour later, the lung sounds are decreased and no wheezes are audible. What is the best action for the nurse to take? a. Encourage the patient to cough and auscultate the lungs again. b. Notify the health care provider and prepare for endotracheal intubation. c. Document the results and continue to monitor the patient's respiratory rate. d. Reposition the patient in high-Fowler's position and reassess breath sounds.

ANS: B The patient's history and clinical manifestations suggest airway edema and the health care provider should be notified immediately, so that intubation can be done rapidly. Placing the patient in a more upright position or having the patient cough will not address the problem of airway edema. Continuing to monitor is inappropriate because immediate action should occur

Following a thymectomy, a 62-year-old male patient with myasthenia gravis receives the usual dose of pyridostigmine (Mestinon). An hour later, the patient complains of nausea and severe abdominal cramps. Which action should the nurse take first? a. Auscultate the patient's bowel sounds. b. Notify the patient's health care provider. c. Administer the prescribed PRN antiemetic drug. d. Give the scheduled dose of prednisone (Deltasone).

ANS: B The patient's history and symptoms indicate a possible cholinergic crisis. The health care provider should be notified immediately, and it is likely that atropine will be prescribed. The other actions will be appropriate if the patient is not experiencing a cholinergic crisis

A hospitalized patient complains of a bilateral headache, 4/10 on the pain scale, that radiates from the base of the skull. Which prescribed PRN medications should the nurse administer initially? a. Lorazepam (Ativan) b. Acetaminophen (Tylenol) c. Morphine sulfate (Roxanol) d. Butalbital and aspirin (Fiorinal)

ANS: B The patient's symptoms are consistent with a tension headache, and initial therapy usually involves a nonopioid analgesic such as acetaminophen, which is sometimes combined with a sedative or muscle relaxant. Lorazepam may be used in conjunction with acetaminophen but would not be appropriate as the initial monotherapy. Morphine sulfate and butalbital and aspirin would be more appropriate for a headache that did not respond to a nonopioid analgesic

The health care provider is considering the use of sumatriptan (Imitrex) for a 54-year-old male patient with migraine headaches. Which information obtained by the nurse is most important to report to the health care provider? a. The patient drinks 1 to 2 cups of coffee daily. b. The patient had a recent acute myocardial infarction. c. The patient has had migraine headaches for 30 years. d. The patient has taken topiramate (Topamax) for 2 months.

ANS: B The triptans cause coronary artery vasoconstriction and should be avoided in patients with coronary artery disease. The other information will be reported to the health care provider, but none of it indicates that sumatriptan would be an inappropriate treatment

A young adult patient who is in the rehabilitation phase 6 months after a severe face and neck burn tells the nurse, "I'm sorry that I'm still alive. My life will never be normal again." Which response by the nurse is best? a. "Most people recover after a burn and feel satisfied with their lives." b. "It's true that your life may be different. What concerns you the most?" c. "It is really too early to know how much your life will be changed by the burn." d. "Why do you feel that way? You will be able to adapt as your recovery progresses."

ANS: B This response acknowledges the patient's feelings and asks for more assessment data that will help in developing an appropriate plan of care to assist the patient with the emotional response to the burn injury. The other statements are accurate, but do not acknowledge the anxiety and depression that the patient is expressing.

A 32-year-old woman who is diagnosed with Chlamydia tells the nurse that she is very angry because her husband is her only sexual partner. Which response should the nurse make first? a. "You may need professional counseling to help resolve your anger." b. "It is understandable that you are angry with your husband right now." c. "Your feelings are justified and you should share them with your husband." d. "It is important that both you and your husband be treated for the infection."

ANS: B This response expresses the nurse's acceptance of the patient's feelings and encourages further discussion and problem solving. The patient may need professional counseling, but more assessment of the patient is needed before making this judgment. The nurse should also assess further before suggesting that the patient share her feelings with the husband because problems such as abuse might be present in the relationship. Although it is important that both partners be treated, the patient's current anger suggests that this is not the appropriate time to bring this up

The nurse will assess a 67-year-old patient who is experiencing a cluster headache for a. nuchal rigidity. b. unilateral ptosis. c. projectile vomiting. d. throbbing, bilateral facial pain.

ANS: B Unilateral eye edema, tearing, and ptosis are characteristic of cluster headaches. Nuchal rigidity suggests meningeal irritation, such as occurs with meningitis. Although nausea and vomiting may occur with migraine headaches, projectile vomiting is more consistent with increased intracranial pressure (ICP). Unilateral sharp, stabbing pain, rather than throbbing pain, is characteristic of cluster headaches.

When obtaining a health history and physical assessment for a 36-year-old female patient with possible multiple sclerosis (MS), the nurse should a. assess for the presence of chest pain. b. inquire about urinary tract problems. c. inspect the skin for rashes or discoloration. d. ask the patient about any increase in libido.

ANS: B Urinary tract problems with incontinence or retention are common symptoms of MS. Chest pain and skin rashes are not symptoms of MS. A decrease in libido is common with MS.

5. After receiving 2 L of normal saline, the central venous pressure for a patient who has septic shock is 10 mm Hg, but the blood pressure is still 82/40 mm Hg. The nurse will anticipate an order for a. nitroglycerine (Tridil). b. norepinephrine (Levophed). c. sodium nitroprusside (Nipride). d. methylprednisolone (Solu-Medrol).

ANS: B When fluid resuscitation is unsuccessful, vasopressor drugs are administered to increase the systemic vascular resistance (SVR) and blood pressure, and improve tissue perfusion. Nitroglycerin would decrease the preload and further drop cardiac output and BP. Methylprednisolone (Solu-Medrol) is considered if blood pressure does not respond first to fluids and vasopressors. Nitroprusside is an arterial vasodilator and would further decrease SVR. DIF: Cognitive Level: Apply (application) REF: 1643 TOP: Nursing Process: Planning MSC: NCLEX: Physiological Integrity

When assessing a patient who spilled hot oil on the right leg and foot, the nurse notes that the skin is dry, pale, hard skin. The patient states that the burn is not painful. What term would the nurse use to document the burn depth? a. First-degree skin destruction b. Full-thickness skin destruction c. Deep partial-thickness skin destruction d. Superficial partial-thickness skin destruction

ANS: B With full-thickness skin destruction, the appearance is pale and dry or leathery and the area is painless because of the associated nerve destruction. Erythema, swelling, and blisters point to a deep partial-thickness burn. With superficial partial-thickness burns, the area is red, but no blisters are present. First-degree burns exhibit erythema, blanching, and pain

Which question by the nurse will help identify autonomic neuropathy in a diabetic patient? a. "Have you observed any recent skin changes?" b. "Do you notice any bloating feeling after eating?" c. "Do you need to increase your insulin dosage when you are stressed?" d. "Have you noticed any painful new ulcerations or sores on your feet?"

ANS: B "Do you notice any bloating feeling after eating?" Autonomic neuropathy can cause delayed gastric emptying, which results in a bloated feeling for the patient. The other questions also are appropriate to ask, but would not help in identifying autonomic neuropathy.

Which action should the nurse take first when teaching a patient who is newly diagnosed with type 2 diabetes about home management of the disease? a. Ask the patient's family to participate in the diabetes education program. b. Assess the patient's perception of what it means to have diabetes mellitus. c. Demonstrate how to check glucose using capillary blood glucose monitoring. d. Discuss the need for the patient to actively participate in diabetes management.

ANS: B Assess the patient's perception of what it means to have diabetes mellitus. Before planning education, the nurse should assess the patient's interest in and ability to self-manage the diabetes. After assessing the patient, the other nursing actions may be appropriate, but planning needs to be individualized to each patient.

Which action is most important for the nurse to take in order to assist a diabetic patient to engage in moderate daily exercise? a. Remind the patient that exercise will improve self-esteem. b. Determine what type of exercise activities the patient enjoys. c. Give the patient a list of activities that are moderate in intensity. d. Teach the patient about the effects of exercise on glucose level.

ANS: B Determine what type of exercise activities the patient enjoys. Since consistency with exercise is important, assessment for the types of exercise that the patient finds enjoyable is the most important action by the nurse in ensuring adherence to an exercise program. The other actions also will be implemented, but are not the most important in improving compliance.

A pregnant patient who has no personal history of diabetes, but does have a parent who is diabetic is scheduled for the first prenatal visit. Which action will the nurse plan to take on this initial visit? a. Teach about appropriate use of regular insulin. b. Discuss the need for a fasting blood glucose level. c. Schedule an oral glucose tolerance test for the twenty fourth week of pregnancy. d. Provide education about increased risk for fetal problems with gestational diabetes.

ANS: B Discuss the need for a fasting blood glucose level. Patients at high risk for gestational diabetes should be screened for diabetes on the initial prenatal visit. An oral glucose tolerance test also may be used to check for diabetes, but it would be done before the twenty fourth week. The other actions also may be needed (depending on whether the patient develops gestational diabetes), but they are not the first actions that the nurse should take.

Intramuscular glucagon is administered to an unresponsive patient for treatment of hypoglycemia. Which action should the nurse take after the patient regains consciousness? a. Assess the patient for symptoms of hyperglycemia. b. Give the patient a snack of crackers and peanut butter. c. Have the patient drink a glass of orange juice or nonfat milk. d. Administer a continuous infusion of 5% dextrose for 24 hours.

ANS: B Give the patient a snack of crackers and peanut butter. Rebound hypoglycemia can occur after glucagon administration, but having a meal containing complex carbohydrates plus protein and fat will help prevent hypoglycemia. Orange juice and nonfat milk will elevate blood sugar rapidly, but the cheese and crackers will stabilize blood sugar. Administration of glucose intravenously might be used in patients who were unable to take in nutrition orally. The patient should be assessed for symptoms of hypoglycemia after glucagon administration.

Which information will the nurse include when teaching a patient who has type 2 diabetes about glyburide (Micronase, DiaBeta, Glynase)? a. Glyburide decreases glucagon secretion from the pancreas. b. Glyburide stimulates insulin production and release from the pancreas. c. Glyburide should be taken even if the morning blood glucose level is low. d. Glyburide should not be used for 48 hours after receiving IV contrast media.

ANS: B Glyburide stimulates insulin production and release from the pancreas. The sulfonylureas stimulate the production and release of insulin from the pancreas. If the glucose level is low, the patient should contact the health care provider before taking the glyburide, because hypoglycemia can occur with this category of medication. Metformin should be held for 48 hours after administration of IV contrast media, but this is not necessary for glyburide. Glucagon secretion is not affected by glyburide.

A diabetic patient is admitted with ketoacidosis and the health care provider writes these orders. Which order should the nurse implement first? a. Administer regular IV insulin 30 U. b. Infuse 1 liter of normal saline per hour. c. Give sodium bicarbonate 50 mEq IV push. d. Start an infusion of regular insulin at 50 U/hr.

ANS: B Infuse 1 liter of normal saline per hour. The most urgent patient problem is the hypovolemia associated with diabetic ketoacidosis (DKA), and the priority is to infuse IV fluids. The other actions can be accomplished after the infusion of normal saline is initiated.

Which of these laboratory values, noted by the nurse when reviewing the chart of a hospitalized diabetic patient, indicates the need for rapid assessment of the patient? a. Hb A1C of 5.8% b. Noon blood glucose of 52 mg/dL c. Hb A1Cof 6.9% d. Fasting blood glucose of 130 mg/dL

ANS: B Noon blood glucose of 52 mg/dL The nurse should assess the patient with a blood glucose level of 52 mg/dL for symptoms of hypoglycemia, and give the patient some carbohydrate-containing beverage such as orange juice. The other values are within an acceptable range for a diabetic patient.

When the nurse is assessing a patient who is recovering from an episode of diabetic ketoacidosis, the patient reports feeling anxious, nervous, and sweaty. Which action should the nurse take first? a. Administer 1 mg glucagon subcutaneously. b. Obtain a glucose reading using a finger stick. c. Have the patient drink 4 ounces of orange juice. d. Give the scheduled dose of lispro (Humalog) insulin.

ANS: B Obtain a glucose reading using a finger stick. The patient's clinical manifestations are consistent with hypoglycemia and the initial action should be to check the patient's glucose with a finger stick or order a stat blood glucose. If the glucose is low, the patient should ingest a rapid-acting carbohydrate, such as orange juice. Glucagon might be given if the patient's symptoms become worse or if the patient is unconscious. Administration of lispro would drop the patient's glucose further.

A patient is admitted with diabetic ketoacidosis (DKA) and has a serum potassium level of 2.9 mEq/L. Which action prescribed by the health care provider should the nurse take first? a. Infuse regular insulin at 20 U/hr. b. Place the patient on a cardiac monitor. c. Administer IV potassium supplements. d. Obtain urine glucose and ketone levels.

ANS: B Place the patient on a cardiac monitor. Hypokalemia can lead to potentially fatal dysrhythmias such as ventricular tachycardia and ventricular fibrillation, which would be detected with ECG monitoring. Since potassium must be infused over at least 1 hour, the nurse should initiate cardiac monitoring before infusion of potassium. Insulin should not be administered without cardiac monitoring, since insulin infusion will further decrease potassium levels. Urine glucose and ketone levels are not urgently needed to manage the patient's care.

Which action will the nurse include in the plan of care for a patient in the rehabilitation phase after a burn injury to the right arm and chest? a. Keep the right arm in a position of comfort. b. Avoid the use of sustained-release narcotics. c. Teach about the purpose of tetanus immunization. d. Apply water-based cream to burned areas frequently.

ANS: D Application of water-based emollients will moisturize new skin and decrease flakiness and itching. To avoid contractures, the joints of the right arm should be positioned in an extended position, which is not the position of comfort. Patients may need to continue the use of opioids during rehabilitation. Tetanus immunization would have been given during the emergent phase of the burn injury

After the nurse has finished teaching a patient about self-administration of the prescribed aspart (NovoLog) insulin, which patient action indicates good understanding of the teaching? a. The patient avoids injecting the insulin into the upper abdominal area. b. The patient cleans the skin with soap and water before insulin administration. c. The patient places the insulin back in the freezer after administering the prescribed insulin dose. d. The patient pushes the plunger down and immediately removes the syringe from the injection site.

ANS: B The patient cleans the skin with soap and water before insulin administration. Cleaning the skin with soap and water or with alcohol is acceptable. Insulin should not be frozen. The patient should leave the syringe in place for about 5 seconds after injection to be sure that all the insulin has been injected. The upper abdominal area is one of the preferred areas for insulin injection.

Which patient action indicates a good understanding of the nurse's teaching about the use of an insulin pump? a. The patient changes the site for the insertion site every week. b. The patient programs the pump to deliver an insulin bolus after eating. c. The patient takes the pump off at bedtime and starts it again each morning. d. The patient states that diet will be less flexible when using the insulin pump.

ANS: B The patient programs the pump to deliver an insulin bolus after eating. In addition to the basal rate of insulin infusion, the patient will adjust the pump to administer a bolus after each meal, with the dosage depending on the oral intake. The insertion site should be changed every 2 or 3 days. There is more flexibility in diet and exercise when an insulin pump is used. The pump will deliver a basal insulin rate 24 hours a day.

The nurse obtains the following information about a patient before administration of metformin (Glucophage). Which finding indicates a need to contact the health care provider before giving the metformin? a. The patient's blood glucose level is 166 mg/dL. b. The patient's blood urea nitrogen (BUN) level is 60 mg/dL. c. The patient is scheduled for a chest x-ray in an hour. d. The patient has gained 2 lb (0.9 kg) since yesterday.

ANS: B The patient's blood urea nitrogen (BUN) level is 60 mg/dL. The BUN indicates impending renal failure and metformin should not be used in patients with renal failure. The other findings are not contraindications to the use of metformin.

A patient with type 1 diabetes has been using self-monitoring of blood glucose (SMBG) as part of diabetes management. During evaluation of the patient's technique of SMBG, the nurse identifies a need for additional teaching when the patient a. washes the puncture site using soap and warm water. b. chooses a puncture site in the center of the finger pad. c. hangs the arm down for a minute before puncturing the site. d. says the result of 130 mg indicates good blood sugar control.

ANS: B chooses a puncture site in the center of the finger pad. The patient is taught to choose a puncture site at the side of the finger pad. The other patient actions indicate that teaching has been effective.

A diagnosis of hyperglycemic hyperosmolar nonketotic coma (HHNC) is made for a patient with type 2 diabetes who is brought to the emergency department in an unresponsive state. The nurse will anticipate the need to a. give 50% dextrose as a bolus. b. insert a large-bore IV catheter. c. initiate oxygen by nasal cannula. d. administer glargine (Lantus) insulin.

ANS: B insert a large-bore IV catheter. HHNC is initially treated with large volumes of IV fluids to correct hypovolemia. Regular insulin is administered, not a long-acting insulin. There is no indication that the patient requires oxygen. Dextrose solutions will increase the patient's blood glucose and would be contraindicated.

A patient with type 1 diabetes who uses glargine (Lantus) and lispro (Humalog) insulin develops a sore throat, cough, and fever. When the patient calls the clinic to report the symptoms and a blood glucose level of 210 mg/dL, the nurse advises the patient to a. use only the lispro insulin until the symptoms of infection are resolved. b. monitor blood glucose every 4 hours and notify the clinic if it continues to rise. c. decrease intake of carbohydrates until glycosylated hemoglobin is less than 7%. d. limit intake of calorie-containing liquids until the glucose is less than 120 mg/dL.

ANS: B monitor blood glucose every 4 hours and notify the clinic if it continues to rise. Infection and other stressors increase blood glucose levels and the patient will need to test blood glucose frequently, treat elevations appropriately with lispro insulin, and call the health care provider if glucose levels continue to be elevated. Discontinuing the glargine will contribute to hyperglycemia and may lead to diabetic ketoacidosis (DKA). Decreasing carbohydrate or caloric intake is not appropriate because the patient will need more calories when ill. Glycosylated hemoglobins are not used to test for short-term alterations in blood glucose.

A patient with type 2 diabetes that is well-controlled with metformin (Glucophage) develops an allergic rash to an antibiotic and the health care provider prescribes prednisone (Deltasone). The nurse will anticipate that the patient may a. need a diet higher in calories while receiving prednisone. b. require administration of insulin while taking prednisone. c. develop acute hypoglycemia while taking the prednisone. d. have rashes caused by metformin-prednisone interactions.

ANS: B require administration of insulin while taking prednisone. Glucose levels increase when patients are taking corticosteroids, and insulin may be required to control blood glucose. Hypoglycemia is not a side effect of prednisone. Rashes are not an adverse effect caused by taking metformin and prednisone simultaneously. The patient may have an increased appetite when taking prednisone, but will not need a diet that is higher in calories.

Which topics will the nurse include when preparing to teach a patient with recurrent genital herpes simplex (select all that apply)? a. Infected areas should be kept moist to speed healing. b. Sitz baths may be used to relieve discomfort caused by the lesions. c. Genital herpes can be cured by consistent use of antiviral medications. d. Recurrent genital herpes episodes usually are shorter than the first episode. e. The virus can infect sexual partners even when you do not have symptoms of infection.

ANS: B, D, E Patients are taught that shedding of the virus and infection of sexual partners can occur even in asymptomatic periods, that recurrent episodes resolve more quickly, and that sitz baths can be used to relieve pain caused by the lesions. Antiviral medications decrease the number of outbreaks, but do not cure herpes simplex infections. Infected areas may be kept dry if this decreases pain and itching

When a 31-year-old male patient returns to the clinic for follow-up after treatment for gonococcal urethritis, a purulent urethral discharge is still present. When trying to determine the reason for the recurrent infection, which question is most appropriate for the nurse to ask the patient? a. "Did you take the prescribed antibiotic for a week?" b. "Did you drink at least 2 quarts of fluids every day?" c. "Were your sexual partners treated with antibiotics?" d. "Do you wash your hands after using the bathroom?"

ANS: C A common reason for recurrence of symptoms is reinfection because infected partners have not been simultaneously treated. Because gonorrhea is treated with one dose of antibiotic, antibiotic therapy for a week is not needed. An adequate fluid intake is important, but a low fluid intake is not a likely cause for failed treatment. Poor hygiene may cause complications such as ocular trachoma but will not cause a failure of treatment

8. The emergency department (ED) nurse receives report that a patient involved in a motor vehicle crash is being transported to the facility with an estimated arrival in 1 minute. In preparation for the patient's arrival, the nurse will obtain a. hypothermia blanket. b. lactated Ringer's solution. c. two 14-gauge IV catheters. d. dopamine (Intropin) infusion.

ANS: C A patient with multiple trauma may require fluid resuscitation to prevent or treat hypovolemic shock, so the nurse will anticipate the need for 2 large bore IV lines to administer normal saline. Lactated Ringer's solution should be used cautiously and will not be ordered until the patient has been assessed for possible liver abnormalities. Vasopressor infusion is not used as the initial therapy for hypovolemic shock. Patients in shock need to be kept warm not cool. DIF: Cognitive Level: Apply (application) REF: 1641 TOP: Nursing Process: Planning MSC: NCLEX: Physiological Integrity

A woman is diagnosed with primary syphilis during her eighth week of pregnancy. The nurse will plan to teach the patient about the a. likelihood of a stillbirth. b. plans for cesarean section c. intramuscular injection of penicillin. d. antibiotic eye drops for the newborn.

ANS: C A single injection of penicillin is recommended to treat primary syphilis. This will treat the mother and prevent transmission of the disease to the fetus. Instillation of erythromycin into the eyes of the newborn is used to prevent gonorrheal eye infections. C-section is used to prevent the transmission of herpes to the newborn. Although stillbirth can occur if the fetus is infected with syphilis, treatment before the tenth week of gestation will eliminate in utero transmission to the fetus.

A patient from a long-term care facility is admitted to the hospital with a sacral pressure ulcer. The base of the wound is yellow and involves subcutaneous tissue. How should the nurse classify this pressure ulcer? a. Stage I b. Stage II c. Stage III d. Stage IV

ANS: C A stage III pressure ulcer has full-thickness skin damage and extends into the subcutaneous tissue. A stage I pressure ulcer has intact skin with some observable damage such as redness or a boggy feel. Stage II pressure ulcers have partial-thickness skin loss. Stage IV pressure ulcers have full-thickness damage with tissue necrosis, extensive damage, or damage to bone, muscle, or supporting tissues

Which assessment is most important for the nurse to make regarding a patient with myasthenia gravis? a. Pupil size b. Grip strength c. Respiratory effort d. Level of consciousness

ANS: C Because respiratory insufficiency may be life threatening, it will be most important to monitor respiratory function. The other data also will be assessed but are not as critical

While the nurse is transporting a patient on a stretcher to the radiology department, the patient begins having a tonic-clonic seizure. Which action should the nurse take? a. Insert an oral airway during the seizure to maintain a patent airway. b. Restrain the patient's arms and legs to prevent injury during the seizure. c. Time and observe and record the details of the seizure and postictal state. d. Avoid touching the patient to prevent further nervous system stimulation.

ANS: C Because the diagnosis and treatment of seizures frequently are based on the description of the seizure, recording the length and details of the seizure is important. Insertion of an oral airway and restraining the patient during the seizure are contraindicated. The nurse may need to move the patient to decrease the risk of injury during the seizure

23. Which finding about a patient who is receiving vasopressin (Pitressin) to treat septic shock is most important for the nurse to communicate to the health care provider? a. The patient's urine output is 18 mL/hr. b. The patient's heart rate is 110 beats/minute. c. The patient is complaining of chest pain. d. The patient's peripheral pulses are weak.

ANS: C Because vasopressin is a potent vasoconstrictor, it may decrease coronary artery perfusion. The other information is consistent with the patient's diagnosis and should be reported to the health care provider but does not indicate a need for a change in therapy. DIF: Cognitive Level: Apply (application) REF: 1643 OBJ: Special Questions: Prioritization TOP: Nursing Process: Assessment MSC: NCLEX: Physiological Integrity

After receiving a change-of-shift report, which patient should the nurse assess first? a. The patient who has multiple black wounds on the feet and ankles b. The newly admitted patient with a stage IV pressure ulcer on the coccyx c. The patient who has been receiving chemotherapy and has a temperature of 102° F d. The patient who needs to be medicated with multiple analgesics before a scheduled dressing change

ANS: C Chemotherapy is an immunosuppressant. Even a low fever in an immunosuppressed patient is a sign of serious infection and should be treated immediately with cultures and rapid initiation of antibiotic therapy. The nurse should assess the other patients as soon as possible after assessing and implementing appropriate care for the immunosuppressed patient

A 49-year-old patient with multiple sclerosis (MS) is to begin treatment with glatiramer acetate (Copaxone). Which information will the nurse include in patient teaching? a. Recommendation to drink at least 4 L of fluid daily b. Need to avoid driving or operating heavy machinery c. How to draw up and administer injections of the medication d. Use of contraceptive methods other than oral contraceptives

ANS: C Copaxone is administered by self-injection. Oral contraceptives are an appropriate choice for birth control. There is no need to avoid driving or drink large fluid volumes when taking glatiramer

A 40-year-old patient is diagnosed with early Huntington's disease (HD). When teaching the patient, spouse, and children about this disorder, the nurse will provide information about the a. use of levodopa-carbidopa (Sinemet) to help reduce HD symptoms. b. prophylactic antibiotics to decrease the risk for aspiration pneumonia. c. option of genetic testing for the patient's children to determine their own HD risks. d. lifestyle changes of improved nutrition and exercise that delay disease progression.

ANS: C Genetic testing is available to determine whether an asymptomatic individual has the HD gene. The patient and family should be informed of the benefits and problems associated with genetic testing. Sinemet will increase symptoms of HD because HD involves an increase in dopamine. Antibiotic therapy will not reduce the risk for aspiration. There are no effective treatments or lifestyle changes that delay the progression of symptoms in HD.

The nurse is reviewing laboratory results on a patient who had a large burn 48 hours ago. Which result requires priority action by the nurse? a. Hematocrit 53% b. Serum sodium 147 mEq/L c. Serum potassium 6.1 mEq/L d. Blood urea nitrogen 37 mg/dL

ANS: C Hyperkalemia can lead to fatal dysrhythmias and indicates that the patient requires cardiac monitoring and immediate treatment to lower the potassium level. The other laboratory values are also abnormal and require changes in treatment, but they are not as immediately life threatening as the elevated potassium level

The nurse determines that teaching about management of migraine headaches has been effective when the patient says which of the following? a. "I can take the (Topamax) as soon as a headache starts." b. "A glass of wine might help me relax and prevent a headache." c. "I will lie down someplace dark and quiet when the headaches begin." d. "I should avoid taking aspirin and sumatriptan (Imitrex) at the same time."

ANS: C It is recommended that the patient with a migraine rest in a dark, quiet area. Topiramate (Topamax) is used to prevent migraines and must be taken for several months to determine effectiveness. Aspirin or other nonsteroidal antiinflammatory medications can be taken with the triptans. Alcohol may precipitate migraine headaches

3. A 19-year-old patient with massive trauma and possible spinal cord injury is admitted to the emergency department (ED). Which assessment finding by the nurse will help confirm a diagnosis of neurogenic shock? a. Inspiratory crackles. b. Cool, clammy extremities. c. Apical heart rate 45 beats/min. d. Temperature 101.2° F (38.4° C).

ANS: C Neurogenic shock is characterized by hypotension and bradycardia. The other findings would be more consistent with other types of shock. DIF: Cognitive Level: Understand (comprehension) REF: 1634 TOP: Nursing Process: Assessment MSC: NCLEX: Physiological Integrity

4. An older patient with cardiogenic shock is cool and clammy and hemodynamic monitoring indicates a high systemic vascular resistance (SVR). Which intervention should the nurse anticipate doing next? a. Increase the rate for the dopamine (Intropin) infusion. b. Decrease the rate for the nitroglycerin (Tridil) infusion. c. Increase the rate for the sodium nitroprusside (Nipride) infusion. d. Decrease the rate for the 5% dextrose in normal saline (D5/.9 NS) infusion.

ANS: C Nitroprusside is an arterial vasodilator and will decrease the SVR and afterload, which will improve cardiac output. Changes in the D5/.9 NS and nitroglycerin infusions will not directly decrease SVR. Increasing the dopamine will tend to increase SVR. DIF: Cognitive Level: Apply (application) REF: 1644 TOP: Nursing Process: Planning MSC: NCLEX: Physiological Integrity

21. The patient with neurogenic shock is receiving a phenylephrine (Neo-Synephrine) infusion through a right forearm IV. Which assessment finding obtained by the nurse indicates a need for immediate action? a. The patient's heart rate is 58 beats/minute. b. The patient's extremities are warm and dry. c. The patient's IV infusion site is cool and pale. d. The patient's urine output is 28 mL over the last hour.

ANS: C The coldness and pallor at the infusion site suggest extravasation of the phenylephrine. The nurse should discontinue the IV and, if possible, infuse the medication into a central line. An apical pulse of 58 is typical for neurogenic shock but does not indicate an immediate need for nursing intervention. A 28-mL urinary output over 1 hour would require the nurse to monitor the output over the next hour, but an immediate change in therapy is not indicated. Warm, dry skin is consistent with early neurogenic shock, but it does not indicate a need for a change in therapy or immediate action. DIF: Cognitive Level: Apply (application) REF: 1643 OBJ: Special Questions: Prioritization TOP: Nursing Process: Assessment MSC: NCLEX: Physiological Integrity

20. A patient who has been involved in a motor vehicle crash arrives in the emergency department (ED) with cool, clammy skin; tachycardia; and hypotension. Which intervention ordered by the health care provider should the nurse implement first? a. Insert two large-bore IV catheters. b. Initiate continuous electrocardiogram (ECG) monitoring. c. Provide oxygen at 100% per non-rebreather mask. d. Draw blood to type and crossmatch for transfusions.

ANS: C The first priority in the initial management of shock is maintenance of the airway and ventilation. ECG monitoring, insertion of IV catheters, and obtaining blood for transfusions should also be rapidly accomplished but only after actions to maximize oxygen delivery have been implemented. DIF: Cognitive Level: Apply (application) REF: 1641 OBJ: Special Questions: Prioritization TOP: Nursing Process: Implementation MSC: NCLEX: Physiological Integrity

A 22-year-old patient seen at the health clinic with a severe migraine headache tells the nurse about having other similar headaches recently. Which initial action should the nurse take? a. Teach about the use of triptan drugs. b. Refer the patient for stress counseling. c. Ask the patient to keep a headache diary. d. Suggest the use of muscle-relaxation techniques.

ANS: C The initial nursing action should be further assessment of the precipitating causes of the headaches, quality, and location of pain, etc. Stress reduction, muscle relaxation, and the triptan drugs may be helpful, but more assessment is needed first

A patient with type 2 diabetes is admitted for an outpatient coronary arteriogram. Which information obtained by the nurse is most important to report to the health care provider before the procedure? a. The patient's admission blood glucose is 128 mg/dL. b. The patient's most recent Hb A1C was 6.5%. c. The patient took the prescribed metformin (Glucophage) today. d. The patient took the prescribed captopril (Capoten) this morning.

ANS: C The patient took the prescribed metformin (Glucophage) today. To avoid lactic acidosis, metformin should be discontinued a day or 2 before the coronary arteriogram and should not be used for 48 hours after IV contrast media are administered. The other patient data also will be reported but do not indicate any need to reschedule the procedure.

25. After reviewing the information shown in the accompanying figure for a patient with pneumonia and sepsis, which information is most important to report to the health care provider? Petechiae noted on chest and legs Cracks heard bilaterally in lung bases no redness or swelling at IV site BUN 34 Hemaocrit 30% platelets 50,000/mL Temp 100F Pulse 102 Respirations 26 BP 110/60 O2 93% on 2L O2 NC a. Temperature and IV site appearance b. Oxygen saturation and breath sounds c. Platelet count and presence of petechiae d. Blood pressure, pulse rate, respiratory rate.

ANS: C The low platelet count and presence of petechiae suggest that the patient may have disseminated intravascular coagulation and that multiple organ dysfunction syndrome (MODS) is developing. The other information will also be discussed with the health care provider but does not indicate that the patient's condition is deteriorating or that a change in therapy is needed immediately. DIF: Cognitive Level: Analyze (analysis) REF: 1640 OBJ: Special Questions: Prioritization TOP: Nursing Process: Assessment MSC: NCLEX: Physiological Integrity

A patient with a systemic bacterial infection feels cold and has a shaking chill. Which assessment finding will the nurse expect next? a. Skin flushing b. Muscle cramps c. Rising body temperature d. Decreasing blood pressure

ANS: C The patient's complaints of feeling cold and shivering indicate that the hypothalamic set point for temperature has been increased and the temperature is increasing. Because associated peripheral vasoconstriction and sympathetic nervous system stimulation will occur, skin flushing and hypotension are not expected. Muscle cramps are not expected with chills and shivering or with a rising temperature

Which statement by a 24-year-old patient indicates that the nurse's teaching about management of primary genital herpes has been effective? a. "I will use acyclovir ointment on the area to relieve the pain." b. "I will use condoms for intercourse until the medication is all gone." c. "I will take the acyclovir (Zovirax) every 8 hours for the next week." d. "I will need to take all of the medication to be sure the infection is cured."

ANS: C The treatment regimen for primary genital herpes infections includes acyclovir 400 mg 3 times daily for 7 to 10 days. The patient is taught to abstain from intercourse until the lesions are gone. (Condoms should be used even when the patient is asymptomatic.) Acyclovir ointment is not effective in treating lesions or reducing pain. Herpes infection is chronic and recurrent

A patient's 4 ´ 3-cm leg wound has a 0.4 cm black area in the center of the wound surrounded by yellow-green semiliquid material. Which dressing should the nurse apply to the wound? a. Dry gauze dressing (Kerlix) b. Nonadherent dressing (Xeroform) c. Hydrocolloid dressing (DuoDerm) d. Transparent film dressing (Tegaderm)

ANS: C The wound requires debridement of the necrotic areas and absorption of the yellow-green slough. A hydrocolloid dressing such as DuoDerm would accomplish these goals. Transparent film dressings are used for red wounds or approximated surgical incisions. Dry dressings will not debride the necrotic areas. Nonadherent dressings will not absorb wound drainage or debride the wound

12. A nurse is assessing a patient who is receiving a nitroprusside (Nipride) infusion to treat cardiogenic shock. Which finding indicates that the medication is effective? a. No new heart murmurs b. Decreased troponin level c. Warm, pink, and dry skin d. Blood pressure 92/40 mm Hg

ANS: C Warm, pink, and dry skin indicates that perfusion to tissues is improved. Since nitroprusside is a vasodilator, the blood pressure may be low even if the medication is effective. Absence of a heart murmur and a decrease in troponin level are not indicators of improvement in shock. DIF: Cognitive Level: Apply (application) REF: 1644 TOP: Nursing Process: Evaluation MSC: NCLEX: Physiological Integrity

After the nurse has taught a patient with a newly diagnosed sexually transmitted infection about expedited partner therapy, which patient statement indicates that the teaching has been effective? a. "I will tell my partner that it is important to be examined at the clinic." b. "I will have my partner take the antibiotics if any STI symptoms occur." c. "I will make sure that my partner takes all of the prescribed medication." d. "I will have my partner use a condom until I have finished the antibiotics."

ANS: C With expedited partner therapy, the patient is given a prescription or medications for the partner. The partner does not need to be evaluated by the health care provider, but is presumed to be infected and should be treated concurrently with the patient. Use of a condom will not treat the presumed STI in the partner

An 18-year-old with newly diagnosed type 1 diabetes has received diet instruction. The nurse determines a need for additional instruction when the patient says, a. "I may have an occasional alcoholic drink if I include it in my meal plan." b. "I will need a bedtime snack because I take an evening dose of NPH insulin." c. "I may eat whatever I want, as long as I use enough insulin to cover the calories." d. "I will eat meals as scheduled, even if I am not hungry, to prevent hypoglycemia."

ANS: C "I may eat whatever I want, as long as I use enough insulin to cover the calories." Most patients with type 1 diabetes need to plan diet choices very carefully. Patients who are using intensified insulin therapy have considerable flexibility in diet choices but still should restrict dietary intake of items such as fat, protein, and alcohol. The other patient statements are correct and indicate good understanding of the diet instruction.

The nurse and LPN/LVN are caring for a type 2 diabetic patient who is admitted for gallbladder surgery. Which nursing action can the nurse delegate to the LPN/LVN? a. Communicate the blood glucose and insulin dose to the circulating nurse in surgery. b. Discuss the reason for the use of insulin therapy during the immediate postoperative period. c. Administer the prescribed lispro (Humalog) insulin before transferring the patient to surgery. d. Plan strategies to minimize the risk for hypo- or hyperglycemia during the postoperative hospitalization.

ANS: C Administer the prescribed lispro (Humalog) insulin before transferring the patient to surgery. LPN/LVN education and scope of practice includes administration of insulin. Communication about patient status with other departments, planning, and patient teaching are skills that require RN education and scope of practice.

A patient screened for diabetes at a clinic has a fasting plasma glucose level of 120mg/dL (6.7 mmol/L). The nurse will plan to teach the patient about a. self-monitoring of blood glucose. b. use of low doses of regular insulin. c. lifestyle changes to lower blood glucose. d. effects of oral hypoglycemic medications.

ANS: C lifestyle changes to lower blood glucose. The patient's impaired fasting glucose indicates prediabetes and the patient should be counseled about lifestyle changes to prevent the development of type 2 diabetes. The patient with prediabetes does not require insulin or the oral hypoglycemics for glucose control and does not need to self-monitor blood glucose.

13. Which assessment information is most important for the nurse to obtain to evaluate whether treatment of a patient with anaphylactic shock has been effective? a. Heart rate b. Orientation c. Blood pressure d. Oxygen saturation

ANS: D Because the airway edema that is associated with anaphylaxis can affect airway and breathing, the oxygen saturation is the most critical assessment. Improvements in the other assessments will also be expected with effective treatment of anaphylactic shock. DIF: Cognitive Level: Apply (application) REF: 1646 TOP: Nursing Process: Evaluation MSC: NCLEX: Physiological Integrity

A patient with rheumatoid arthritis has been taking corticosteroids for 11 months. Which nursing action is most likely to detect early signs of infection in this patient? a. Monitor white blood cell count. b. Check the skin for areas of redness. c. Check the temperature every 2 hours. d. Ask about fatigue or feelings of malaise.

ANS: D Common clinical manifestations of inflammation and infection are frequently not present when patients receive immunosuppressive medications. The earliest manifestation of an infection may be "just not feeling well."

Which information about a 60-year-old patient with MS indicates that the nurse should consult with the health care provider before giving the prescribed dose of dalfampridine (Ampyra)? a. The patient has relapsing-remitting MS. b. The patient walks a mile a day for exercise. c. The patient complains of pain with neck flexion. d. The patient has an increased serum creatinine level.

ANS: D Dalfampridine should not be given to patients with impaired renal function. The other information will not impact whether the dalfampridine should be administered

A 76-year-old patient is being treated with carbidopa/levodopa (Sinemet) for Parkinson's disease. Which information is most important for the nurse to report to the health care provider? a. Shuffling gait b. Tremor at rest c. Cogwheel rigidity of limbs d. Uncontrolled head movement

ANS: D Dyskinesia is an adverse effect of the Sinemet, indicating a need for a change in medication or decrease in dose. The other findings are typical with Parkinson's disease.

A 76-year-old patient is being treated with carbidopa/levodopa (Sinemet) for Parkinson's disease. Which information is most important for the nurse to report to the health care provider? a. Shuffling gait b. Tremor at rest c. Cogwheel rigidity of limbs d. Uncontrolled head movement

ANS: D Dyskinesia is an adverse effect of the Sinemet, indicating a need for a change in medication or decrease in dose. The other findings are typical with Parkinson's disease.

A 62-year-old patient who has Parkinson's disease is taking bromocriptine (Parlodel). Which information obtained by the nurse may indicate a need for a decrease in the dose? a. The patient has a chronic dry cough. b. The patient has four loose stools in a day. c. The patient develops a deep vein thrombosis. d. The patient's blood pressure is 92/52 mm Hg.

ANS: D Hypotension is an adverse effect of bromocriptine, and the nurse should check with the health care provider before giving the medication. Diarrhea, cough, and deep vein thrombosis are not associated with bromocriptine use

A 62-year-old patient who has Parkinson's disease is taking bromocriptine (Parlodel). Which information obtained by the nurse may indicate a need for a decrease in the dose? a. The patient has a chronic dry cough. b. The patient has four loose stools in a day. c. The patient develops a deep vein thrombosis. d. The patient's blood pressure is 92/52 mm Hg.

ANS: D Hypotension is an adverse effect of bromocriptine, and the nurse should check with the health care provider before giving the medication. Diarrhea, cough, and deep vein thrombosis are not associated with bromocriptine use

A 32-year-old man who has a profuse, purulent urethral discharge with painful urination is seen at the clinic. Which information will be most important for the nurse to obtain? a. Contraceptive use b. Sexual orientation c. Immunization history d. Recent sexual contacts

ANS: D Information about sexual contacts is needed to help establish whether the patient has been exposed to a sexually transmitted infection (STI) and because sexual contacts also will need treatment. The other information also may be gathered but is not as important in determining the plan of care for the patient's current symptoms

The nurse could delegate care of which patient to a licensed practical/vocational nurse (LPN/LVN)? a. The patient who has increased tenderness and swelling around a leg wound b. The patient who was just admitted after suturing of a full-thickness arm wound c. The patient who needs teaching about home care for a draining abdominal wound d. The patient who requires a hydrocolloid dressing change for a stage III sacral ulcer

ANS: D LPN/LVN education and scope of practice include sterile dressing changes for stable patients. Initial wound assessments, patient teaching, and evaluation for possible poor wound healing or infection should be done by the registered nurse (RN).

Which information about a 72-year-old patient who has a new prescription for phenytoin (Dilantin) indicates that the nurse should consult with the health care provider before administration of the medication? a. Patient has generalized tonic-clonic seizures. b. Patient experiences an aura before seizures. c. Patient's most recent blood pressure is 156/92 mm Hg. d. Patient has minor elevations in the liver function tests.

ANS: D Many older patients (especially with compromised liver function) may not be able to metabolize phenytoin. The health care provider may need to choose another antiseizure medication. Phenytoin is an appropriate medication for patients with tonic-clonic seizures, with or without an aura. Hypertension is not a contraindication for phenytoin therapy.

The nurse will perform which action when doing a wet-to-dry dressing change on a patient's stage III sacral pressure ulcer? a. Soak the old dressings with sterile saline 30 minutes before removing them. b. Pour sterile saline onto the new dry dressings after the wound has been packed. c. Apply antimicrobial ointment before repacking the wound with moist dressings. d. Administer the ordered PRN hydrocodone (Lortab) 30 minutes before the dressing change.

ANS: D Mechanical debridement with wet-to-dry dressings is painful, and patients should receive pain medications before the dressing change begins. The new dressings are moistened with saline before being applied to the wound. Soaking the old dressings before removing them will eliminate the wound debridement that is the purpose of this type of dressing. Application of antimicrobial ointments is not indicated for a wet-to-dry dressing

A young adult patient who is receiving antibiotics for an infected leg wound has a temperature of 101.8° F (38.7° C). Which action by the nurse is most appropriate? a. Apply a cooling blanket. b. Notify the health care provider. c. Give the prescribed PRN aspirin (Ascriptin) 650 mg. d. Check the patient's oral temperature again in 4 hours.

ANS: D Mild to moderate temperature elevations (less than 103° F) do not harm the young adult patient and may benefit host defense mechanisms. The nurse should continue to monitor the temperature. Antipyretics are not indicated unless the patient is complaining of fever-related symptoms. There is no need to notify the patient's health care provider or to use a cooling blanket for a moderate temperature elevation

A 55-year-old woman in the sexually transmitted infection (STI) clinic tells the nurse that she is concerned she may have been exposed to gonorrhea by her partner. To determine whether the patient has gonorrhea, the nurse will plan to a. interview the patient about symptoms of gonorrhea. b. take a sample of cervical discharge for Gram staining. c. draw a blood specimen or rapid plasma reagin (RPR) testing. d. obtain secretions for a nucleic acid amplification test (NAAT).

ANS: D NAAT has a high sensitivity (similar to a culture) for gonorrhea. Because women have few symptoms of gonorrhea, asking the patient about symptoms may not be helpful in making a diagnosis. Smears and Gram staining are not useful because the female genitourinary tract has many normal flora that resemble N. gonorrhoeae. RPR testing is used to detect syphilis

16. When the nurse educator is evaluating the skills of a new registered nurse (RN) caring for patients experiencing shock, which action by the new RN indicates a need for more education? a. Placing the pulse oximeter on the ear for a patient with septic shock b. Keeping the head of the bed flat for a patient with hypovolemic shock c. Increasing the nitroprusside (Nipride) infusion rate for a patient with a high SVR d. Maintaining the room temperature at 66° to 68° F for a patient with neurogenic shock

ANS: D Patients with neurogenic shock may have poikilothermia. The room temperature should be kept warm to avoid hypothermia. The other actions by the new RN are appropriate. DIF: Cognitive Level: Apply (application) REF: 1634 | 1636 OBJ: Special Questions: Delegation TOP: Nursing Process: Evaluation MSC: NCLEX: Safe and Effective Care Environment

After the home health nurse teaches a patient's family member about how to care for a sacral pressure ulcer, which finding indicates that additional teaching is needed? a. The family member uses a lift sheet to reposition the patient. b. The family member uses clean tap water to clean the wound. c. The family member places contaminated dressings in a plastic grocery bag. d. The family member dries the wound using a hair dryer set on a low setting.

ANS: D Pressure ulcers need to be kept moist to facilitate wound healing. The other actions indicate a good understanding of pressure ulcer care

A new nurse performs a dressing change on a stage II left heel pressure ulcer. Which action by the new nurse indicates a need for further teaching about pressure ulcer care? a. The new nurse uses a hydrocolloid dressing (DuoDerm) to cover the ulcer. b. The new nurse inserts a sterile cotton-tipped applicator into the pressure ulcer. c. The new nurse irrigates the pressure ulcer with sterile saline using a 30-mL syringe. d. The new nurse cleans the ulcer with a sterile dressing soaked in half-strength peroxide.

ANS: D Pressure ulcers should not be cleaned with solutions that are cytotoxic, such as hydrogen peroxide. The other actions by the new nurse are appropriate

6. To evaluate the effectiveness of the pantoprazole (Protonix) ordered for a patient with systemic inflammatory response syndrome (SIRS), which assessment will the nurse perform? a. Auscultate bowel sounds. b. Palpate for abdominal pain. c. Ask the patient about nausea. d. Check stools for occult blood.

ANS: D Proton pump inhibitors are given to decrease the risk for stress ulcers in critically ill patients. The other assessments also will be done, but these will not help in determining the effectiveness of the pantoprazole administration. DIF: Cognitive Level: Apply (application) REF: 1646 TOP: Nursing Process: Evaluation MSC: NCLEX: Physiological Integrity

Which nursing diagnosis is of highest priority for a patient with Parkinson's disease who is unable to move the facial muscles? a. Activity intolerance b. Self-care deficit: toileting c. Ineffective self-health management d. Imbalanced nutrition: less than body requirements

ANS: D The data about the patient indicate that poor nutrition will be a concern because of decreased swallowing. The other diagnoses may also be appropriate for a patient with Parkinson's disease, but the data do not indicate that they are current problems for this patient

Which nursing diagnosis is of highest priority for a patient with Parkinson's disease who is unable to move the facial muscles? a. Activity intolerance b. Self-care deficit: toileting c. Ineffective self-health management d. Imbalanced nutrition: less than body requirements

ANS: D The data about the patient indicate that poor nutrition will be a concern because of decreased swallowing. The other diagnoses may also be appropriate for a patient with Parkinson's disease, but the data do not indicate that they are current problems for this patient

A young male patient who is a paraplegic has a stage II sacral pressure ulcer and is being cared for at home by his mother. To prevent further tissue damage, what instructions are most important for the nurse to teach the mother? a. Change the patient's bedding frequently. b. Use a hydrocolloid dressing over the ulcer. c. Record the size and appearance of the ulcer weekly. d. Change the patient's position at least every 2 hours.

ANS: D The most important intervention is to avoid prolonged pressure on bony prominences by frequent repositioning. The other interventions may also be included in family teaching, but the most important instruction is to change the patient's position at least every 2 hours

Which prescribed intervention will the nurse implement first for a patient in the emergency department who is experiencing continuous tonic-clonic seizures? a. Give phenytoin (Dilantin) 100 mg IV. b. Monitor level of consciousness (LOC). c. Obtain computed tomography (CT) scan. d. Administer lorazepam (Ativan) 4 mg IV.

ANS: D To prevent ongoing seizures, the nurse should administer rapidly acting antiseizure medications such as the benzodiazepines. A CT scan is appropriate, but prevention of any seizure activity during the CT scan is necessary. Phenytoin will also be administered, but it is not rapidly acting. Patients who are experiencing tonic-clonic seizures are nonresponsive, although the nurse should assess LOC after the seizure.

The nurse should plan to use a wet-to-dry dressing for which patient? a. A patient who has a pressure ulcer with pink granulation tissue b. A patient who has a surgical incision with pink, approximated edges c. A patient who has a full-thickness burn filled with dry, black material d. A patient who has a wound with purulent drainage and dry brown areas

ANS: D Wet-to-dry dressings are used when there is minimal eschar to be removed. A full-thickness wound filled with eschar will require interventions such as surgical debridement to remove the necrotic tissue. Wet-to-dry dressings are not needed on approximated surgical incisions. Wet-to-dry dressings are not used on uninfected granulating wounds because of the damage to the granulation tissue

During the emergent phase of burn care, which assessment will be most useful in determining whether the patient is receiving adequate fluid infusion? a. Check skin turgor. b. Monitor daily weight. c. Assess mucous membranes. d. Measure hourly urine output.

ANS: D When fluid intake is adequate, the urine output will be at least 0.5 to 1 mL/kg/hour. The patient's weight is not useful in this situation because of the effects of third spacing and evaporative fluid loss. Mucous membrane assessment and skin turgor also may be used, but they are not as adequate in determining that fluid infusions are maintaining adequate perfusion.

The nurse is caring for a patient with diabetes who had abdominal surgery 3 days ago. Which finding is most important for the nurse to report to the health care provider? a. Blood glucose 136 mg/dL b. Oral temperature 101° F (38.3° C) c. Patient complaint of increased incisional pain d. Separation of the proximal wound edges by 1 cm

ANS: D Wound separation 3 days postoperatively indicates possible wound dehiscence and should be immediately reported to the health care provider. The other findings will also be reported but do not require intervention as rapidly

Which patient statement after the nurse has completed teaching a patient with type 2 diabetes about taking glipizide (Glucotrol) indicates a need for additional teaching? a. "Other medications besides the Glucotrol may affect my blood sugar." b. "If I overeat at a meal, I will still take just the usual dose of medication." c. "When I become ill, I may have to take insulin to control my blood sugar." d. "My diabetes is not as likely to cause complications as if I needed to take insulin."

ANS: D "My diabetes is not as likely to cause complications as if I needed to take insulin." The patient should understand that type 2 diabetes places the patient at risk for many complications and that good glucose control is as important when taking oral agents as when using insulin. The other statements are accurate and indicate good understanding of the use of glipizide.

Amitriptyline (Elavil) is prescribed for a diabetic patient who has burning foot pain at night. Which information should the nurse include when teaching the patient about the new medication? a. Amitriptyline will decrease the depression caused by your foot pain. b. Amitriptyline will correct some of the blood vessel changes that cause pain. c. Amitriptyline will improve sleep and make you less aware of nighttime pain. d. Amitriptyline will help prevent the transmission of pain impulses to the brain.

ANS: D Amitriptyline will help prevent the transmission of pain impulses to the brain. Tricyclic antidepressants decrease the transmission of pain impulses to the spinal cord and brain. Tricyclics also improve sleep quality and are used for depression, but that is not the major purpose for their use in diabetic neuropathy. The blood vessel changes that contribute to neuropathy are not affected by tricyclics.

A patient with newly diagnosed type 2 diabetes mellitus asks the nurse what "type 2" means in relation to diabetes. Which statement by the nurse about type 2 diabetes is correct? a. Insulin is not used to control blood glucose in patients with type 2 diabetes. b. Complications of type 2 diabetes are less serious than those of type 1 diabetes. c. Type 2 diabetes is usually diagnosed when the patient is admitted with a hyperglycemic coma. d. Changes in diet and exercise may be sufficient to control blood glucose levels in type 2 diabetes.

ANS: D Changes in diet and exercise may be sufficient to control blood glucose levels in type 2 diabetes. For some patients, changes in lifestyle are sufficient for blood glucose control. Insulin is frequently used for type 2 diabetes, complications are equally severe as for type 1 diabetes, and type 2 diabetes is usually diagnosed with routine laboratory testing or after a patient develops complications such as frequent yeast infections.

To evaluate the effectiveness of treatment for a patient with type 2 diabetes who is scheduled for a follow-up visit in the clinic, which test will the nurse plan to schedule for the patient? a. Urine dipstick for glucose b. Oral glucose tolerance test c. Fasting blood glucose level d. Glycosylated hemoglobin level

ANS: D Glycosylated hemoglobin level The glycosylated hemoglobin (Hb A1C) test shows the overall control of glucose over 90 to 120 days. A fasting blood level indicates only the glucose level at one time. Urine glucose testing is not an accurate reflection of blood glucose level and does not reflect the glucose over a prolonged time. Oral glucose tolerance testing is done to diagnose diabetes, but is not used for monitoring glucose control once diabetes has been diagnosed.

After the home health nurse has taught a patient and family about how to use glargine and regular insulin safely, which action by the patient indicates that the teaching has been successful? a. The patient administers the glargine 30 to 45 minutes before eating each meal. b. The patient's family fills the syringes weekly and stores them in the refrigerator. c. The patient draws up the regular insulin and then the glargine in the same syringe. d. The patient disposes of the open vials of glargine and regular insulin after 4 weeks.

ANS: D The patient disposes of the open vials of glargine and regular insulin after 4 weeks. Insulin can be stored at room temperature for 4 weeks. Glargine should not be mixed with other insulins or prefilled and stored. Short-acting regular insulin is administered before meals, while glargine is given once daily.

Which information about a patient who receives rosiglitazone (Avandia) is most important for the nurse to report immediately to the health care provider? a. The patient's blood pressure is 154/92. b. The patient has a history of emphysema. c. The patient's noon blood glucose is 86 mg/dL. d. The patient has chest pressure when ambulating.

ANS: D The patient has chest pressure when ambulating. Rosiglitazone can cause myocardial ischemia. The nurse should immediately notify the health care provider and expect orders to discontinue the medication. There is no urgent need to discuss the other data with the health care provider.

Which action by a type 1 diabetic patient indicates that the nurse should implement teaching about exercise and glucose control? a. The patient always carries hard candies when engaging in exercise. b. The patient goes for a vigorous walk when the glucose is 200 mg/dL. c. The patient has a peanut butter sandwich before going for a bicycle ride. d. The patient increases daily exercise when ketones are present in the urine.

ANS: D The patient increases daily exercise when ketones are present in the urine. When the patient is ketotic, exercise may result in an increase in blood glucose level. Type 1 diabetic patients should be taught to avoid exercise when ketosis is present. The other statements are correct.

The nurse teaches the diabetic patient who rides a bicycle to work every day to administer morning insulin into the a. arm. b. thigh. c. buttock. d. abdomen.

ANS: D abdomen. Patients should be taught not to administer insulin into a site that will be exercised because exercise will increase the rate of absorption. The thigh, buttock, and arm are all exercised by riding a bicycle.

A hospitalized diabetic patient who received 34 U of NPH insulin at 7:00 AM is away from the nursing unit, awaiting diagnostic testing when lunch trays are distributed. To prevent hypoglycemia, the best action by the nurse is to a. save the lunch tray to be provided upon the patient's return to the unit. b. call the diagnostic testing area and ask that a 5% dextrose IV be started. c. ensure that the patient drinks a glass of milk or orange juice at noon in the diagnostic testing area. d. request that the patient be returned to the unit to eat lunch if testing will not be completed promptly.

ANS: D request that the patient be returned to the unit to eat lunch if testing will not be Consistency for mealtimes assists with regulation of blood glucose, so the best option is for the patient to have lunch at the usual time. Waiting to eat until after the procedure is likely to cause hypoglycemia. Administration of an IV solution is unnecessarily invasive for the patient. A glass of milk or juice will keep the patient from becoming hypoglycemic but will cause a rapid rise in blood glucose because of the rapid absorption of the simple carbohydrate in these items.

A public health nurse has reviewed local data about the incidence and prevalence of burn injuries in the community. These data are likely to support what health promotion effort? A) Education about home safety B) Education about safe storage of chemicals C) Education about workplace health threats D) Education about safe driving

Ans: A Feedback: A large majority of burns occur in the home setting; educational interventions should address this epidemiologic trend.

A nurse on a burn unit is caring for a patient in the acute phase of burn care. While performing an assessment during this phase of burn care, the nurse recognizes that airway obstruction related to upper airway edema may occur up to how long after the burn injury? A) 2 days B) 3 days C) 5 days D) 1 week

Ans: A Feedback: Airway obstruction caused by upper airway edema can take as long as 48 hours to develop. Changes detected by x-ray and arterial blood gases may occur as the effects of resuscitative fluid and the chemical reaction of smoke ingredients with lung tissues become apparent.

A patient is admitted to the burn unit after being transported from a facility 1000 miles away. The patient has burns to the groin area and circumferential burns to both upper thighs. When assessing the patient's legs distal to the wound site, the nurse should be cognizant of the risk of what complication? A) Ischemia B) Referred pain C) Cellulitis D) Venous thromboembolism (VTE)

Ans: A Feedback: As edema increases, pressure on small blood vessels and nerves in the distal extremities causes an obstruction of blood flow and consequent ischemia. This complication is similar to compartment syndrome. Referred pain, cellulitis, and VTE are not noted complications that occur distal to the injury site.

A burn patient is transitioning from the acute phase of the injury to the rehabilitation phase. The patient tells the nurse, ìI can't wait to have surgery to reconstruct my face so I look normal again.î What would be the nurse's best response? A) ìThat's something that you and your doctor will likely talk about after your scars mature.î B) ìThat is something for you to talk to your doctor about because it's not a nursing responsibility.î C) ìI know this is really important to you, but you have to realize that no one can make you look like you used to.î D) ìUnfortunately, it's likely that you will have most of these scars for the rest of your life.î

Ans: A Feedback: Burn reconstruction is a treatment option after all scars have matured and is discussed within the first few years after injury. Even though this is not a nursing responsibility, the nurse should still respond appropriately to the patient's query. It is true that the patient will not realistically look like he or she used to, but this does not instill hope.

A patient has been admitted to a burn intensive care unit with extensive full-thickness burns over 25% of the body. After ensuring cardiopulmonary stability, what would be the nurse's immediate, priority concern when planning this patient's care? A) Fluid status B) Risk of infection C) Nutritional status D) Psychosocial coping

Ans: A Feedback: During the early phase of burn care, the nurse is most concerned with fluid resuscitation, to correct large-volume fluid loss through the damaged skin. Infection control and early nutritional support are important, but fluid resuscitation is an immediate priority. Coping is a higher priority later in the recovery period.

A patient in the emergent/resuscitative phase of a burn injury has had blood work and arterial blood gases drawn. Upon analysis of the patient's laboratory studies, the nurse will expect the results to indicate what? A) Hyperkalemia, hyponatremia, elevated hematocrit, and metabolic acidosis B) Hypokalemia, hypernatremia, decreased hematocrit, and metabolic acidosis C) Hyperkalemia, hypernatremia, decreased hematocrit, and metabolic alkalosis D) Hypokalemia, hyponatremia, elevated hematocrit, and metabolic alkalosis

Ans: A Feedback: Fluid and electrolyte changes in the emergent/resuscitative phase of a burn injury include hyperkalemia related to the release of potassium into the extracellular fluid, hyponatremia from large amounts of sodium lost in trapped edema fluid, hemoconcentration that leads to an increased hematocrit, and loss of bicarbonate ions that results in metabolic acidosis.

A patient has sustained a severe burn injury and is thought to have an impaired intestinal mucosal barrier. Since this patient is considered at an increased risk for infection, what intervention will best assist in avoiding increased intestinal permeability and prevent early endotoxin translocation? A) Early enteral feeding B) Administration of prophylactic antibiotics C) Bowel cleansing procedures D) Administration of stool softeners

Ans: A Feedback: If the intestinal mucosa receives some type of protection against permeability change, infection could be avoided. Early enteral feeding is one step to help avoid this increased intestinal permeability and prevent early endotoxin translocation. Antibiotics are seldom prescribed prophylactically because of the risk of promoting resistant strains of bacteria. A bowel cleansing procedure would not be ordered for this patient. The administration of stool softeners would not assist in avoiding increased intestinal permeability and prevent early endotoxin translocation.

An emergency department nurse has just received a patient with burn injuries brought in by ambulance. The paramedics have started a large-bore IV and covered the burn in cool towels. The burn is estimated as covering 24% of the patient's body. How should the nurse best address the pathophysiologic changes resulting from major burns during the initial burn-shock period? A) Administer IV fluids B) Administer broad-spectrum antibiotics C) Administer IV potassium chloride D) Administer packed red blood cells

Ans: A Feedback: Pathophysiologic changes resulting from major burns during the initial burn-shock period include massive fluid losses. Addressing these losses is a major priority in the initial phase of treatment. Antibiotics and PRBCs are not normally administered. Potassium chloride would exacerbate the patient's hyperkalemia.

An emergency department nurse learns from the paramedics that they are transporting a patient who has suffered injury from a scald from a hot kettle. What variables will the nurse consider when determining the depth of burn? A) The causative agent B) The patient's preinjury health status C) The patient's prognosis for recovery D) The circumstances of the accident

Ans: A Feedback: The following factors are considered in determining the depth of a burn: how the injury occurred, causative agent (such as flame or scalding liquid), temperature of the burning agent, duration of contact with the agent, and thickness of the skin. The patient's preinjury status, circumstances of the accident, and prognosis for recovery are important, but are not considered when determining the depth of the burn.

A patient who is in the acute phase of recovery from a burn injury has yet to experience adequate pain control. What pain management strategy is most likely to meet this patient's needs? A) A patient-controlled analgesia (PCA) system B) Oral opioids supplemented by NSAIDs C) Distraction and relaxation techniques supplemented by NSAIDs D) A combination of benzodiazepines and topical anesthetics

Ans: A Feedback: The goal of treatment is to provide a long-acting analgesic that will provide even coverage for this long-term discomfort. It is helpful to use escalating doses when initiating the medication to reach the level of pain control that is acceptable to the patient. The use of patient-controlled analgesia (PCA) gives control to the patient and achieves this goal. Patients cannot normally achieve adequate pain control without the use of opioids, and parenteral administration is usually required.

A patient is brought to the emergency department with a burn injury. The nurse knows that the first systemic event after a major burn injury is what? A) Hemodynamic instability B) Gastrointestinal hypermotility C) Respiratory arrest D) Hypokalemia

Ans: A Feedback: The initial systemic event after a major burn injury is hemodynamic instability, which results from loss of capillary integrity and a subsequent shift of fluid, sodium, and protein from the intravascular space into the interstitial spaces. This precedes GI changes. Respiratory arrest may or may not occur, largely depending on the presence or absence of smoke inhalation. Hypokalemia does not take place in the initial phase of recovery.

A nurse has reported for a shift at a busy burns and plastics unit in a large university hospital. Which patient is most likely to have life-threatening complications? A) A 4-year-old scald victim burned over 24% of the body B) A 27-year-old male burned over 36% of his body in a car accident C) A 39-year-old female patient burned over 18% of her body D) A 60-year-old male burned over 16% of his body in a brush fire

Ans: A Feedback: Young children and the elderly continue to have increased morbidity and mortality when compared to other age groups with similar injuries and present a challenge for burn care. This is an important factor when determining the severity of injury and possible outcome for the patient.

A patient is in the acute phase of a burn injury. One of the nursing diagnoses in the plan of care is Ineffective Coping Related to Trauma of Burn Injury. What interventions appropriately address this diagnosis? Select all that apply. A) Promote truthful communication. B) Avoid asking the patient to make decisions. C) Teach the patient coping strategies. D) Administer benzodiazepines as ordered. E) Provide positive reinforcement.

Ans: A, C, E Feedback: The nurse can assist the patient to develop effective coping strategies by setting specific expectations for behavior, promoting truthful communication to build trust, helping the patient practice appropriate strategies, and giving positive reinforcement when appropriate. The patient may benefit from being able to make decisions regarding his or her care. Benzodiazepines may be needed for short-term management of anxiety, but they are not used to enhance coping.

A patient's burns have required a homograft. During the nurse's most recent assessment, the nurse observes that the graft is newly covered with purulent exudate. What is the nurse's most appropriate response? A) Perform mechanical dÈbridement to remove the exudate and prevent further infection. B) Inform the primary care provider promptly because the graft may need to be removed. C) Perform range of motion exercises to increase perfusion to the graft site and facilitate healing. D) Document this finding as an expected phase of graft healing.

Ans: B Feedback: An infected graft may need to be removed, thus the care provider should be promptly informed. ROM exercises will not resolve this problem and the nurse would not independently perform dÈbridement.

A patient has experienced an electrical burn and has developed thick eschar over the burn site. Which of the following topical antibacterial agents will the nurse expect the physician to order for the wound? A) Silver sulfadiazine 1% (Silvadene) water-soluble cream B) Mafenide acetate 10% (Sulfamylon) hydrophilic-based cream C) Silver nitrate 0.5% aqueous solution D) Acticoat

Ans: B Feedback: Mafenide acetate 10% hydrophilic-based cream is the agent of choice when there is a need to penetrate thick eschar. Silver products do not penetrate eschar; Acticoat is a type of silver dressing.

The nurse is preparing the patient for mechanical dÈbridement and informs the patient that this will involve which of the following procedures? A) A spontaneous separation of dead tissue from the viable tissue B) Removal of eschar until the point of pain and bleeding occurs C) Shaving of burned skin layers until bleeding, viable tissue is revealed D) Early closure of the wound

Ans: B Feedback: Mechanical dÈbridementcan be achieved through the use of surgical scissors, scalpels, or forceps to remove the eschar until the point of pain and bleeding occurs. Mechanical dÈbridement can also be accomplished through the use of topical enzymatic dÈbridement agents. The spontaneous separation of dead tissue from the viable tissue is an example of natural dÈbridement. Shaving the burned skin layers and early wound closure are examples of surgical dÈbridement.

A patient who was burned in a workplace accident has completed the acute phase of treatment and the plan of care has been altered to prioritize rehabilitation. What nursing action should be prioritized during this phase of treatment? A) Monitoring fluid and electrolyte imbalances B) Providing education to the patient and family C) Treating infection D) Promoting thermoregulation

Ans: B Feedback: Patient and family education is a priority during rehabilitation. There should be no fluid and electrolyte imbalances in the rehabilitation phase. The presence of impaired thermoregulation or infection would suggest that the patient is still in the acute phase of burn recovery.

A nurse who is taking care of a patient with burns is asked by a family member why the patient is losing so much weight. The patient is currently in the intermediate phase of recovery. What would be the nurse's most appropriate response to the family member? A) ìHe's on a calorie-restricted diet in order to divert energy to wound healing.î B) ìHis body has consumed his fat deposits for fuel because his calorie intake is lower than normal.î C) ìHe actually hasn't lost weight. Instead, there's been a change in the distribution of his body fat.î D) ìHe lost many fluids while he was being treated in the emergency phase of burn care.î

Ans: B Feedback: Patients lose a great deal of weight during recovery from severe burns. Reserve fat deposits are catabolized as a result of hypermetabolism. Patients are not placed on a calorie restriction during recovery and fluid losses would not account for weight loss later in the recovery period. Changes in the overall distribution of body fat do not occur.

A home care nurse is performing a visit to a patient's home to perform wound care following the patient's hospital treatment for severe burns. While interacting with the patient, the nurse should assess for evidence of what complication? A) Psychosis B) Post-traumatic stress disorder C) Delirium D) Vascular dementia

Ans: B Feedback: Post-traumatic stress disorder (PTSD) is the most common psychiatric disorder in burn survivors, with a prevalence that may be as high as 45%. As a result, it is important for the nurse to assess for this complication of burn injuries. Psychosis, delirium, and dementia are not among the noted psychiatric and psychosocial complications of burns.

A nurse who provides care on a burn unit is preparing to apply a patient's ordered topical antibiotic ointment. What action should the nurse perform when administering this medication? A) Apply the new ointment without disturbing the existing layer of ointment. B) Apply the ointment using a sterile tongue depressor. C) Apply a layer of ointment approximately 1/16 inch thick. D) Gently irrigate the wound bed after applying the antibiotic ointment.

Ans: C Feedback: After removing the old ointment from the wound bed, the nurse should apply a layer of ointment 1/16-inch thick using clean gloves. The wound would not be irrigated after application of new ointment.

A triage nurse in the emergency department (ED) receives a phone call from a frantic father who saw his 4-year-old child tip a pot of boiling water onto her chest. The father has called an ambulance. What would the nurse in the ED receiving the call instruct the father to do? A) Cover the burn with ice and secure with a towel. B) Apply butter to the area that is burned. C) Immerse the child in a cool bath. D) Avoid touching the burned area under any circumstances.

Ans: C Feedback: After the flames or heat source have been removed or extinguished, the burned area and adherent clothing are soaked with cool water briefly to cool the wound and halt the burning process. Cool water is the best first-aid measure. Ice and butter are contraindicated. Appropriate first aid necessitates touching the burn.

A patient experienced a 33% TBSA burn 72 hours ago. The nurse observes that the patient's hourly urine output has been steadily increasing over the past 24 hours. How should the nurse best respond to this finding? A) Obtain an order to reduce the rate of the patient's IV fluid infusion. B) Report the patient's early signs of acute kidney injury (AKI). C) Recognize that the patient is experiencing an expected onset of diuresis. D) Administer sodium chloride as ordered to compensate for this fluid loss.

Ans: C Feedback: As capillaries regain integrity, 48 or more hours after the burn, fluid moves from the interstitial to the intravascular compartment and diuresis begins. This is an expected development and does not require a reduction in the IV infusion rate or the administration of NaCl. Diuresis is not suggestive of AKI.

A patient with severe burns is admitted to the intensive care unit to stabilize and begin fluid resuscitation before transport to the burn center. The nurse should monitor the patient closely for what signs of the onset of burn shock? A) Confusion B) High fever C) Decreased blood pressure D) Sudden agitation

Ans: C Feedback: As fluid loss continues and vascular volume decreases, cardiac output continues to decrease and the blood pressure drops, marking the onset of burn shock. Shock and the accompanying hemodynamic changes are not normally accompanied by confusion, fever, or agitation.

A patient with a partial-thickness burn injury had Biobrane applied 2 weeks ago. The nurse notices that the Biobrane is separating from the burn wound. What is the nurse's most appropriate intervention? A) Reinforce the Biobrane dressing with another piece of Biobrane. B) Remove the Biobrane dressing and apply a new dressing. C) Trim away the separated Biobrane. D) Notify the physician for further emergency-related orders.

Ans: C Feedback: As the Biobrane gradually separates, it is trimmed, leaving a healed wound. When the Biobrane dressing adheres to the wound, the wound remains stable and the Biobrane can remain in place for 3 to 4 weeks. There is no need to reinforce the Biobrane nor to remove it and apply a new dressing. There is not likely any need to notify the physician for further orders.

A patient has experienced burns to his upper thighs and knees. Following the application of new wound dressings, the nurse should perform what nursing action? A) Instruct the patient to keep the wound site in a dependent position. B) Administer PRN analgesia as ordered. C) Assess the patient's peripheral pulses distal to the dressing. D) Assist with passive range of motion exercises to ìsetî the new dressing.

Ans: C Feedback: Dressings can impede circulation if they are wrapped too tightly. The peripheral pulses must be checked frequently and burned extremities elevated. Dependent positioning does not need to be maintained. PRN analgesics should be administered prior to the dressing change. ROM exercises do not normally follow a dressing change.

A nurse is teaching a patient with a partial-thickness wound how to wear his elastic pressure garment. How would the nurse instruct the patient to wear this garment? A) 4 to 6 hours a day for 6 months B) During waking hours for 2 to 3 months after the injury C) Continuously D) At night while sleeping for a year after the injury

Ans: C Feedback: Elastic pressure garments are worn continuously (i.e., 23 hours a day).

The current phase of a patient's treatment for a burn injury prioritizes wound care, nutritional support, and prevention of complications such as infection. Based on these care priorities, the patient is in what phase of burn care? A) Emergent B) Immediate resuscitative C) Acute D) Rehabilitation

Ans: C Feedback: The acute or intermediate phase of burn care follows the emergent/resuscitative phase and begins 48 to 72 hours after the burn injury. During this phase, attention is directed toward continued assessment and maintenance of respiratory and circulatory status, fluid and electrolyte balance, and gastrointestinal function. Infection prevention, burn wound care (i.e., wound cleaning, topical antibacterial therapy, wound dressing, dressing changes, wound dÈbridement, and wound grafting), pain management, and nutritional support are priorities at this stage. Priorities during the emergent or immediate resuscitative phase include first aid, prevention of shock and respiratory distress, detection and treatment of concomitant injuries, and initial wound assessment and care. The priorities during the rehabilitation phase include prevention of scars and contractures, rehabilitation, functional and cosmetic reconstruction, and psychosocial counseling.

A nurse is developing a care plan for a patient with a partial-thickness burn, and determines that an appropriate goal is to maintain position of joints in alignment. What is the best rationale for this intervention? A) To prevent neuropathies B) To prevent wound breakdown C) To prevent contractures D) To prevent heterotopic ossification

Ans: C Feedback: To prevent the complication of contractures, the nurse will establish a goal to maintain position of joints in alignment. Gentle range of motion exercises and a consult to PT and OT for exercises and positioning recommendations are also appropriate interventions for the prevention of contractures. Joint alignment is not maintained specifically for preventing neuropathy, wound breakdown, or heterotopic ossification.

A patient is brought to the emergency department from the site of a chemical fire, where he suffered a burn that involves the epidermis, dermis, and the muscle and bone of the right arm. On inspection, the skin appears charred. Based on these assessment findings, what is the depth of the burn on the patient's arm? A) Superficial partial-thickness B) Deep partial-thickness C) Full partial-thickness D) Full-thickness

Ans: D Feedback: A full-thickness burn involves total destruction of the epidermis and dermis and, in some cases, underlying tissue as well. Wound color ranges widely from white to red, brown, or black. The burned area is painless because the nerve fibers are destroyed. The wound can appear leathery; hair follicles and sweat glands are destroyed. Edema may also be present. Superficial partial-thickness burns involve the epidermis and possibly a portion of the dermis; the patient will experience pain that is soothed by cooling. Deep partial-thickness burns involve the epidermis, upper dermis, and portion of the deeper dermis; the patient will complain of pain and sensitivity to cold air. Full partial thickness is not a depth of burn.

The nurse caring for a patient who is recovering from full-thickness burns is aware of the patient's risk for contracture and hypertrophic scarring. How can the nurse best mitigate this risk? A) Apply skin emollients as ordered after granulation has occurred. B) Keep injured areas immobilized whenever possible to promote healing. C) Administer oral or IV corticosteroids as ordered. D) Encourage physical activity and range of motion exercises.

Ans: D Feedback: Exercise and the promotion of mobility can reduce the risk of contracture and hypertrophic scarring. Skin emollients are not normally used in the treatment of burns, and these do not prevent scarring. Steroids are not used to reduce scarring, as they also slow the healing process.

A patient's burns are estimated at 36% of total body surface area; fluid resuscitation has been ordered in the emergency department. After establishing intravenous access, the nurse should anticipate the administration of what fluid? A) 0.45% NaCl with 20 mEq/L KCl B) 0.45% NaCl with 40 mEq/L KCl C) Normal saline D) Lactated Ringer's

Ans: D Feedback: Fluid resuscitation with lactated Ringers (LR) should be initiated using the American Burn Association's (ABA) fluid resuscitation formulas. LR is the crystalloid of choice because its composition and osmolality most closely resemble plasma and because use of normal saline is associated with hyperchloremic acidosis. Potassium chloride solutions would exacerbate the hyperkalemia that occurs following burn injuries.

An occupational health nurse is called to the floor of a factory where a worker has sustained a flash burn to the right arm. The nurse arrives and the flames have been extinguished. The next step is to ìcool the burn.î How should the nurse cool the burn? A) Apply ice to the site of the burn for 5 to 10 minutes. B) Wrap the patient's affected extremity in ice until help arrives. C) Apply an oil-based substance or butter to the burned area until help arrives. D) Wrap cool towels around the affected extremity intermittently.

Ans: D Feedback: Once the burn has been sustained, the application of cool water is the best first-aid measure. Soaking the burn area intermittently in cool water or applying cool towels gives immediate and striking relief from pain, and limits local tissue edema and damage. However, never apply ice directly to the burn, never wrap the person in ice, and never use cold soaks or dressings for longer than several minutes; such procedures may worsen the tissue damage and lead to hypothermia in people with large burns. Butter is contraindicated.

A nurse is caring for a patient who has sustained a deep partial-thickness burn injury. In prioritizing the nursing diagnoses for the plan of care, the nurse will give the highest priority to what nursing diagnosis? A) Activity Intolerance B) Anxiety C) Ineffective Coping D) Acute Pain

Ans: D Feedback: Pain is inevitable during recovery from any burn injury. Pain in the burn patient has been described as one of the most severe causes of acute pain. Management of the often-severe pain is one of the most difficult challenges facing the burn team. While the other nursing diagnoses listed are valid, the presence of pain may contribute to these diagnoses. Management of the patient's pain is the priority, as it may have a direct correlation to the other listed nursing diagnoses.

A nurse is caring for a patient with burns who is in the later stages of the acute phase of recovery. The plan of nursing care should include which of the following nursing actions? A) Maintenance of bed rest to aid healing B) Choosing appropriate splints and functional devices C) Administration of beta adrenergic blockers D) Prevention of venous thromboembolism

Ans: D Feedback: Prevention of deep vein thrombosis (DVT) is an important factor in care. Early mobilization of the patient is important. The nurse monitors the splints and functional devices, but these are selected by occupational and physical therapists. The hemodynamic changes accompanying burns do not normally require the use of beta blockers.

A nurse is performing a home visit to a patient who is recovering following a long course of inpatient treatment for burn injuries. When performing this home visit, the nurse should do which of the following? A) Assess the patient for signs of electrolyte imbalances. B) Administer fluids as ordered. C) Assess the risk for injury recurrence. D) Assess the patient's psychosocial state.

Ans: D Feedback: Recovery from burns can be psychologically challenging; the nurse's assessments must address this reality. Fluid and electrolyte imbalances are infrequent during the rehabilitation phase of recovery. Burns are not typically a health problem that tends to recur; the experience of being burned tends to foster vigilance.

An emergency department nurse has just admitted a patient with a burn. What characteristic of the burn will primarily determine whether the patient experiences a systemic response to this injury? A) The length of time since the burn B) The location of burned skin surfaces C) The source of the burn D) The total body surface area (TBSA) affected by the burn

Ans: D Feedback: Systemic effects are a result of several variables. However, TBSA and wound severity are considered the major factors that affect the presence or absence of systemic effects.

A patient is brought to the ED by paramedics, who report that the patient has partial-thickness burns on the chest and legs. The patient has also suffered smoke inhalation. What is the priority in the care of a patient who has been burned and suffered smoke inhalation? A) Pain B) Fluid balance C) Anxiety and fear D) Airway management

Ans: D Feedback: Systemic threats from a burn are the greatest threat to life. The ABCs of all trauma care apply during the early postburn period. While all options should be addressed, pain, fluid balance, and anxiety and fear do not take precedence over airway management.

While performing a patient's ordered wound care for the treatment of a burn, the patient has made a series of sarcastic remarks to the nurse and criticized her technique. How should the nurse best interpret this patient's behavior? A) The patient may be experiencing an adverse drug reaction that is affecting his cognition and behavior. B) The patient may be experiencing neurologic or psychiatric complications of his injuries. C) The patient may be experiencing inconsistencies in the care that he is being provided. D) The patient may be experiencing anger about his circumstances that he is deflecting toward the nurse.

Ans: D Feedback: The patient may experience feelings of anger. The anger may be directed outward toward those who escaped unharmed or toward those who are now providing care. While drug reactions, complications, and frustrating inconsistencies in care cannot be automatically ruled out, it is not uncommon for anger to be directed at caregivers.

A patient arrives in the emergency department after being burned in a house fire. The patient's burns cover the face and the left forearm. What extent of burns does the patient most likely have? A) 13% B) 25% C) 9% D) 18%

Ans: D Feedback: When estimating the percentage of body area or burn surface area that has been burned, the Rule of Nines is used: the face is 9%, and the forearm is 9% for a total of 18% in this patient.

A patient is treated in the emergency department (ED) for shock of unknown etiology. The first action by the nurse should be to a. check the blood pressure. b. obtain an oxygen saturation. c. attach a cardiac monitor. d. check level of consciousness.

B

A patient who has been involved in a motor-vehicle crash is admitted to the ED with cool, clammy skin, tachycardia, and hypotension. All of these orders are written. Which one will the nurse act on first? a. Insert two 14-gauge IV catheters. b. Administer oxygen at 100% per non-rebreather mask. c. Place the patient on continuous cardiac monitor. d. Draw blood to type and crossmatch for transfusions.

B

A patient who has just been admitted with septic shock has a BP of 70/46, pulse 136, respirations 32, temperature 104.0° F, and blood glucose 246 mg/dl. Which order will the nurse accomplish first? a. Start insulin drip to maintain blood glucose at 110 to 150 mg/dl. b. Give normal saline IV at 500 ml/hr. c. Titrate norepinephrine (Levophed) to keep MAP at 65 to 70 mm Hg. d. Infuse drotrecogin- (Xigris) 24 mcg/kg.

B

A patient who is receiving chemotherapy is admitted to the hospital with acute dehydration caused by nausea and vomiting. Which action will the nurse include in the plan of care to best prevent the development of shock, systemic inflammatory response syndrome (SIRS), and multiorgan dysfunction syndrome (MODS)? a. Administer all medications through the patient's indwelling central line. b. Place the patient in a private room. c. Restrict the patient to foods that have been well-cooked or processed. d. Insert a nasogastric (NG) tube for enteral feeding.

B

The nurse is caring for a patient newly diagnosed with HIV. The patient asks what would determine the actual development of AIDS. The nurse's response is based on the knowledge that what is a diagnostic criterion for AIDS? A Presence of HIV antibodies B CD4+ T cell count below 200/µL C Presence of oral hairy leukoplakia D White blood cell count below 5000/µL

B CD4+ T cell count below 200/µL Diagnostic criteria for AIDS include a CD4+ T cell count below 200/µL and/or the development of specified opportunistic infections, cancers, wasting syndrome, or dementia. The other options may be found in patients with HIV disease but do not define the advancement of HIV infection to AIDS.

The patient has vancomycin-resistant enterococci (VRE) infection in a surgical wound. What infection precautions should the nurse use to best prevent transmission of the infection to the nurse, other patients, staff, and those outside the hospital? A Droplet precautions B Contact precautions C Airborne precautions D Standard precautions

B Contact precautions Correct Contact precautions are used to minimize the spread of pathogens that are acquired from direct or indirect contact. Droplet precautions are used with pathogens that are spread through the air at close contact and that affect the respiratory system or mucous membranes (e.g., influenza, pertussis). Airborne precautions are used if the organism can cause infection over long distances when suspended in the air (e.g., TB, rubeola). Standard precautions are used with all patients and included in the transmission-based precautions above.

A hospital has seen a recent increase in the incidence of hospital care-associated infections (HAIs). Which measure should be prioritized in the response to this trend? A Use of gloves during patient contact B Frequent and thorough hand washing C Prophylactic, broad-spectrum antibiotics D Fitting and appropriate use of N95 masks

B Frequent and thorough hand washing Correct Hand washing remains the mainstay of the prevention of HAIs. Gloves, masks, and antibiotics may be appropriate in specific circumstances, but none of these replaces the central role of vigilant, thorough hand washing between patients and when moving from one task to another, even with the same patient.B Frequent and thorough hand washing Correct

The mother does not want her child to have any extra immunizations for diseases that no longer occur. What teaching about immunization should the nurse provide this mother? A There is currently no need for those older vaccines. B There is a reemergence of some of the infections, such as pertussis. C There is no longer an immunization available for some of those diseases. D The only way to protect your child is to have the federally required vaccines.

B There is a reemergence of some of the infections, such as pertussis. Correct Teaching the mother that some of the diseases are reemerging and the damage they can do to her child gives the mother the information to make an informed decision. The immunizations still exist and do protect individuals.

A 25-year-old male patient has been diagnosed with HIV. The patient does not want to take more than one antiretroviral drug. What reasons can the nurse tell the patient about for taking more than one drug? A Together they will cure HIV. B Viral replication will be inhibited. Correct C They will decrease CD4+ T cell counts. D It will prevent interaction with other drugs.

B Viral replication will be inhibited. Correct The major advantage of using several classes of antiretroviral drugs is that viral replication can be inhibited in several ways, making it more difficult for the virus to recover and decreasing the likelihood of drug resistance that is a major problem with monotherapy. Combination therapy also delays disease progression and decreases HIV symptoms and opportunistic diseases. HIV cannot be cured. CD4+ T cell counts increase with therapy. There are dangerous interactions with many antiretroviral drugs and other commonly used drugs.

The patient at highest risk for venous thromboembolism (VTE) is A. 62-year old man with spider veins who is having arthroscopic knee surgery B. 32-year old woman who smokes, takes oral contraceptives, and is planning a trip to Europe C. 26-year old woman who is 3 days postpartum and received maintenance IV fluids for 12 hours during labor D. An active 72-year old man at home after a TURP procedure for benign prostatic hyperplasia

B. 32-year old woman who smokes, takes oral contraceptives, and is planning a trip to Europe

Which patient should be taught preventive measures for CKD by the nurse because this patient is most likely to develop CKD? A. A 50-year-old white female with hypertension B. A 61-year-old Native American male with diabetes C. A 40-year-old Hispanic female with cardiovascular disease D. A 28-year-old African American female with a urinary tract infection

B. A 61-year-old Native American male with diabetes It is especially important for the nurse to teach CKD prevention to the 61-year-old Native American with diabetes. This patient is at highest risk because diabetes causes about 50% of CKD. This patient is the oldest, and Native Americans with diabetes develop CKD 6 times more frequently than other ethnic groups. Hypertension causes about 25% of CKD. Hispanics have CKD about 1.5 times more than non-Hispanics. African Americans have the highest rate of CKD because hypertension is significantly increased in African Americans. A UTI will not cause CKD unless it is not treated or UTIs occur recurrently.

A patient with a history of end-stage kidney disease secondary to diabetes mellitus has presented to the outpatient dialysis unit for his scheduled hemodialysis. Which assessments should the nurse prioritize before, during, and after his treatment? A. Level of consciousness B. Blood pressure and fluid balance C. Temperature, heart rate, and blood pressure D. Assessment for signs and symptoms of infection

B. Blood pressure and fluid balance Although all of the assessments are relevant to the care of a patient receiving hemodialysis, the nature of the procedure indicates a particular need to monitor the patient's blood pressure and fluid balance.

The first priority of collaborative care of a patient with suspected acute aortic dissection is to A. Reduce anxiety B. Control blood pressure C. Monitor for chest pain D. Increase myocardial contractility

B. Control blood pressure

A 24-year-old female donated a kidney via a laparoscopic donor nephrectomy to a non-related recipient. The patient is experiencing a lot of pain and refuses to get up to walk. How should the nurse handle this situation? A. Have the transplant psychologist convince her to walk. B. Encourage even a short walk to avoid complications of surgery. C. Tell the patient that no other patients have ever refused to walk. D. Tell the patient she is lucky she did not have an open nephrectomy.

B. Encourage even a short walk to avoid complications of surgery. Because ambulating will improve bowel, lung, and kidney function with improved circulation, even a short walk with assistance should be encouraged after pain medication. The transplant psychologist or social worker's role is to determine if the patient is emotionally stable enough to handle donating a kidney, while postoperative care is the nurse's role. Trying to shame the patient into walking by telling her that other patients have not refused and telling the patient she is lucky she did not have an open nephrectomy (implying how much more pain she would be having if it had been open) will not be beneficial to the patient or her postoperative recovery.

Which clinical manifestations are seen in both patients with Beurger's Disease and Reynaud's phenomenon (select all that apply)? A. Intermittent fevers B. Sensitivity to cold temperatures C. Gangrenous ulcers on fibers and toes D. Color changes of fingers and toes E. Episodes of superficial vein thrombosis

B. Sensitivity to cold temperatures C. Gangrenous ulcers on fibers and toes D. Color changes of fingers and toes

Which assessment finding is a consequence of the oliguric phase of AKI? A. Hypovolemia B. Hyperkalemia C. Hypernatremia D. Thrombocytopenia

B. Hyperkalemia In AKI the serum potassium levels increase because the normal ability of the kidneys to excrete potassium is impaired. Sodium levels are typically normal or diminished, whereas fluid volume is normally increased because of decreased urine output. Thrombocytopenia is not a consequence of AKI, although altered platelet function may occur in AKI.

Diffusion, osmosis, and ultrafiltration occur in both hemodialysis and peritoneal dialysis. The nurse should know that ultrafiltration in peritoneal dialysis is achieved by which method? A. Increasing the pressure gradient B. Increasing osmolality of the dialysate C. Decreasing the glucose in the dialysate D. Decreasing the concentration of the dialysate

B. Increasing osmolality of the dialysate Ultrafiltration in peritoneal dialysis is achieved by increasing the osmolality of the dialysate with additional glucose. In hemodialysis the increased pressure gradient from increased pressure in the blood compartment or decreased pressure in the dialysate compartment causes ultrafiltration. Decreasing the concentration of the dialysate in either peritoneal or hemodialysis will decrease the amount of fluid removed from the blood stream.

Patients with a heart transplantation are at risk for which of the following complications in the first year after transplantation (select all that apply)? A. Cancer B. Infection C. Rejection D. Vasculopathy E. Sudden cardiac death

B. Infection C. Rejection E. Sudden cardiac death

Which are probable clinical findings in a person with acute VTE (select all that apply): A. Pallor and coolness of foot and calf B. Mild to moderate calls pain and tenderness C. Grossly diminished or absent pedal pulses D. Unilateral edema and induration of the thigh E. Palpable cord along a superficial varicose vein

B. Mild to moderate calls pain and tenderness D. Unilateral edema and induration of the thigh

A patient is admitted to the hospital with a diagnosis of abdominal aortic aneurysm. Which of the following s/sx would suggest aneurysm rupture? A. Sudden shortness of breath and hemoptysis B. Sudden, severe low back pain and bruising along flank C. Gradually increasing substernal chest pain D. Sudden, patchy blue mottling on feet and toes and rest pain

B. Sudden, severe low back pain and bruising along flank

A patient with hypovolemic shock has a urinary output of 15 ml/hr. The nurse understands that the compensatory physiologic mechanism that leads to altered urinary output is a. activation of the sympathetic nervous system (SNS), causing vasodilation of the renal arteries. b. stimulation of cardiac -adrenergic receptors, leading to increased cardiac output. c. release of aldosterone and antidiuretic hormone (ADH), which cause sodium and water retention. d. movement of interstitial fluid to the intravascular space, increasing renal blood flow.

C

The triage nurse receives a call from a community member who is driving an unconscious friend with multiple injuries after a motorcycle accident to the hospital. The caller states that they will be arriving in 1 minute. In preparation for the patient's arrival, the nurse will obtain a. a liter of lactated Ringer's solution. b. 500 ml of 5% albumin. c. two 14-gauge IV catheters. d. a retention catheter.

C

When caring for a patient with cardiogenic shock and possible MODS, which information obtained by the nurse will help confirm the diagnosis of MODS? a. The patient has crackles throughout both lung fields. b. The patient complains of 8/10 crushing chest pain. c. The patient has an elevated ammonia level and confusion. d. The patient has cool extremities and weak pedal pulses.

C

When the nurse is caring for a patient in cardiogenic shock who is receiving dobutamine (Dobutrex) and nitroglycerin (Tridil) infusions, the best evidence that the medications are effective is that the a. systolic BP increases to greater than 100 mm Hg. b. cardiac monitor shows sinus rhythm at 96 beats/min. c. PAWP drops to normal range. d. troponin and creatine kinase levels decrease.

C

A pregnant woman who was tested and diagnosed with HIV infection is very upset. What should the nurse teach this patient about her baby's risk of being born with HIV infection? A "The baby will probably be infected with HIV." B "Only an abortion will keep your baby from having HIV." C "Treatment with antiretroviral therapy will decrease the baby's chance of HIV infection." D "The duration and frequency of contact with the organism will determine if the baby gets HIV infection."

C "Treatment with antiretroviral therapy will decrease the baby's chance of HIV infection." Correct On average, 25% of infants born to women with untreated HIV will be born with HIV. The risk of transmission is reduced to less than 2% if the infected pregnant woman is treated with antiretroviral therapy. Duration and frequency of contact with the HIV organism is one variable that influences whether transmission of HIV occurs. Volume, virulence, and concentration of the organism as well as host immune status are variables related to transmission via blood, semen, vaginal secretions, or breast milk.

.ID: 809582119 The nurse is providing care for a patient who has been living with HIV for several years. Which assessment finding most clearly indicates an acute exacerbation of the disease? A A new onset of polycythemia B Presence of mononucleosis-like symptoms C A sharp decrease in the patient's CD4+ count D A sudden increase in the patient's WBC count

C A sharp decrease in the patient's CD4+ count Correct A decrease in CD4+ count signals an exacerbation of the severity of HIV. Polycythemia is not characteristic of the course of HIV. A patient's WBC count is very unlikely to suddenly increase, with decreases being typical. Mononucleosis-like symptoms such as malaise, headache, and fatigue are typical of early HIV infection and seroconversion.

The woman is afraid she may get HIV from her bisexual husband. What should the nurse include when teaching her about preexposure prophylaxis (select all that apply)? A Take fluconazole (Diflucan). B Take amphotericin B (Fungizone). C Use condoms for risk-reducing sexual relations. D Take emtricitabine and tenofovir (Truvada) regularly. E Have regular HIV testing for herself and her husband.

C Use condoms for risk-reducing sexual relations. Correct D Take emtricitabine and tenofovir (Truvada) regularly. Correct E Have regular HIV testing for herself and her husband. Correct Using male or female condoms, having monthly HIV testing for the patient and her husband, and the woman taking emtricitabine and tenofovir regularly has shown to decrease the infection of heterosexual women having sex with a partner who participates in high-risk behavior. Fluconazole and amphotericin B are taken for Candida albicans, Coccidioides immitis, and Cryptococcosus neoformans, which are all opportunistic diseases associate with HIV infection.

Priority nursing measures after an abdominal aortic aneurysm repair include A. Assessment of cranial nerves and mental status B. Administration of IV heparin and aPTT monitoring C. Administration of IV fluids and monitoring kidney function D. Elevation of legs and application of graduated compression stockings

C. Administration of IV fluids and monitoring kidney function

A 50-year old woman weighs 85kg and has a history of cigarette smoking, high blood pressure, high sodium intake, and sedentary lifestyle. When developing a plan of care for her, the nurse determines the most important risk factors for peripheral artery disease that need to be modified are: A. Weight and diet B. Activity level and diet C. Cigarette smoking and high blood pressure D. Sedentary lifestyle and high blood pressure

C. Cigarette smoking and high blood pressure

The patient sustained a full-thickness burn encompassing the entire right arm. What is the best indicator an escharotomy achieved its desired effect? A. Patient rates the pain at less than 4. B. Blood pressure remains above 120/80 mm Hg. C. Right fingers blanch with a 2-second refill. D. Patient maintains full range of motion for the right arm.

C. Circulation to the extremities can be severely impaired by deep circumferential burns and subsequent edema that impairs the blood supply. An escharotomy (electrocautery incision through the full-thickness eschar) is performed to restore circulation. Normal refill is less than 2 seconds.

The physician has decided to use renal replacement therapy to remove large volumes of fluid from a patient who is hemodynamically unstable in the intensive care unit. The nurse should expect which treatment to be used for this patient? A. Hemodialysis (HD) 3 times per week B. Automated peritoneal dialysis (APD) C. Continuous venovenous hemofiltration (CVVH) D. Continuous ambulatory peritoneal dialysis (CAPD)

C. Continuous venovenous hemofiltration (CVVH) CVVH removes large volumes of water and solutes from the patient over a longer period of time by using ultrafiltration and convection. HD 3 times per week would not be used for this patient because fluid and solutes build up and then are rapidly removed. With APD (used at night instead of during the day) fluid and solutes build up during the day and would not benefit this patient as much. CAPD will not as rapidly remove large amounts of fluid as CVVH can do.

Rest pain is a manifestation of PAD that occurs due to a chronic A. Vasospasm of small cutaneous arteries in the get B. Increase in retrograde venous blood flow in the legs C. Decease in arterial blood flow to the nerves of the feet D. Decrease in arterial blood flow to the leg muscles during exercise

C. Decrease in arterial blood flow to the nerves of the feet

During hemodialysis, the patient develops light-headedness and nausea. What should the nurse do for the patient? A. Administer hypertonic saline. B. Administer a blood transfusion. C. Decrease the rate of fluid removal. D. Administer antiemetic medications.

C. Decrease the rate of fluid removal. The patient is experiencing hypotension from a rapid removal of vascular volume. The rate and volume of fluid removal will be decreased, and 0.9% saline solution may be infused. Hypertonic saline is not used because of the high sodium load. A blood transfusion is not indicated. Antiemetic medications may help the nausea but would not help the hypovolemia.

The nurse is caring for a patient with chronic constrictive pericarditis. Which assessment finding reflects a more serious complication of the condition? A. Fatigue B. Peripheral edema C. Jugular vein distension D. Thickened pericardium on echocardiography

C. Jugular vein distension

A patient is recovering in the intensive care unit (ICU) after receiving a kidney transplant approximately 24 hours ago. What is an expected assessment finding for this patient during this early stage of recovery? A. Hypokalemia B. Hyponatremia C. Large urine output D. Leukocytosis with cloudy urine output

C. Large urine output Patients frequently experience diuresis in the hours and days immediately following a kidney transplant. Electrolyte imbalances and signs of infection are unexpected findings that warrant prompt intervention.

The nurse is caring for a patient newly admitted with heart failure secondary to dilated cardiomyopathy. Which is a priority intervention? A. Encourage caregivers to learn CPR B. Consider a consultation for palliative care C. Monitor the patients response to prescribed medications D. Arrange for the patient to enter a cardiac rehabilitation program

C. Monitor the patients response to prescribed medications

A compensatory mechanism involved in HF that leads to inappropriate fluid retention and additional workload of the heart is A. Ventricular dilation B. Ventricular hypertrophy C. Neurohormonal response D. SNS activation

C. Neurohormonal response

A patient with infective edocarditis develops a sudden left leg pain with pallor, paresthesia, and a loss of peripheral pulses. The nurse's initial action should be to A. Elevate the leg to promote venous circulation B. Start anticoagulation therapy with IV heparin C. Notify the physician of the change in peripheral perfusion D. Place in reverse Trendelenberg to promote perfusion

C. Notify the physician of the change in peripheral perfusion

One of the nurse's most important roles in relation to acute poststreptococcal glomerulonephritis is to a. promote early diagnosis and treatment of sore throats and skin lesions. b. encourage patients to obtain antibiotic therapy for upper respiratory tract infections. c. teach patients with APSGN that long-term prophylactic antibiotic therapy is necessary to prevent recurrence. d. monitor patients for respiratory symptoms that indicate the disease is affecting the alveolar basement membrane.

Correct answer: a Rationale: Acute poststreptococcal glomerulonephritis (APSGN) develops 5 to 21 days after an infection of the tonsils, pharynx, or skin (e.g., streptococcal sore throat, impetigo) by nephrotoxic strains of group A β-hemolytic streptococci. The most important ways to prevent the development of APSGN are early diagnosis and treatment of sore throats and skin lesions.

A patient is admitted to the hospital with severe renal colic. The nurse's first priority in management of the patient is to a. administer opioids as prescribed. b. obtain supplies for straining all urine. c. encourage fluid intake of 3 to 4 L/day. d. keep the patient NPO in preparation for surgery.

Correct answer: a Rationale: Pain management and patient comfort are primary nursing responsibilities in managing an obstructing stone and renal colic.

In planning nursing interventions to increase bladder control in the patient with urinary incontinence, the nurse includes a. teaching the patient to use Kegel exercises. b. clamping and releasing a catheter to increase bladder tone. c. teaching the patient biofeedback mechanisms to suppress the urge to void. d. counseling the patient concerning choice of incontinence containment device.

Correct answer: a Rationale: Pelvic floor muscle training (i.e., Kegel exercises) is used to manage stress, urge, or mixed urinary incontinence.

The immunologic mechanisms involved in acute poststreptococcal glomerulonephritis include a. tubular blocking by precipitates of bacteria and antibody reactions. b. deposition of immune complexes and complement along the GBM. c. thickening of the GBM from autoimmune microangiopathic changes. d. destruction of glomeruli by proteolytic enzymes contained in the GBM.

Correct answer: b Rationale: All forms of immune complex disease are characterized by an accumulation of antigen, antibody, and complement in the glomeruli, which can result in tissue injury. The immune complexes activate complement. Complement activation results in the release of chemotactic factors that attract polymorphonuclear leukocytes, histamine, and other inflammatory mediators. The result of these processes is glomerular injury.

The nurse identifies a risk factor for kidney and bladder cancer in a patient who relates a history of a. aspirin use. b. tobacco use. c. chronic alcohol abuse. d. use of artificial sweeteners.

Correct answer: b Rationale: Cigarette smoking is the most significant risk factor for renal cell carcinoma. An increased incidence has also been identified in first-degree relatives of patients with this condition. Other risk factors include obesity, hypertension, and exposure to asbestos, cadmium, and gasoline. Risk for renal cancer is also increased in individuals who have acquired cystic disease of the kidney in association with end-stage renal disease. Risk factors for bladder cancer include smoking, exposure to dyes used in the rubber and cable industries, and chronic abuse of phenacetin-containing analgesics.

A patient with a ureterolithotomy returns from surgery with a nephrostomy tube in place. Postoperative nursing care of the patient includes a. encouraging the patient to drink fruit juices and milk. b. encouraging fluids of at least 2 to 3 L/day after nausea has subsided. c. irrigating the nephrostomy tube with 10 mL of normal saline solution as needed. d. notifying the physician if nephrostomy tube drainage is more than 30 mL/hr.

Correct answer: b Rationale: The nephrostomy tube is inserted directly into the renal pelvis and attached to connecting tubing for closed drainage. The catheter should never be kinked, compressed, or clamped. If the patient complains of excessive pain in the area, or if drainage around the tube is excessive, check the catheter for patency. If irrigation is ordered, strict aseptic technique is required. To prevent overdistention of the renal pelvis and renal damage, no more than 5 mL of sterile saline solution is gently instilled at one time. Infection and secondary stone formation are complications associated with the insertion of a nephrostomy tube. Patients should drink 2 to 3 L of fluid per day to reduce risk of infection and stone formation

The nurse teaches the female patient who has frequent UTIs that she should a. take tub baths with bubble bath. b. urinate before and after sexual intercourse. c. take prophylactic sulfonamides for the rest of her life. d. restrict fluid intake to prevent the need for frequent voiding

Correct answer: b Rationale: When teaching a patient to prevent a recurrence of a urinary tract infection, the nurse should explain the importance of emptying the bladder before and after sexual intercourse.

A patient has had a cystectomy and ileal conduit diversion performed. Four days postoperatively, mucous shreds are seen in the drainage bag. The nurse should a. notify the physician. b. notify the charge nurse. c. irrigate the drainage tube. d. chart it as a normal observation.

Correct answer: d Rationale: Patients with an ileal conduit have mucus in the urine. The mucus is secreted by intestinal mucosa, which is used to create the ileal conduit, in response to the irritating effect of urine.

The nurse recommends genetic counseling for the children of a patient with a. nephrotic syndrome. b. chronic pyelonephritis. c. malignant nephrosclerosis. d. adult-onset polycystic kidney disease.

Correct answer: d Rationale: The adult form of polycystic kidney disease (PKD) is an autosomal dominant disorder. If one parent has the disease, the child has a 50% chance of developing PKD. Many patients who have adult PKD have had children by the time the disease is diagnosed. Patients need appropriate counseling regarding plans for having more children, and genetic counseling resources should be provided for the children.

The edema that occurs in nephrotic syndrome is due to a. increased hydrostatic pressure caused by sodium retention. b. decreased aldosterone secretion from adrenal insufficiency. c. increased fluid retention caused by decreased glomerular filtration. d. decreased colloidal osmotic pressure caused by loss of serum albumin.

Correct answer: d Rationale: The increased permeability of the glomerular membrane found in nephrotic syndrome is responsible for the massive excretion of protein in the urine. This results in decreased serum protein levels and subsequent edema formation. Ascites and anasarca (i.e., massive generalized edema) develop if hypoalbuminemia is severe.

In teaching a patient with pyelonephritis about the disorder, the nurse informs the patient that the organisms that cause pyelonephritis most commonly reach the kidneys through a. the bloodstream. b. the lymphatic system. c. a descending infection. d. an ascending infection.

Correct answer: d Rationale: The organisms that usually cause urinary tract infections (UTIs) are introduced via the ascending route from the urethra, and the infections originate in the perineum.

A diabetic patient who has had vomiting and diarrhea for the past 3 days is admitted to the hospital with a blood glucose of 748 mg/ml (41.5 mmol/L) and a urinary output of 120 ml in the first hour. The vital signs are blood pressure (BP) 72/62; pulse 128, irregular and thready; respirations 38; and temperature 97° F (36.1° C). The patient is disoriented and lethargic with cold, clammy skin and cyanosis in the hands and feet. The nurse recognizes that the patient is experiencing the a. progressive stage of septic shock. b. compensatory stage of diabetic shock. c. refractory stage of cardiogenic shock. d. progressive stage of hypovolemic shock.

D

When caring for a critically ill patient who is being mechanically ventilated, the nurse will astutely monitor for which of the following clinical manifestations of multiple organ dysfunction syndrome (MODS)? A) Increased gastrointestinal (GI) motility B) Increased serum albumin C) Decreased blood urea nitrogen (BUN)/creatinine ratio D) Decreased respiratory compliance

D Clinical manifestations of MODS include symptoms of respiratory distress, signs and symptoms of decreased renal perfusion, decreased serum albumin and prealbumin, decreased GI motility, acute neurologic changes, myocardial dysfunction, disseminated intravascular coagulation (DIC), and changes in glucose metabolism.

A massive gastrointestinal bleed has resulted in hypovolemic shock in an elderly patient. Which of the following is a priority nursing diagnosis? A) Acute pain B) Impaired tissue integrity C) Decreased cardiac output D) Ineffective tissue perfusion

D The many deleterious effects of shock are all related to inadequate perfusion and oxygenation of every body system. This nursing diagnosis supersedes the other diagnoses.

Multiple patients arrive in the emergency department from a house fire. Which patient is a priority? A. Patient with erythremic, dry burns over the arms and a history of taking prednisone B. Patient with moist blisters over the chest and who reports pain as 10 C. Patient with dry, black skin on one hand and a history of diabetes mellitus D. Patient with multiple reddened skin areas on the chest and with high-pitched respiratory sounds

D. Airway injury is a priority, and stridor results from a narrowing of the airway caused by edema. A history of prednisone use or diabetes is a concern for long-term infection risk, but the airway is always first.

A drug used in the management of a patient with ADHF and pulmonary edema that will decrease both preload and afterload and provide relief of anxiety is A. Amrinone B. Furosemide C. Dobutamine D. Morphine sulfate

D. Morphine sulfate

In planning care and patient teaching for the patient with venous leg ulcers, the nurse recognizes that the most important intervention in healing and control of this condition is A. Sclerotherapy B. Taking chestnut extract daily C. Using moist environment dressings D. Apply graduated compression stockings

D. Apply graduated compression stockings

The patient has had type 1 diabetes mellitus for 25 years and is now reporting fatigue, edema, and an irregular heartbeat. On assessment, the nurse finds that the patient has newly developed hypertension and difficulty with blood glucose control. The nurse should know that which diagnostic study will be most indicative of chronic kidney disease (CKD) in this patient? A. Serum creatinine B. Serum potassium C. Microalbuminuria D. Calculated glomerular filtration rate (GFR)

D. Calculated glomerular filtration rate (GFR) The best study to determine kidney function or chronic kidney disease (CKD) that would be expected in the patient with diabetes is the calculated GFR that is obtained from the patient's age, gender, race, and serum creatinine. It would need to be abnormal for 3 months to establish a diagnosis of CKD. A creatinine clearance test done with a blood sample and a 24-hour urine collection is also important. Serum creatinine is not the best test for CKD because the level varies with different patients. Serum potassium levels could explain why the patient has an irregular heartbeat. The finding of microalbuminuria can alert the patient with diabetes about potential renal involvement and potentially failing kidneys. However, urine albumin levels are not used for diagnosis of CKD.

The patient was diagnosed with prerenal AKI. The nurse should know that what is most likely the cause of the patient's diagnosis? A. IV tobramycin (Nebcin) B. Incompatible blood transfusion C. Poststreptococcal glomerulonephritis D. Dissecting abdominal aortic aneurysm

D. Dissecting abdominal aortic aneurysm A dissecting abdominal aortic aneurysm is a prerenal cause of AKI because it can decrease renal artery perfusion and therefore the glomerular filtrate rate. Aminoglycoside antibiotic administration, a hemolytic blood transfusion reaction, and poststretpcoccal glomerulonephritis are intrarenal causes of AKI.

Priority nursing management for a patient with myocarditis includes interventions related to: A. Meticulous skin care B. Antibiotic prophylaxis C. Tight glycemic control D. Oxygenation and ventilation

D. Oxygenation and ventilation

The nurse preparing to administer a dose of calcium acetate (PhosLo) to a patient with chronic kidney disease (CKD) should know that this medication should have a beneficial effect on which laboratory value? A. Sodium B. Potassium C. Magnesium D. Phosphorus

D. Phosphorus Phosphorus and calcium have inverse or reciprocal relationships, meaning that when phosphorus levels are high, calcium levels tend to be low. Therefore administration of calcium should help to reduce a patient's abnormally high phosphorus level, as seen with CKD. PhosLo will not have an effect on sodium, potassium, or magnesium levels.

When caring for a patient during the oliguric phase of acute kidney injury (AKI), what is an appropriate nursing intervention? A. Weigh patient three times weekly. B. Increase dietary sodium and potassium. C. Provide a low-protein, high-carbohydrate diet. D. Restrict fluids according to previous daily loss.

D. Restrict fluids according to previous daily loss. Patients in the oliguric phase of acute kidney injury will have fluid volume excess with potassium and sodium retention. Therefore they will need to have dietary sodium, potassium, and fluids restricted. Daily fluid intake is based on the previous 24-hour fluid loss (measured output plus 600 ml for insensible loss). The diet also needs to provide adequate, not low, protein intake to prevent catabolism. The patient should also be weighed daily, not just three times each week

Which clinical finding would most likely indicate decreased cardiac output in a patient with aortic valve regurgitation? A. Reduction in peripheral edema and weight B. Carotid venous distension and new onset atrial fibrillation C. Significant pulsus paradoxus and diminished peripheral pulses D. Shortness of breath on minimal exertion and diastolic murmur

D. Shortness of breath on minimal exertion and diastolic murmur

The recommended treatment for an initial VTE in an otherwise healthy person with no significant co-morbidities include: A. IV agratoban (Acova) as an inpatient B. IV unfractioned heparin add an inpatient C. Subcutaneous unfractioned heparin add an outpatient D. Subcutaneous LMWH as an outpatient

D. Subcutaneous LMWH as an outpatient

How should the nurse explain the importance of follow-up cystoscopies?

Follow-up cystoscopies on a regular basis are essential to evaluate the effectiveness of the treatment and detect any new tumors while they are in a superficial stage. Two thirds of patients have tumor recurrence within 5 years and nearly 95% have recurrence by 15 years.

Priority Decision: Based on the assessment data presented, what are the priority nursing diagnoses? Are there any collaborative problems? Patient Profile: Male patient, married, 55-year-old mechanic who has been healthy all of his life until he passed some blood in his urine and saw a urologist at his wife's insistence. A urine specimen for cytology revealed atypical cells, and a diagnosis of bladder cancer was made following a cystoscopy with biopsy of bladder tissue. The tumor was removed with a transurethral resection and laser cauterization. Intravesical therapy with bacille Calmette-Guérin (BCG), a weak- ened strain of Mycobacterium bovis, is planned. Subjective Data • Has smoked a pack of cigarettes a day since he was a teenager • Says he dreads having the chemotherapy because he has heard cancer drugs cause severe side effects Objective Data • Cystoscopy and biopsy results: Moderately differentiated stage II tumor on the left lateral bladder wall, with T1N0M0 pathologic stage • Continues to have gross hematuria

Nursing diagnoses: • Anxiety related to unknown outcome • Impaired urinary elimination related to effects of treatment • Acute pain related to effects of treatment • Risk for infection related to effects of treatment • Risk for sexual dysfunction related to reactions to treatment Collaborative problems: Potential complication: bladder injury, impaired skin integrity

Post-op care for transurethral resection and laser cauterization?

Opioids and stool softeners may be used for a short time after the procedure. Patient will be taught to drink a large volume of fluid and monitor the color and consistency of the urine. The urine will be pink at first but should not be red or contain clots. Seven to 10 days after the tumor resection Patient may observe dark red or rust-colored flecks in the urine from the healing tumor resection sites.

How can patient help to prevent future bladder tumors from occurring?

Patient should stop smoking; it is the only significant risk factor in his history. Maintenance therapy after the initial induction regimen may be beneficial, as there is a high rate of disease recurrence.

What information and instructions would the nurse provide about intravesical therapy?

The drug will be instilled into the empty bladder via a urethral catheter at weekly intervals for 6 to 12 weeks. The drug needs to be retained for about 2 hours with patient's position changed about every 15 minutes to ensure that the drug comes into maximum contact with all areas of the bladder. He may have irritative symptoms, such as frequency, urgency, and bladder spasms, in addition to hematuria during the weeks of treatment. Bacille Calmette-Guérin (BCG) therapy may cause flu-like symptoms or systemic infection because BCG stimulates the immune system rather than directly destroying cancer cells. The usual side effects of cancer chemotherapy are not experienced with BCG therapy or with intravesical chemotherapy.

What is the nurse's priority when changing the appliance of a patient with an ileal conduit? a. Keep the skin free of urine. b. Inspect the peristomal area. c. Cleanse and dry the area gently. d. Affix the appliance to the faceplate

a. Keep the skin free of urine. The nurse's priority is to keep the skin free of urine because the peristomal skin is at high risk for damage from the urine if it is alkaline. The peristomal area will be assessed; the area will be gently cleaned and dried, and the appliance will be affixed to the faceplate if one is being used, but these are not as much of a priority as keeping the skin free of urine to prevent skin damage.

Eight months after the delivery of her first child, a 31-year-old woman has sought care because of occasional incontinence that she experiences when sneezing or laughing. Which measure should the nurse first recommend in an attempt to resolve the woman's incontinence? a. Kegel exercises b. Use of adult incontinence pads c. Intermittent self-catheterization d. Dietary changes including fluid restriction

a. Kegel exercises Patients who experience stress incontinence frequently benefit from Kegel exercises (pelvic floor muscle exercises). The use of incontinence pads does not resolve the problem, and intermittent self-catheterization would be a premature recommendation. Dietary changes are not likely to influence the patient's urinary continence.

A client's burn is infected and mafenide (Sulfamylon) is prescribed. The nurse's knowledge about this medication would indicate that which organism is involved? a. pseudomonas aeruginosa b. tubercle bacillus c. Methicillin resistant staphylococcus aureus (MRSA) d. Candida albicans

a. Mafenide is useful in treatment of partial and full thickness burns to prevent septicemia caused by organisms suche as pseudomonas aeruginosa.

The nurse plans care for the patient with APSGN based on what knowledge? a. Most patients with APSGN recover completely or rapidly improve with conservative management. b. Chronic glomerulonephritis leading to renal failure is a common sequela to acute glomerulonephritis. c. Pulmonary hemorrhage may occur as a result of antibodies also attacking the alveolar basement membrane. d. A large percentage of patients with APSGN develop rapidly progressive glomerulonephritis, resulting in kidney failure.

a. Most patients recover completely from acute poststreptococcal glomerulonephritis (APSGN) with supportive treatment. Chronic glomerulonephritis that progresses insidiously over years and rapidly progressive glomerulonephritis that results in renal failure within weeks or months occur in only a few patients with APSGN. In Goodpasture syndrome, antibodies are present against both the GBM and the alveolar basement membrane of the lungs and dysfunction of both renal and pulmonary are present.

The urinalysis of a male patient reveals a high microorganism count. What data should the nurse use to determine the area of the urinary tract that is infected (select all that apply)? a. Pain location Correct b. Fever and chills Incorrect c. Mental confusion Incorrect d. Urinary hesitancy Incorrect e. Urethral discharge Correct f. Post-void dribbling

a. Pain location e. Urethral discharge Although all the manifestations are evident with urinary tract infections (UTIs), pain location is useful in differentiating among pyelonephritis, cystitis, and urethritis because flank pain is characteristic of pyelonephritis, but dysuria is characteristic of cystitis and urethritis. Urethral discharge is indicative of urethritis, not pyelonephritis or cystitis. Fever and chills and mental confusion are nonspecific indicators of UTIs. Urinary hesitancy and postvoid dribbling may occur with a UTI but may also occur with prostate enlargement in the male patient.

The patient has a thoracic spinal cord lesion and incontinence that occurs equally during the day and night. What type of incontinence is this patient experiencing? a. Reflex incontinence b. Overflow incontinence c. Functional incontinence d. Incontinence after trauma

a. Reflex incontinence occurs with no warning, equally during the day and night, and with spinal cord lesions above S2. Overflow incontinence is when the pressure of urine in the overfull bladder overcomes sphincter control and is caused by bladder or urethral outlet obstruction. Functional incontinence is loss of urine resulting from cognitive, functional, or environmental factors. Incontinence after trauma or surgery occurs when fistulas have occurred or after a prostatectomy.

"Which characteristic is more likely with acute pyelonephritis than with a lower UTI? a. Fever b. Dysuria c. Urgency d. Frequency

a. Systemic manifestations of fever and chills with leukocytosis and nausea and vomiting are more common in pyelonephritis than in a lower UTI. Dysuria, frequency, and urgency can be present with both.

Which urinary diversion is a continent diversion created by formation of an ileal pouch with a stoma for catheterization? a. Kock pouch b. Ileal conduit c. Orthotopic neobladder d. Cutaneous ureterostomy

a. The Kock pouch is a continent diversion created by formation of an ileal pouch with an external stoma requiring catheterization. Ileal conduit is the most common incontinent diversion using a stoma of resected ileum with implanted ureters. Orthotopic neobladder is a new bladder from a reshaped segment of intestine in the anatomic position of the bladder with urine discharged through the urethra. A cutaneous ureterostomy diverts the ureter from the bladder to the abdominal skin but there is frequent scarring and strictures of the ureters, so ileal conduits are used more often.

A male burn pt who was struck by lightning arrives at the emergency department with full thickness burns to the arms and chest and with a cervical collar in place. Which assessment finding is the nurse's priority? a. serum K+ of 5.6 mEq/L b. Arterial blood ph of 7.35 c. Cervical spine fracture d. hemoglobin 18g/dL

a. The pt's potassium level puts them at risk for life threatening cardiac dysrhythmias.

The nurse counsels a 64-year-old man on dietary restrictions to prevent recurrent uric acid renal calculi. Which foods should the nurse teach the patient to avoid? a. Venison, crab, and liver b. Spinach, cabbage, and tea c. Milk, yogurt, and dried fruit Incorrect d. Asparagus, lentils, and chocolate

a. Venison, crab, and liver Foods high in purines (e.g., venison, crab, liver) should be avoided to prevent uric acid calculi formation. Foods high in calcium (e.g., milk, yogurt, dried fruit, lentils, chocolate) should be avoided to prevent calcium calculi formation. Foods high in oxalate (e.g., spinach, cabbage, tea, asparagus, chocolate) should be avoided to prevent oxalate calculi formation

A child was admitted to the ED with a thermal burn to the right arm and leg. Which assessment by the nurse requires immediate action? a. coughing and wheezing b. bright red skin with small blister on the burn sites c. thirst d. singed hair

a. coughing and wheezing may indicate that the child has inhaled smoke or toxic fumes. Maintaining airway patency is the highest nursing priority in this situation.

The nurse gives instructions to a 62-year-old woman taking tolterodine (Detrol) to treat urinary urgency and incontinence. The nurse should include which statement? a. "Stop smoking for 2 to 3 weeks before starting to take this medication." b. "Suck on sugarless candy or chew sugarless gum if you develop a dry mouth." c. "Have your vision checked every 6 months because this drug can cause cataracts." d. "Ask your physician to prescribe an extended-release form if you have loose stools."

b. "Suck on sugarless candy or chew sugarless gum if you develop a dry mouth." Dry mouth is a common side effect of tolterodine. Patients can suck on hard candy or ice chips or chew gum if dry mouth occurs. Tobacco use does not affect the initiation of this medication. Visual changes (but not cataracts) can occur while taking this medication. Constipation may occur as a side effect of this medication.

The nurse is caring for a client who suffered an inhalation injury from a wood stove. The carbon monoxide blood report reveals a level of 12%. Based on this level, the nurse would anticipate which of the following signs in the client? a. coma b. flushing c. dizziness d. tachycardia

b. 11-20% - signs include flushing, headache, decreased visual acuity, decreased cerebral functioning, and slight breathlessness. 21-40% - signs include nausea, vomiting, dizziness, tinnitus, vertigo, confusion, drowsiness, pale to reddish-purple skin, tachycardia; levels of 41-60% result in seizure and come and levels higher than 60% result in death

Priority Decision: Following electrohydraulic lithotripsy for treatment of renal calculi, the patient has a nursing diagnosis of risk for infection related to the introduction of bacteria following manipulation of the urinary tract. What is the most appropriate nursing intervention for this patient? a. Monitor for hematuria. b. Encourage fluid intake of 3 L/day. c. Apply moist heat to the flank area. d. Strain all urine through gauze or a special strainer.

b. A high fluid intake maintains dilute urine, which decreases bacterial concentration in addition to washing stone fragments and expected blood through the urinary system following lithotripsy. High urine output also prevents supersaturation of minerals. Moist heat to the flank may be helpful to relieve muscle spasms during renal colic and all urine should be strained in patients with renal stones to collect and identify stone composition but these are not related to infection.

During assessment of the patient who has a nephrectomy, what should the nurse expect to find? a. Shallow, slow respirations b. Clear breath sounds in all lung fields c. Decreased breath sounds in the lower left lobe d. Decreased breath sounds in the right and left lower lobes

b. A nephrectomy incision is usually in the flank, just below the diaphragm or in the abdominal area. Although the patient is reluctant to breathe deeply because of incisional pain, the lungs should be clear. Decreased sounds and shallow respirations are abnormal and would require intervention.

In providing care for the patient with adult-onset polycystic kidney disease, what should the nurse do? a. Help the patient to cope with the rapid progression of the disease b. Suggest genetic counseling resources for the children of the patient c. Expect the patient to have polyuria and poor concentration ability of the kidneys d. Implement measures for the patient's deafness and blindness in addition to the renal problems

b. Adult-onset polycystic kidney disease is an inherited autosomal dominant disorder that often manifests after the patient has children but the children should receive genetic counseling regarding their life choices. The disease progresses slowly, eventually causing progressive renal failure. Hereditary medullary cystic disease causes poor concentration ability of the kidneys and classic Alport syndrome is a hereditary nephritis that is associated with deafness and deformities of the optic lens.

A teaching plan developed by the nurse for the patient with a new ileal conduit includes instructions to do what? a. Clean the skin around the stoma with alcohol every day. b. Use a wick to keep the skin dry during appliance changes. c. Use sterile supplies and technique during care of the stoma. d. Change the appliance every day and wash it with soap and warm water.

b. Because the stoma continuously drains urine, a wick formed of a rolled-up 4 × 4 gauze or a tampon is held against the stoma to absorb the urine while the skin is cleaned and a new appliance is attached. The skin is cleaned with warm water only because soap and other agents cause drying and irritation and clean, not sterile, technique is used. The appliance should be left in place for as long as possible before it loosens and allows leakage onto the skin, perhaps up to 14 days.

A nurse is admitting a patient with the diagnosis of advanced renal carcinoma. Based upon this diagnosis, the nurse will expect to find what clinical manifestations as the "classic triad" occurring in patients with renal cancer? a. Fever, chills, flank pain b. Hematuria, flank pain, palpable mass c. Hematuria, proteinuria, palpable mass d. Flank pain, palpable abdominal mass, and proteinuria

b. Hematuria, flank pain, palpable mass There are no characteristic early symptoms of renal carcinoma. The classic manifestations of gross hematuria, flank pain, and a palpable mass are those of advanced disease.

The nurse provides nutritional counseling for a 45-year-old man with nephrotic syndrome. The nurse determines that teaching has been successful if the patient selects which breakfast menu? a. Scrambled eggs, milk, yogurt, and sliced ham b. Oatmeal, nondairy creamer, banana, and orange juice c. Cottage cheese, peanut butter, white bread, and coffee d. Waffle, bacon strips, tomato juice, and canned peaches

b. Oatmeal, nondairy creamer, banana, and orange juice Patients with nephrotic syndrome should follow a low-sodium (2 to 3 g/day), low- to moderate-protein (0.5 to 0.6 g/kg/day) diet. Ham, milk products, peanut butter, and bacon are high in sodium. Eggs, milk products, and peanut butter are high in protein.

A patient has a right ureteral catheter placed following a lithotripsy for a stone in the ureter. In caring for the patient after the procedure, what is an appropriate nursing action? a. Milk or strip the catheter every 2 hours. b. Measure ureteral urinary drainage every 1 to 2 hours. c. Irrigate the catheter with 30-mL sterile saline every 4 hours. d. Encourage ambulation to promote urinary peristaltic action.

b. Output from ureteral catheters must be monitored every 1 to 2 hours because an obstruction will cause overdistention of the renal pelvis and renal damage. The renal pelvis has a capacity of only 3 to 5 mL and if irrigation is ordered, no more than 5 mL of sterile saline is used. The patient with a ureteral catheter is usually kept on bed rest until specific orders for ambulation are given. Suprapubic tubes may be milked to prevent obstruction of the catheter by sediment and clots.

A 22-year-old patient's blood pressure at her physical done for her new job was 110/68. At the health fair two months later, her blood pressure is 154/96. What renal problem should the nurse be aware of that could contribute to this abrupt rise in blood pressure? a. Renal trauma b. Renal artery stenosis c. Renal vein thrombosis d. Benign nephrosclerosis

b. Renal artery stenosis Renal artery stenosis contributes to an abrupt rise in blood pressure, especially in people under 30 or over 50 years of age. Renal trauma usually has hematuria. Renal vein thrombosis causes flank pain, hematuria, fever, or nephrotic syndrome. Benign nephrosclerosis usually occurs in adults 30 to 50 years of age and is a result of vascular changes resulting from hypertension.

The female patient with a UTI also has renal calculi. The nurse knows that these are most likely which type of stone? a. Cystine b. Struvite c. Uric acid d. Calcium phosphate

b. Struvite calculi are most common in women and always occur with UTIs. They are also usually large staghorn type.

Which nursing intervention is most appropriate in providing care for an adult patient with newly diagnosed adult onset polycystic kidney disease (PKD)? a. Help the patient cope with the rapid progression of the disease. b. Suggest genetic counseling resources for the children of the patient. c. Expect the patient to have polyuria and poor concentration ability of the kidneys. d. Implement appropriate measures for the patient's deafness and blindness in addition to the renal problems.

b. Suggest genetic counseling resources for the children of the patient. PKD is one of the most common genetic diseases. The adult form of PKD may range from a relatively mild disease to one that progresses to chronic kidney disease. Polyuria, deafness, and blindness are not associated with PKD.

A female patient with a UTI has a nursing diagnosis of risk for infection related to lack of knowledge regarding prevention of recurrence. What should the nurse include in the teaching plan instructions for this patient? a. Empty the bladder at least 4 times a day. b. Drink at least 2 quarts of water every day. c. Wait to urinate until the urge is very intense. d. Clean the urinary meatus with an antiinfective agent after voiding

b. The bladder should be emptied at least every 3 to 4 hours. Fluid intake should be increased to about 2000 mL/ day without caffeine, alcohol, citrus juices, and chocolate drinks, because they are potential bladder irritants. Cleaning the urinary meatus with an antiinfective agent after voiding will irritate the meatus but the perineal area should be wiped from front to back after urination and defecation to prevent fecal contamination of the meatus.

The nurse is caring for a patient with superficial partial-thickness burns of the face sustained within the last 12 hours. Upon assessment, the nurse would expect to find which of the following symptoms? A. Blisters B. Reddening of the skin C. Destruction of all skin layers D. Damage to sebaceous glands

b. The clinical appearance of superficial partial-thickness burns includes erythema, blanching with pressure, and pain and minimal swelling with no vesicles or blistering during the first 24 hours.

Which infection is asymptomatic in the male patient at first and then progresses to cystitis, frequent urination, burning on voiding, and epididymitis? a. Urosepsis b. Renal tuberculosis c. Urethral diverticula d. Goodpasture syndrome

b. The manifestations of renal tuberculosis are described. Urosepsis is when the UTI has spread systemically. Urethral diverticula are localized outpouching of the urethra and occur more often in women. Goodpasture syndrome manifests with flu-like symptoms with pulmonary symptoms that include cough, shortness of breath, and pulmonary insufficiency and renal manifestations that include hematuria, weakness, pallor, anemia, and renal failure.

With which diagnosis will the patient benefit from being taught to do self-catheterization? a. Renal trauma b. Urethral stricture c. Renal artery stenosis d. Accelerated nephrosclerosis

b. The patient with urethral stricture will benefit from being taught to dilate the urethra by self-catheterization every few days. Renal trauma is treated related to the severity of the injury with bed rest, fluids, and analgesia. Renal artery stenosis includes control of hypertension with possible surgical revascularization. Accelerated nephrosclerosis is associated with malignant hypertension that must be aggressively treated as well as monitoring kidney function.

The nurse is caring for a patient with a nephrostomy tube. The tube has stopped draining. After receiving orders, what should the nurse do? a. Keep the patient on bed rest. b. Use 5 mL of sterile saline to irrigate. c. Use 30 mL of water to gently irrigate. d. Have the patient turn from side to side.

b. Use 5 mL of sterile saline to irrigate With a nephrostomy tube, if the tube is occluded and irrigation is ordered, the nurse should use 5 mL or less of sterile saline to gently irrigate it. The patient with a ureteral catheter may be kept on bed rest after insertion, but this is unrelated to obstruction. Only sterile solutions are used to irrigate any type of urinary catheter. With a suprapubic catheter, the patient should be instructed to turn from side to side to ensure patency.

The nurse should expect to apply which type of ordered antiseptic to a client with a burn wound, once the area has been cleansed with sterile saline? a. copper containing b. silver containing c. biguanide d. acetic acid

b. silver sulfadiazine is a metallic type of antiseptic that is widely used on burns. The silver in the solution is toxic to bacteria, and prevents them from reproducing.

The nurse is administering fluids intravenously as prescribed to a client who sustained superficial partial thickness burn injuries of the back and legs. In evaluating the adequacy of fluid resuscitation, the nurse understands that which of the following would provide the most reliable indicator for determining the adequacy? a. vital signs b. urine output c. mental status d. peripheral pulses

b. successful or adequate fluid resuscitation in the client is signaled by stable vital signs, adequate urine output, palpable peripheral pulses and clear sensorium.

The nurse plans care for a male pt who suffered thermal burns to the entire posterior aspect of his body when he fell on an outdoor grill. Which pt need is likely to be the primary problem of this pt in the emergent phase? a. maintain tissue oxygenation b. halt progression of the burn c. maintain intravascular volume d. prevent invasion of pathogens

b. the first priority is halting the severity of the burn, to limit the depth of the burn and quick action must be a priority.

Which drugs are used to treat overflow incontinence (select all that apply)? a. Baclofen (Lioresal) b. Anticholinergic drugs c. α-Adrenergic blockers d. 5α-reductase inhibitors e. Bethanechol (Urecholine)

c, d, e. α-Adrenergic blockers block the stimulation of the smooth muscle of the bladder, 5α-reductase inhibitors decrease outlet resistance, and bethanechol enhances bladder contractions. Baclofen or diazepam is used to relax the external sphincter for reflex incontinence. Anticholinergics are used to relax bladder tone and increase sphincter tone with urge incontinence.

On assessment of the patient with a renal calculus passing down the ureter, what should the nurse expect the patient to report? a. A history of chronic UTIs b. Dull, costovertebral flank pain c. Severe, colicky back pain radiating to the groin d. A feeling of bladder fullness with urgency and frequency

c. A classic sign of the passage of a calculus down the ureter is intense, colicky back pain that may radiate into the testicles, labia, or groin and may be accompanied by mild shock with cool, moist skin. Many patients with renal stones do not have a history of chronic UTIs. Stones obstructing a calyx or at the ureteropelvic junction may produce dull costovertebral flank pain and large bladder stones may cause bladder fullness and lower obstructive symptoms.

A client with burn injury asks the nurse what the term full thickness means. The nurse should respond that burns classified as full thickness involve tissue destruction down to which level? a. epidermis b. dermis c. subcutaneous tissue d. internal organs

c. A full thickness burn involves all skin layers, including the epidermis and dermis, and may extend into the subcutaneous tissue and fat.

What is included in nursing care that applies to the management of all urinary catheters in hospitalized patients? a. Measuring urine output every 1 to 2 hours to ensure patency b. Turning the patient frequently from side to side to promote drainage c. Using strict sterile technique during irrigation and obtaining culture specimens d. Daily cleaning of the catheter insertion site with soap and water and application of lotion

c. All urinary catheters in hospitalized patients pose a very high risk for infection, especially antibiotic-resistant, health care-associated infections, and scrupulous aseptic technique is essential in the insertion and maintenance of all catheters. Routine irrigations are not performed. Turning the patient to promote drainage is recommended only for suprapubic catheters. Cleaning the insertion site with soap and water should be performed for urethral and suprapubic catheters but lotion or powder should be avoided and site care for other catheters may require special interventions.

What is the most common cause of acute pyelonephritis resulting from an ascending infection from the lower urinary tract? a. The kidney is scarred and fibrotic. b. The organism is resistant to antibiotics. c. There is a preexisting abnormality of the urinary tract. d. The patient does not take all of the antibiotics for treatment of a UTI

c. Ascending infections from the bladder to the kidney are prevented by the normal anatomy and physiology of the urinary tract unless a preexisting condition, such as vesicoureteral reflux or lower urinary tract dysfunction (bladder tumors, prostatic hyperplasia, strictures, or stones), is present. Resistance to antibiotics and failure to take a full prescription of antibiotics for a UTI usually result in relapse or reinfection of the lower urinary tract.

"What can patients at risk for renal lithiasis do to prevent the stones in many cases? a. Lead an active lifestyle b. Limit protein and acidic foods in the diet c. Drink enough fluids to produce dilute urine d. Take prophylactic antibiotics to control UTIs

c. Because crystallization of stone constituents can precipitate and unite to form a stone when in supersaturated concentrations, one of the best ways to prevent stones of any type is by drinking adequate fluids to keep the urine dilute and flowing (e.g., an output of about 2 L of urine a day). Sedentary lifestyle is a risk factor for renal stones but exercise also causes fluid loss and a need for additional fluids. Protein foods high in purine should be restricted only for the small percentage of patients with uric acid stones and although UTIs contribute to stone formation, prophylactic antibiotics are not indicated.

An older male patient visits his primary care provider because of burning on urination and production of urine that he describes as "foul smelling." The health care provider should assess the patient for what factor that may put him at risk for a urinary tract infection (UTI)? a. High-purine diet b. Sedentary lifestyle c. Benign prostatic hyperplasia (BPH) d. Recent use of broad-spectrum antibiotics

c. Benign prostatic hyperplasia (BPH) BPH causes urinary stasis, which is a predisposing factor for UTIs. A sedentary lifestyle and recent antibiotic use are unlikely to contribute to UTIs, whereas a diet high in purines is associated with renal calculi.

Which type of urinary tract calculi are the most common and frequently obstruct the ureter? a. Cystine b. Uric acid c. Calcium oxalate d. Calcium phosphate

c. Calcium oxalate calculi are most common and small enough to get trapped in the ureter.

The patient has scleroderma and is experiencing hypertension. The nurse should know that this could be related to which renal problem? a. Obstructive uropathy b. Goodpasture syndrome c. Chronic glomerulonephritis d. Calcium oxalate urinary calculi

c. Chronic glomerulonephritis Hypertension occurs with chronic glomerulonephritis that may be found in patients with scleroderma. Obstructive uropathy, Goodpasture syndrome, and calcium oxalate urinary calculi are not related to scleroderma and do not cause hypertension.

Which instruction should the nurse provide when teaching a patient to exercise the pelvic floor? a. Tighten both buttocks together. b. Squeeze thighs together tightly. c. Contract muscles around rectum. d. Lie on back and lift legs together.

c. Contract muscles around rectum. To teach pelvic floor exercises, or Kegel exercise, the nurse should instruct the patient (without contracting the legs, buttocks, or abdomen) to contract the muscles around the rectum (pelvic floor muscles) as if stopping a stool, which should result in a pelvic lifting sensation.

When caring for a patient with an electrical burn injury, the nurse should question a health care provider's order for A. Mannitol 75 gm IV. B. Urine for myoglobulin. C. Lactated Ringer's at 25 ml/hr. D. Sodium bicarbonate 24 mEq every 4 hours.

c. Electrical injury puts the patient at risk for myoglobinuria, which can lead to acute renal tubular necrosis (ATN). Treatment consists of infusing lactated Ringer's at a rate sufficient to maintain urinary output at 75 to 100 ml/hr. Mannitol can also be used to maintain urine output. Sodium bicarbonate may be given to alkalinize the urine. The urine would also be monitored for the presence of myoglobin. An infusion rate of 25 ml/hr is not sufficient to maintain adequate urine output in prevention and treatment of ATN.

A nurse is caring for a patient with second- and third-degree burns to 50% of the body. The nurse prepares fluid resuscitation based on knowledge of the Parkland (Baxter) formula that includes which of the following recommendations? A. The total 24-hour fluid requirement should be administered in the first 8 hours. B. One half of the total 24-hour fluid requirement should be administered in the first 8 hours. C. One third of the total 24-hour fluid requirement should be administered in the first 4 hours. D. One half of the total 24-hour fluid requirement should be administered in the first 4 hours.

c. Fluid resuscitation with the Parkland (Baxter) formula recommends that one half of the total fluid requirement should be administered in the first 8 hours, one quarter of total fluid requirement should be administered in the second 8 hours, and one quarter of total fluid requirement should be administered in the third 8 hours.

Glomerulonephritis is characterized by glomerular damage caused by a. growth of microorganisms in the glomeruli. b. release of bacterial substances toxic to the glomeruli. c. accumulation of immune complexes in the glomeruli. d. hemolysis of red blood cells circulating in the glomeruli.

c. Glomerulonephritis is not an infection but rather an antibody-induced injury to the glomerulus, where either autoantibodies against the glomerular basement membrane (GBM) directly damage the tissue or antibodies reacting with nonglomerular antigens are randomly deposited as immune complexes along the GBM. Prior infection by bacteria or viruses may stimulate the antibody production but is not present or active at the time of glomerular damage.

To assist the patient with stress incontinence, what is the best thing the nurse should teach the patient to do? a. Void every 2 hours to prevent leakage. b. Use absorptive perineal pads to contain urine. c. Perform pelvic floor muscle exercises 40 to 50 times per day. d. Increase intraabdominal pressure during voiding to empty the bladder completely.

c. Pelvic floor exercises (Kegel exercises) increase the tone of the urethral sphincters and should be done in sets of 10 or more contractions four to five times a day (total of 40 to 50 per day). Frequent bladder emptying is recommended for patients with urge incontinence and an increase in pressure on the bladder is recommended for patients with overflow incontinence. Absorptive perineal pads should be only a temporary measure because long- term use discourages continence and can lead to skin problems.

When caring for a patient with nephrotic syndrome, the nurse should know the patient understands dietary teaching when the patient selects which food item? a. Peanut butter and crackers b. One small grilled pork chop c. Salad made of fresh vegetables d. Spaghetti with canned spaghetti sauce

c. Salad made of fresh vegetables Of the options listed, only salad made with fresh vegetables would be acceptable for the diet that limits sodium and protein as well as saturated fat if hyperlipidemia is present. Peanut butter and crackers are processed so they contain significant sodium, and peanut butter contains some protein. A pork chop is a high-protein food with saturated fat. Canned spaghetti sauce is also high in sodium.

What results in the edema associated with nephrotic syndrome? a. Hypercoagulability b. Hyperalbuminemia c. Decreased plasma oncotic pressure d. Decreased glomerular filtration rate

c. The massive proteinuria that results from increased glomerular membrane permeability in nephrotic syndrome leaves the blood without adequate proteins (hypoalbuminemia) to create an oncotic colloidal pressure to hold fluid in the vessels. Without oncotic pressure, fluid moves into the interstitium, causing severe edema. Hypercoagulability occurs in nephrotic syndrome but is not a factor in edema formation and glomerular filtration rate (GFR) is not necessarily affected in nephrotic syndrome.

While caring for a 77-year-old woman who has a urinary catheter, the nurse monitors the patient for the development of a UTI. What clinical manifestations is the patient most likely to experience? a. Cloudy urine and fever b. Urethral burning and bloody urine c. Vague abdominal discomfort and disorientation d. Suprapubic pain and slight decline in body temperature

c. The usual classic manifestations of UTI are often absent in older adults, who tend to experience nonlocalized abdominal discomfort and cognitive impairment characterized by confusion or decreased level of consciousness rather than dysuria and suprapubic pain.

Thirty percent of patients with kidney cancer have metastasis at the time of diagnosis. Why does this occur? a. The only treatment modalities for the disease are palliative. b. Diagnostic tests are not available to detect tumors before they metastasize. c. Classic symptoms of hematuria and palpable mass do not occur until the disease is advanced. d. Early metastasis to the brain impairs the patient's ability to recognize the seriousness of symptoms.

c. There are no early characteristic symptoms of cancer of the kidney and gross hematuria, flank pain, and a palpable mass do not occur until the disease is advanced. The treatment of choice is a partial or radical nephrectomy, which can be successful in early disease. Many kidney cancers are diagnosed as incidental imaging findings. Targeted therapy is the preferred treatment for metastatic kidney cancer. Radiation is palliative. The most common sites of metastases are the lungs, liver, and long bones.

A woman with no history of UTIs who is experiencing urgency, frequency, and dysuria comes to the clinic, where a dipstick and microscopic urinalysis indicate bacteriuria. What should the nurse anticipate for this patient? a. Obtaining a clean-catch midstream urine specimen for culture and sensitivity b. No treatment with medication unless she develops fever, chills, and flank pain c. Empirical treatment with trimethoprim-sulfamethoxazole (TMP-SMX, Bactrim) for 3 days d. Need to have a blood specimen drawn for a complete blood count (CBC) and kidney function tests

c. Unless a patient has a history of recurrent UTIs or a complicated UTI, trimethoprim-sulfamethoxazole (TMP- SMX) or nitrofurantoin (Macrodantin) is usually used to empirically treat an initial UTI without a culture and sensitivity or other testing. Asymptomatic bacteriuria does not justify treatment but symptomatic UTIs should always be treated.

A child has just been admitted to the pediatric burn unit. Currently, the child is being evaluated for burns to his chest and upper legs. He complains of thirst and asks for a drink. What is the most appropriate nursing action? a. give a small glass of clear liquid b. give a small glass of a full liquid c. keep the child NPO d. order a pediatric meal tray with extra liquids

c. Until a complete assessment and treatment plan are initiated, the child should be kept NPO. A complication of major burns is paralytic ileus, so until that has been ruled out, oral fluids should not be provided.

A nurse is caring for a client with a new donor site that was harvested to treat a burn. The nurse should position the client to: a. allow ventilation of the site b. make the site dependent c. avoid pressure on the site d. keep the site fully covered

c. a universal concern in the care of donor sites for burn care is to keep the site away from sources of pressure.

The adult client was burned as a result of an explosion. The burn initially affected the client's entire face (anterior half of the head) and the upper half of the anterior torso, and there were circumferential burns to the lower half of both arms. The client's clothes caught on fire, and the client ran, causing subsequent burn injuries to the posterior surface of the head and the upper half of the posterior torso. Using the rule of nines, what would be the extent of the burn injury? a. 18% b. 24% c. 36% d. 48%

c. anterior head = 4.5%, upper half of anterior torso = 9%, lower half of both arms is 9%, posterior head 4.5%, upper half of posterior torso 9%, total 36%

The nurse is caring for a client following an autograft and grafting to a burn would on the right knee. Which of the following would the nurse anticipate to be prescribed for the client? a. out of bed b. brp c. Immobilization of the affected leg d. placing the affected leg in a dependent position

c. autografts placed over joints or on the lower extremities after surgery often are elevated and immobilized for 3-7 days. this period allows the autograft time to adhere to the wound bed.

The condition of a client with extensive third degree burns begins to deteriorate. The nurse is aware that which type of shock may occur as a result of inadequate circulating blood volume that occurs with a burn injury? a. cardiogenic b. distributive c. hypovolemic d. septic

c. burns and the resulting low circulating fluid volume can cause hypovolemic shock.

What is the best method for preventing hypovolemic shock in a client admitted with severe burns? a. administering dopamine b. applying medical antishock trousers c. infusing i.v. fluids d. infusing fresh frozen plasma

c. during the early postburn period, large amounts of plasma fluid extravasates into interstitial spaces. Restoring the fluid loss is necessary to prevent hypovolemic shock; this is best accomplished with crystalloid and colloid solutions.

A client is undergoing fluid replacement after being burned on 20% of her body 12 hours ago. The nursing assessment reveals a blood pressure of 90/50, a pulse of 110, and urine output of 20 mL over the past hour. The nurse reports the findings to the physician and anticipates which of the following prescriptions? a. transfusing 1 unit of packed red blood cells b. administering a diuretic to increase urine output c. increasing the amount of IV lactated Ringers solution administered per hour d. changing the IV lactated Ringer's solution to one that contains dextrose in water.

c. fluid management during the first 24 hours following a burn injury generally includes the infusion of LR solution. Fluid resuscitation is determined by urine output and hourly urine output should be at least 30mL/hr. The client's urine output is indicative of insufficient fluid resuscitation, which places the client at risk for inadequate perfusion of the brain, heart, kidneys, and other body organs. Therefore, should expect ↑ of LR's.

The nurse manager is observing a new nursing graduate caring for a burn client in protective isolation. The nurse manager intervenes if the new nursing graduate planned to implement which incorrect component of protective isolation technique? a. using sterile sheets and linens b. performing strict hand washing technique c. wearing gloves and gown only when giving direct care to the client d. wearing protective garb, including a mask, gloves, cap, shoe covers, gowns, and plastic apron

c. ppe should be worn whenever entering the client's room

A female pt in the acute phase of burn care has electrical burns on the left side of her body, type 2 diabetes mellitus, and a serum glucose of 485 mg/dL. What is the nurse's priority for preventing a life threatening complication of hyperglycemia for the burn patient? a. replace the blood lost b. maintain a neutral pH c. Maintain fluid balance d. Replace serum potassium

c. this pt is most likely experiencing hyperglycemic hyperosmolar nonketotic syndrome (HHNKS) which dehydrates a patient rapidly. This increases the pt's risk for hypovolemia and hypotension.

When monitoring initial fluid replacement for the patient with 40% TBSA deep partial-thickness and full-thickness burns, which finding is of most concern to the nurse? a) Serum K+ of 4.5 mEq/L b) Urine output of 35 mL/hr c) Decreased bowel sounds d) Blood pressure of 86/72 mm Hg

d Rationale: Adequacy of fluid replacement is assessed by urine output and cardiac parameters. Urine output should be 0.5 to 1 mL/kg/hr. Mean arterial pressure should be >65 mm Hg, systolic BP >90 mm Hg, and heart rate <120 beats/min. A blood pressure of 86/72 indicates inadequate fluid replacement. However, the MAP is calculated at 77 mm Hg.

A patient who is admitted to a burn unit is hypovolemic. A new nurse asks an experienced nurse about the patient's condition. Which response if made by the experienced nurse is most appropriate? a) "Blood loss from burned tissue is the most likely cause of hypovolemia." b) "Third spacing of fluid into fluid-filled vesicles is usually the cause of hypovolemia." c) "The usual cause of hypovolemia is vaporation of fluid from denuded body surfaces." d) "Increased capillary permeability causes fluid shifts out of blood vessels and results in hypovolemia."

d Rationale: Hypovolemic shock is caused by a massive shift of fluids out of the blood vessels as a result of increased capillary permeability. Water, sodium, and plasma proteins move into interstitial spaces and other surrounding tissue.

During the emergent phase of burn injury, the nurse assesses for the presence of hypovolemia. In burn patients, hypovolemia occurs primarily as a result of a. Blood loss from injured tissue. b. Third spacing of fluid into fluid-filled vesicles. c. Evaporation of fluid from denuded body surfaces. d. Capillary permeability with fluid shift to the interstitium.

d Rationale: Hypovolemic shock is caused by a massive shift of fluids out of the blood vessels as a result of increased capillary permeability. Water, sodium, and plasma proteins move into interstitial spaces and other surrounding tissue.

The nurse plans emergent care for four male pt's who have burns covering between 40-50% of the total body surface area. Rank these patients according to their risk for an inhalation injury beginning with the pt who has the highest risk. a. has posterior chemical burns from an exhibit at a parking lot b. has osteoporosis and electrical burns of the lower extremities c. has thermal burns of the right side and is a volunteer fireman d. has chronic bronchitis and thermal burns around the abdomen

d, c, a, b.

Which characteristics are associated with urge incontinence (select all that apply)? a. Treated with Kegel exercises b. Found following prostatectomy c. Common in postmenopausal women d. Involuntary urination preceded by urgency e. Caused by overactivity of the detrusor muscle f. Bladder contracts by reflex, overriding central inhibition

d, e, f. Urge incontinence is involuntary urination preceded by urgency caused by overactivity of the detrusor muscle when the bladder contracts by reflex, which overrides central inhibition. Treatment includes treating the underlying cause and retraining the bladder with urge suppression, anticholinergic drugs, or containment devices. The other options are characteristic of stress incontinence. Patients may have a combination of urge and stress incontinence.

The client arrives at the emergency department following a burn injury that occurred in the basement at home and an inhalation injury is suspected. Which of the following would the nurse anticipate to be prescribed for the client? a. 100% oxygen via an aerosol mask b. Oxygen via nasal cannula at 15L/min c. Oxygen via nasal cannula at 10L/min d. 100% oxygen via a tight fitting, non rebreather face mask

d.

"Which test is required for a diagnosis of pyelonephritis? a. Renal biopsy b. Blood culture c. Intravenous pyelogram (IVP) d. Urine for culture and sensitivity

d. A urine specimen specifically obtained for culture and sensitivity is required to diagnose pyelonephritis because it will show pyuria, the specific bacteriuria, and what drug the bacteria is sensitive to for treatment. The renal biopsy is used to diagnose chronic pyelonephritis or cancer. Blood cultures would be done if bacteremia is suspected. Intravenous pyelogram (IVP) would increase renal irritation, but CT urograms may be used to assess for signs of infection in the kidney and complications of pyelonephritis.

What manifestation in the patient will indicate the need for restriction of dietary protein in management of acute poststreptococcal glomerulonephritis (APSGN)? a. Hematuria b. Proteinuria c. Hypertension d. Elevated blood urea nitrogen (BUN)

d. An elevated blood urea nitrogen (BUN) indicates that the kidneys are not clearing nitrogenous wastes from the blood and protein may be restricted until the kidney recovers. Proteinuria indicates loss of protein from the blood and possibly a need for increased protein intake. Hypertension is treated with sodium and fluid restriction, diuretics, and antihypertensive drugs. The hematuria is not specifically treated.

When caring for the patient with interstitial cystitis, what can the nurse teach the patient to do? a. Avoid foods that make the urine more alkaline. b. Use high-potency vitamin therapy to decrease the autoimmune effects of the disorder. c. Always keep a voiding diary to document pain, voiding frequency, and patterns of nocturia. d. Use the dietary supplement calcium glycerophosphate (Prelief) to decrease bladder irritation.

d. Calcium glycerophosphate (Prelief) alkalinizes the urine and can help to relieve the irritation from acidic foods. A diet low in acidic foods is recommended and if a multivitamin is used, high-potency vitamins should be avoided because these products may irritate the bladder. A voiding diary is useful in diagnosis but does not need to be kept indefinitely.

The nurse is providing care for a patient who has been admitted to the hospital for the treatment of nephrotic syndrome. What are priority nursing assessments in the care of this patient? a. Assessment of pain and level of consciousness b. Assessment of serum calcium and phosphorus levels c. Blood pressure and assessment for orthostatic hypotension d. Daily weights and measurement of the patient's abdominal girth

d. Daily weights and measurement of the patient's abdominal girth Peripheral edema is characteristic of nephrotic syndrome, and a key nursing responsibility in the care of patients with the disease is close monitoring of abdominal girth, weights, and extremity size. Pain, level of consciousness, and orthostatic blood pressure are less important in the care of patients with nephrotic syndrome. Abnormal calcium and phosphorus levels are not commonly associated with the diagnosis of nephrotic syndrome.

The nurse is caring for a 73-year-old man patient with a history of benign prostatic hyperplasia and symptoms of a possible urinary tract infection. Which diagnostic finding would support this diagnosis? a. White blood cell count is 7500 cells/µL. b. Antistreptolysin-O (ASO) titer is 106 Todd units/mL. c. Glucose, protein, and ketones are present in the urine. d. Nitrites and leukocyte esterase are present in the urine.

d. Nitrites and leukocyte esterase are present in the urine. A diagnosis of urinary tract infection is suspected if there are nitrites (indicating bacteriuria), white blood cells (WBCs), and leukocyte esterase (an enzyme present in WBCs indicating pyuria). Presence of glucose and ketones indicate uncontrolled diabetes mellitus. An elevated WBC count (>11,000 cells/µL) indicates a bacterial infection. Antistreptolysin-O (ASO) titer is a blood test to measure antibodies against streptolysin O, a substance produced by group A Streptococcus bacteria.

Prevention of calcium oxalate stones would include dietary restriction of which foods or drinks? a. Milk and milk products b. Dried beans and dried fruits c. Liver, kidney, and sweetbreads d. Spinach, cabbage, and tomatoes

d. Oxalate-rich foods should be limited to reduce oxalate excretion. Foods high in oxalate include spinach, rhubarb, asparagus, cabbage, and tomatoes, in addition to chocolate, coffee, and cocoa. Currently, it is believed that high dietary calcium intake may actually lower the risk for renal stones by reducing the intestinal oxalate absorption and therefore the urinary excretion of oxalate. Milk, milk products, dried beans, and dried fruits are high sources of calcium. Organ meats are high in purine, which contributes to uric acid lithiasis.

Which disease causes connective tissue changes that cause glomerulonephritis? a. Gout b. Amyloidosis c. Diabetes mellitus d. Systemic lupus erythematosus

d. Systemic lupus erythematosus causes connective tissue damage that affects the glomerulus. Gout deposits uric acid crystals in the kidney. Amyloidosis deposits hyaline bodies in the kidney. Diabetes mellitus causes microvascular damage affecting the kidney.

A victim of an industrial accident has chemical spilled on his face and body. The chemical, which has a pH of 7.51, is flushed with water by paramedics. What is the most important information for the receiving nurse to obtain about the pt from the paramedics? a. containment of chemical b. duration of water flushing c. other injuries of the victim d. specific location of accident

d. The nurse must know where the accident occurred to determine if the pt was rescued from an enclosed space. If so, the pt is at high risk for an inhalation injury because the enclosure concentrates the noxious fumes making an inhalation injury more likely.

A patient with suprapubic pain and symptoms of urinary frequency and urgency has two negative urine cultures. What is one assessment finding that would indicate interstitial cystitis? a. Residual urine greater than 200 mL b. A large, atonic bladder on urodynamic testing c. A voiding pattern that indicates psychogenic urinary retention d. Pain with bladder filling that is transiently relieved by urination

d. The symptoms of interstitial cystitis (IC) imitate those of an infection of the bladder but the urine is free of infectious agents. Unlike a bladder infection, the pain with IC increases as urine collects in the bladder and is temporarily relieved by urination. Acidic urine is very irritating to the bladder in IC and the bladder is small but urinary retention is not common.

"A patient with bladder cancer undergoes cystectomy with formation of an ileal conduit. During the patient's first postoperative day, what should the nurse plan to do? a. Measure and fit the stoma for a permanent appliance. b. Encourage high oral intake to flush mucus from the conduit. c. Teach the patient to self-catheterize the stoma every 4 to 6 hours. d. Empty the drainage bag every 2 to 3 hours and measure the urinary output.

d. Urine drains continuously from an ileal conduit and the drainage bag must be emptied every 2 to 3 hours and measured to ensure adequate urinary output. Fitting for a permanent appliance is not done until the stoma shrinks to its normal size in a few weeks. With an ileal conduit, mucus is present in the urine because it is secreted by the ileal segment as a result of the irritating effect of the urine but the surgery causes paralytic ileus and the patient will be NPO for several days postoperatively. Self-catheterization is performed when patients have formation of a continent Kock pouch.

A male pt suffered full thickness burns to the chest and back and the nurse notes the pressure alarm on his mechanical ventilator is sounding every 5 minutes. What is the most relevant assessment to prevent respiratory complications in this pt? a. pH b. PaCO2 c. Breath sounds d. chest expansion

d. assessing the pt's chest expansion is initially the most important because a sever burn that includes the anterior and posterior thorax can restrict chest expansion from eschar or scar tissue.

The nurse is caring for a client who sustained superficial partial thickness burns on the anterior lower legs and anterior thorax. Which of the following does the nurse expect to note during the resuscitation/emergent phase of the burn injury? a. decreased heart rate b. increased urinary output c. increased blood pressure d. elevated hematocrit levels

d. during the resuscitation/emergent phase, the hematocrit level increases to above normal because of hemoconcentration from the large fluid shift.

The patient is wondering why anesthesia is needed when the lithotripsy being done is noninvasive. The nurse explains that the anesthesia is required to ensure the patient's position is maintained during the procedure. The nurse knows that this type of lithotripsy is called a. laser lithotripsy. b. electrohydraulic lithotripsy. c. percutaneous ultrasonic lithotripsy. d. extracorporeal shock-wave lithotripsy (ESWL).

d. extracorporeal shock-wave lithotripsy (ESWL). ESWL is noninvasive, but anesthesia is used to maintain the patient's position. The other types of lithotripsy are invasive. Laser lithotripsy uses an ureteroscope and small fiber to reach the stone. Electrohydraulic lithotripsy positions a probe directly on the stone; then continuous saline irrigation flushes are used to rinse the stone out. Percutaneous ultrasonic lithotripsy places an ultrasonic probe in the renal pelvis via a percutaneous nephroscope inserted through an incision in the flank.

Delegation Decision: Which nursing interventions could be delegated to unlicensed assistive personnel (UAP) (select all that apply)? a. Assess the need for catheterization. b. Use bladder scanner to estimate residual urine. c. Teach patient pelvic floor muscle (Kegel) exercises. d. Insert indwelling catheter for uncomplicated patient. e. Assist incontinent patient to commode at regular intervals. f. Provide perineal care with soap and water around a urinary catheter.

e, f. The unlicensed assistive personnel (UAP) may assist the incontinent patient to void at regular intervals and provide perineal care. An RN should perform the assessments and teaching. In long-term care and rehabilitation facilities, UAP may use bladder scanners after they are trained.


Set pelajaran terkait

chapter 5 - corporate governance

View Set

Research Exam 2 Ch. 7, 11, 14, & 15

View Set

Ch 18 Cost Behavior and Cost Volume Profit Analysis

View Set

Psychology: Intro to Psychology Chapters 9-12

View Set

Chapter 4 Section 1 - Geography of Ancient Egypt

View Set

Organizing the Body of the Speech; Beginning and Ending the Speech; Outlining the Speech

View Set

GNED 06 Lecture 2: Historical Antecedents of the World Transcript

View Set

chapter seven part 4 amino acids

View Set